分からない問題はここに書いてね374

このエントリーをはてなブックマークに追加
1132人目の素数さん
さあ、今日も1日頑張ろう★☆

前スレ
分からない問題はここに書いてね373
http://uni.2ch.net/test/read.cgi/math/1343211724/
2132人目の素数さん:2012/08/17(金) 08:33:13.07
http://www.math.tohoku.ac.jp/exam/questions/exam-1999-s1.gif

これの1問目(2)を教えて下さい。イデアルの生成元が全部φで0にうつされる事からI⊂Kerはわかりますが、逆が分かりません。
3132人目の素数さん:2012/08/17(金) 09:41:44.48
割り算してみりゃすぐ分かるだろ
4132人目の素数さん:2012/08/17(金) 10:07:47.30
          __ノ)-'´ ̄ ̄`ー- 、_
        , '´  _. -‐'''"二ニニ=-`ヽ、
      /   /:::::; -‐''"        `ーノ
     /   /:::::/           \
     /    /::::::/          | | |  |
     |   |:::::/ /     |  | | | |  |
      |   |::/ / / |  | ||  | | ,ハ .| ,ハ|
      |   |/ / / /| ,ハノ| /|ノレ,ニ|ル' 
     |   |  | / / レ',二、レ′ ,ィイ|゙/   私は只の数ヲタなんかとは付き合わないわ。
.     |   \ ∠イ  ,イイ|    ,`-' |      頭が良くて数学が出来てかっこいい人。それが必要条件よ。
     |     l^,人|  ` `-'     ゝ  |        さらに Ann.of Math に論文書けば十分条件にもなるわよ。
      |      ` -'\       ー'  人          一番嫌いなのは論文数を増やすためにくだらない論文を書いて
    |        /(l     __/  ヽ、           良い論文の出版を遅らせるお馬鹿な人。
     |       (:::::`‐-、__  |::::`、     ヒニニヽ、         あなたの論文が Ann of Math に accept される確率は?
    |      / `‐-、::::::::::`‐-、::::\   /,ニニ、\            それとも最近は Inv. Math. の方が上かしら?
   |      |::::::::::::::::::|` -、:::::::,ヘ ̄|'、  ヒニ二、 \
.   |      /::::::::::::::::::|::::::::\/:::O`、::\   | '、   \
   |      /:::::::::::::::::::/:::::::::::::::::::::::::::::'、::::\ノ  ヽ、  |
  |      |:::::/:::::::::/:::::::::::::::::::::::::::::::::::'、',::::'、  /:\__/‐、
  |      |/:::::::::::/::::::::::::::::::::::::::::::::::O::| '、::| く::::::::::::: ̄|
   |     /_..-'´ ̄`ー-、:::::::::::::::::::::::::::::::::::|/:/`‐'::\;;;;;;;_|
   |    |/::::::::::::::::::::::\:::::::::::::::::::::::::::::|::/::::|::::/:::::::::::/
    |   /:::::::::::::::::::::::::::::::::|:::::::::::::::::::::O::|::|::::::|:::::::::::::::/
5132人目の素数さん:2012/08/17(金) 12:46:03.76
>>2
k[x,y,z]/I で考えると y^2→xz, x^3→yz, x^2 y→z^2 で
f(x,y,z)=f0(z)+x f1(z)+x^2 f2(z)+y f3(z)+xy f3(z) となる
φf=f0(t^5)+t^3 f1(t^5)+t^6 f2(z)+t^4 f3(t^5)+t^7 f3(t^5) の各項は
t^(5n), t^(5n+3), t^(5n+6), t^(5n+4), t^(5n+7) だから一致せず 0 にならない
6132人目の素数さん:2012/08/17(金) 16:04:37.16
          __ノ)-'´ ̄ ̄`ー- 、_
        , '´  _. -‐'''"二ニニ=-`ヽ、
      /   /:::::; -‐''"        `ーノ
     /   /:::::/           \
     /    /::::::/          | | |  |
     |   |:::::/ /     |  | | | |  |
      |   |::/ / / |  | ||  | | ,ハ .| ,ハ|
      |   |/ / / /| ,ハノ| /|ノレ,ニ|ル' 
     |   |  | / / レ',二、レ′ ,ィイ|゙/   私は只の数ヲタなんかとは付き合わないわ。
.     |   \ ∠イ  ,イイ|    ,`-' |      頭が良くて数学が出来てかっこいい人。それが必要条件よ。
     |     l^,人|  ` `-'     ゝ  |        さらに Ann.of Math に論文書けば十分条件にもなるわよ。
      |      ` -'\       ー'  人          一番嫌いなのは論文数を増やすためにくだらない論文を書いて
    |        /(l     __/  ヽ、           良い論文の出版を遅らせるお馬鹿な人。
     |       (:::::`‐-、__  |::::`、     ヒニニヽ、         あなたの論文が Ann of Math に accept される確率は?
    |      / `‐-、::::::::::`‐-、::::\   /,ニニ、\            それとも最近は Inv. Math. の方が上かしら?
   |      |::::::::::::::::::|` -、:::::::,ヘ ̄|'、  ヒニ二、 \
.   |      /::::::::::::::::::|::::::::\/:::O`、::\   | '、   \
   |      /:::::::::::::::::::/:::::::::::::::::::::::::::::'、::::\ノ  ヽ、  |
  |      |:::::/:::::::::/:::::::::::::::::::::::::::::::::::'、',::::'、  /:\__/‐、
  |      |/:::::::::::/::::::::::::::::::::::::::::::::::O::| '、::| く::::::::::::: ̄|
   |     /_..-'´ ̄`ー-、:::::::::::::::::::::::::::::::::::|/:/`‐'::\;;;;;;;_|
   |    |/::::::::::::::::::::::\:::::::::::::::::::::::::::::|::/::::|::::/:::::::::::/
7132人目の素数さん:2012/08/17(金) 16:38:49.83
1個のサイコロを4回投げるとき出る目の数を順にa,b,c,dとする
a+b+c+dが3の倍数になる確率とa≦b≦c≦dとなる確率を求めろ

おねがいします
8132人目の素数さん:2012/08/17(金) 17:08:38.60
1/3
6H4/6^6
9132人目の素数さん:2012/08/17(金) 17:25:16.38
f(n):与えられたnに、サイコロを振り出た目を加え、それが3の倍数になる確率とすると
f(0)=f(1)=f(2)=1/3
f(a)=f(a+b)=f(a+b+c)=f(a+b+c+d)=1/3

a<b<c<dの場合 C[6,4]=15
a=b<c<d,a<b=c<d,a<b<c=dの場合 C[6,3]=20 合計60
a=b=c<d,a=b<c=d,a<b=c=dの場合 C[6,2]=15 合計45
a=b=c=dの場合 C[6,1]=6
従って、(15+60+45+6)/6^4
10132人目の素数さん:2012/08/17(金) 17:33:51.46
>>5
ありがとうございます
11132人目の素数さん:2012/08/17(金) 17:40:12.37
          __ノ)-'´ ̄ ̄`ー- 、_
        , '´  _. -‐'''"二ニニ=-`ヽ、
      /   /:::::; -‐''"        `ーノ
     /   /:::::/           \
     /    /::::::/          | | |  |
     |   |:::::/ /     |  | | | |  |
      |   |::/ / / |  | ||  | | ,ハ .| ,ハ|
      |   |/ / / /| ,ハノ| /|ノレ,ニ|ル' 
     |   |  | / / レ',二、レ′ ,ィイ|゙/   私は只の数ヲタなんかとは付き合わないわ。
.     |   \ ∠イ  ,イイ|    ,`-' |      頭が良くて数学が出来てかっこいい人。それが必要条件よ。
     |     l^,人|  ` `-'     ゝ  |        さらに Ann.of Math に論文書けば十分条件にもなるわよ。
      |      ` -'\       ー'  人          一番嫌いなのは論文数を増やすためにくだらない論文を書いて
    |        /(l     __/  ヽ、           良い論文の出版を遅らせるお馬鹿な人。
     |       (:::::`‐-、__  |::::`、     ヒニニヽ、         あなたの論文が Ann of Math に accept される確率は?
    |      / `‐-、::::::::::`‐-、::::\   /,ニニ、\            それとも最近は Inv. Math. の方が上かしら?
   |      |::::::::::::::::::|` -、:::::::,ヘ ̄|'、  ヒニ二、 \
.   |      /::::::::::::::::::|::::::::\/:::O`、::\   | '、   \
   |      /:::::::::::::::::::/:::::::::::::::::::::::::::::'、::::\ノ  ヽ、  |
  |      |:::::/:::::::::/:::::::::::::::::::::::::::::::::::'、',::::'、  /:\__/‐、
  |      |/:::::::::::/::::::::::::::::::::::::::::::::::O::| '、::| く::::::::::::: ̄|
   |     /_..-'´ ̄`ー-、:::::::::::::::::::::::::::::::::::|/:/`‐'::\;;;;;;;_|
   |    |/::::::::::::::::::::::\:::::::::::::::::::::::::::::|::/::::|::::/:::::::::::/
12132人目の素数さん:2012/08/18(土) 01:14:31.83
解ける人いませんか?友人に出された問題です


数列の問題です。

数列:57、51、109、123、119、104、51、43、53、?

?に当てはまる数字は何でしょうか。 
13132人目の素数さん:2012/08/18(土) 01:15:18.38
          __ノ)-'´ ̄ ̄`ー- 、_
        , '´  _. -‐'''"二ニニ=-`ヽ、
      /   /:::::; -‐''"        `ーノ
     /   /:::::/           \
     /    /::::::/          | | |  |
     |   |:::::/ /     |  | | | |  |
      |   |::/ / / |  | ||  | | ,ハ .| ,ハ|
      |   |/ / / /| ,ハノ| /|ノレ,ニ|ル' 
     |   |  | / / レ',二、レ′ ,ィイ|゙/   私は只の数ヲタなんかとは付き合わないわ。
.     |   \ ∠イ  ,イイ|    ,`-' |      頭が良くて数学が出来てかっこいい人。それが必要条件よ。
     |     l^,人|  ` `-'     ゝ  |        さらに Ann.of Math に論文書けば十分条件にもなるわよ。
      |      ` -'\       ー'  人          一番嫌いなのは論文数を増やすためにくだらない論文を書いて
    |        /(l     __/  ヽ、           良い論文の出版を遅らせるお馬鹿な人。
     |       (:::::`‐-、__  |::::`、     ヒニニヽ、         あなたの論文が Ann of Math に accept される確率は?
    |      / `‐-、::::::::::`‐-、::::\   /,ニニ、\            それとも最近は Inv. Math. の方が上かしら?
   |      |::::::::::::::::::|` -、:::::::,ヘ ̄|'、  ヒニ二、 \
.   |      /::::::::::::::::::|::::::::\/:::O`、::\   | '、   \
   |      /:::::::::::::::::::/:::::::::::::::::::::::::::::'、::::\ノ  ヽ、  |
  |      |:::::/:::::::::/:::::::::::::::::::::::::::::::::::'、',::::'、  /:\__/‐、
  |      |/:::::::::::/::::::::::::::::::::::::::::::::::O::| '、::| く::::::::::::: ̄|
   |     /_..-'´ ̄`ー-、:::::::::::::::::::::::::::::::::::|/:/`‐'::\;;;;;;;_|
   |    |/::::::::::::::::::::::\:::::::::::::::::::::::::::::|::/::::|::::/:::::::::::/
14132人目の素数さん:2012/08/18(土) 01:24:38.37
>>12
常識的に考えて59
15132人目の素数さん:2012/08/18(土) 01:32:25.71
>>14
解説もお願いできますか?
16132人目の素数さん:2012/08/18(土) 02:27:25.85
>>13
どんな数字でも当てはまる
17132人目の素数さん:2012/08/18(土) 10:41:41.98
>>12
クイズはいらね
18132人目の素数さん:2012/08/18(土) 12:22:21.16
友人に聞けばいいだろjk
19132人目の素数さん:2012/08/18(土) 14:05:56.47
55
20132人目の素数さん:2012/08/18(土) 15:23:59.56
          __ノ)-'´ ̄ ̄`ー- 、_
        , '´  _. -‐'''"二ニニ=-`ヽ、
      /   /:::::; -‐''"        `ーノ
     /   /:::::/           \
     /    /::::::/          | | |  |
     |   |:::::/ /     |  | | | |  |
      |   |::/ / / |  | ||  | | ,ハ .| ,ハ|
      |   |/ / / /| ,ハノ| /|ノレ,ニ|ル' 
     |   |  | / / レ',二、レ′ ,ィイ|゙/   私は只の数ヲタなんかとは付き合わないわ。
.     |   \ ∠イ  ,イイ|    ,`-' |      頭が良くて数学が出来てかっこいい人。それが必要条件よ。
     |     l^,人|  ` `-'     ゝ  |        さらに Ann.of Math に論文書けば十分条件にもなるわよ。
      |      ` -'\       ー'  人          一番嫌いなのは論文数を増やすためにくだらない論文を書いて
    |        /(l     __/  ヽ、           良い論文の出版を遅らせるお馬鹿な人。
     |       (:::::`‐-、__  |::::`、     ヒニニヽ、         あなたの論文が Ann of Math に accept される確率は?
    |      / `‐-、::::::::::`‐-、::::\   /,ニニ、\            それとも最近は Inv. Math. の方が上かしら?
   |      |::::::::::::::::::|` -、:::::::,ヘ ̄|'、  ヒニ二、 \
.   |      /::::::::::::::::::|::::::::\/:::O`、::\   | '、   \
   |      /:::::::::::::::::::/:::::::::::::::::::::::::::::'、::::\ノ  ヽ、  |
  |      |:::::/:::::::::/:::::::::::::::::::::::::::::::::::'、',::::'、  /:\__/‐、
  |      |/:::::::::::/::::::::::::::::::::::::::::::::::O::| '、::| く::::::::::::: ̄|
   |     /_..-'´ ̄`ー-、:::::::::::::::::::::::::::::::::::|/:/`‐'::\;;;;;;;_|
   |    |/::::::::::::::::::::::\:::::::::::::::::::::::::::::|::/::::|::::/:::::::::::/
21132人目の素数さん:2012/08/18(土) 19:54:44.41
数年ぶりに覗いたが、>>1にAAないんだな
22132人目の素数さん:2012/08/18(土) 19:57:32.15
>>12
任意の数字が入るお
そのように一般項を構成できるお
23132人目の素数さん:2012/08/18(土) 21:25:44.03
すいません。
「半群 G に単位元 e が存在する場合は唯だ一つ」
に納得できません。
e, e' が存在するとして、単位元の定義 ae = ea = a (∀a)に代入すると
e'e = ee' = e'
となって、ここからどう推移すれば e' = e になるのでしょう?
24132人目の素数さん:2012/08/18(土) 21:29:00.43
e と e' はまったく同じ条件
ee' = e'e = e
25132人目の素数さん:2012/08/18(土) 21:39:47.97
>>24
感覚的にしか思えないのですが‥‥‥
26132人目の素数さん:2012/08/18(土) 21:41:32.34
ならこれも同じく感覚的に見えるのか
e'e = ee' = e'
27132人目の素数さん:2012/08/18(土) 21:44:04.28
>>25
e, e' にそれぞれの定義をあてはめて二つの式を準備すればよい
ae = ea = a(∀a)
ae'=e'a=a(∀a)
お互いをお互いの式に代入すればよい
e'e=ee'=e'
ee'=e'e=e
よって
e=e'
28132人目の素数さん:2012/08/18(土) 21:48:14.36
AAに見える
29132人目の素数さん:2012/08/18(土) 21:50:12.69
a(・∀・a)
30Sengoku18:2012/08/18(土) 22:43:02.88
eが唯一であるためには、
結合則 SxS−>S
だけがあればよい。?

A monoid is a set G, with a low of composition which is associative and having a unit element.

31132人目の素数さん:2012/08/18(土) 22:44:37.24
くたばれ 仙石!
32132人目の素数さん:2012/08/18(土) 22:51:12.28
>>30
いらない。
33132人目の素数さん:2012/08/18(土) 22:59:48.51
>>30
いろいろやってみたが、今のところトートロジに陥りぬけだせない
可換であることは必要みたい
34132人目の素数さん:2012/08/18(土) 23:02:55.59
いらない。
35132人目の素数さん:2012/08/18(土) 23:04:33.62
うーん、可換抜きでは抜け出せないな‥‥どういう道筋だろう?
36132人目の素数さん:2012/08/18(土) 23:04:36.18
 associative でない反軍とは?
37132人目の素数さん:2012/08/18(土) 23:08:13.95
集合Aに2項演算 ・:AxA→Aが定義されている。
元 e∈A は任意のa∈Aに対し、a・e=e・a=aを満たすとき演算・に関するAの単位元という。

これだけ。
38132人目の素数さん:2012/08/18(土) 23:10:35.38
教科書よめばすむことを

ひまだね
39132人目の素数さん:2012/08/18(土) 23:11:41.97
教科書(笑)
40132人目の素数さん:2012/08/18(土) 23:12:55.21
>>39
なんで?
41132人目の素数さん:2012/08/18(土) 23:13:10.89
数学Tの問題です

3x^2+(y+6)x-(2y-3)(y+1)

これの因数分解ってどうやるんですか?
たすき掛けを使うみたいですけどよくわかりません
42132人目の素数さん:2012/08/18(土) 23:16:47.91
>>36
結合則を見たさなければ、定義上、それは半群ではないのでは?
43132人目の素数さん:2012/08/18(土) 23:19:53.10
>>40
教科書という言葉が、微笑ましくてさ。
44132人目の素数さん:2012/08/18(土) 23:20:25.09
>>37
その定義、ae = ea と単位元について可換であるように定義されているけれども、もしかして要らないのでは?という話
教科書から逸脱して考えてみるのも悪くないのでは?

しかし今のところ私んところでは、可換でないと唯一性は導けていない。
45132人目の素数さん:2012/08/18(土) 23:25:23.05
>>43
独力で証明できるやつはそんなにいないだろ

「代数入門の本」の方がよかった?
46132人目の素数さん:2012/08/18(土) 23:28:01.29
>>41
3x^2+(y+6)x-(2y-3)(y+1)

3  -(2y-3) -2y+3
 X           y+6
1  (y+1) 3y+3

3  (2y-3) 2y-3
 X           -y-6
1  -(y+1) -3y-1

3  (y+1) y+1
 X           -5y-8
1  -(2y-3) -6y-9

3  -(y+1) -y-1
 X           5y+8
1  (2y-3) 6y+9

以上から

(3x-(2y-3))(x+(y+1))
47132人目の素数さん:2012/08/18(土) 23:32:37.16
>>46さんくす!
48132人目の素数さん:2012/08/18(土) 23:34:53.08
>>44
半群であるなら、右単位元と右擬逆元があれば群である、という有名な命題はあるね。
49132人目の素数さん:2012/08/18(土) 23:37:54.16
>>47
御免。たすきの記述が間違っていた。
以下に訂正

3  -(2y-3) -2y+3
 X           y+6
1  (y+1) 3y+3

3  (2y-3) 2y-3
 X           -y-6
1  -(y+1) -3y-3

3  (y+1) y+1
 X           -5y+10
1  -(2y-3) -6y+9

3  -(y+1) -y-1
 X           5y-10
1  (2y-3) 6y-9
50132人目の素数さん:2012/08/19(日) 00:05:07.49
associativeじゃない半群はmagmaじゃない?
51132人目の素数さん:2012/08/19(日) 00:11:56.73
>>50
だから associative って「結合的」 でしょ?
結合的でなければ、そもそも半群とは呼ばないような気がするんですが、専門的にはそういうのもありなんですか?
52132人目の素数さん:2012/08/19(日) 00:17:10.26
associativeなmagmaが半群
演算は定義されてて閉じてる集合がmagma
53132人目の素数さん:2012/08/19(日) 00:49:21.77
semigroups―magmas where the operation is associative;
monoids―semigroups with identity elements;
groups―monoids with inverse elements, or equivalently, associative loops or associative quasigroups;
54万石17:2012/08/19(日) 00:51:22.97
identity element があれば それはただ一個である。
55ベータ:2012/08/19(日) 00:52:32.66
しね! 仙石一族!
56132人目の素数さん:2012/08/19(日) 01:26:58.80
>>33
(1)右単位元 eR の存在 ∀x∈G [x eR=x]
(2)左単位元 eL の存在 ∀x∈G [eL x=x]
(1)から eL eR=eL, (2)から eL eR=eR ∴ eL=eR
別の右単位元 eR' があっても同じ論法で eL=eR' ∴ eR'=eR
別の左単位元 eL' があっても同じ論法で eL'=eR ∴ eL'=eL
57132人目の素数さん:2012/08/19(日) 02:21:15.78
>>56
それは右単位元だけ存在する、とか左単位元だけ存在する、という場合にはうまくいかないですよね。
右単位元と左単位元が存在すれば、両方とも同じ値。この場合即、同時に(同じ論法で)単位元は唯一になるわけですか。
58132人目の素数さん:2012/08/19(日) 05:22:11.17
y=x^2の傾き2の動径PQの中点Mの軌跡を求めろ

お願いします
59132人目の素数さん:2012/08/19(日) 05:43:31.57
>>58
は?
60132人目の素数さん:2012/08/19(日) 05:43:45.97
動径じゃなくて弦だろ
61132人目の素数さん:2012/08/19(日) 09:54:55.42
n^2を4で割った余りは1または0であることの証明(nは整数)について
n=1のときはどうすればいいんでしょうか
62132人目の素数さん:2012/08/19(日) 09:56:18.09
>>61
n=1のとき
n^2=1^2=1=4*0+1
6361:2012/08/19(日) 09:59:37.66
>>62
ありがとうございます
余りが3になるかと勘違いしていました。吊ってきます
64132人目の素数さん:2012/08/19(日) 10:40:39.04
0<x<e<yがx^y=y^xを満たす時、yをxの関数で表したいのですが、うまくできません。逆にxをyで表すのはランベルトのW関数を用いて
x=-yW(-log(y)/y)/log(y)
となるのですが。
どなたかどのようにすればyをxで表せるか教えてください。
65132人目の素数さん:2012/08/19(日) 11:30:17.22
          __ノ)-'´ ̄ ̄`ー- 、_
        , '´  _. -‐'''"二ニニ=-`ヽ、
      /   /:::::; -‐''"        `ーノ
     /   /:::::/           \
     /    /::::::/          | | |  |
     |   |:::::/ /     |  | | | |  |
      |   |::/ / / |  | ||  | | ,ハ .| ,ハ|
      |   |/ / / /| ,ハノ| /|ノレ,ニ|ル' 
     |   |  | / / レ',二、レ′ ,ィイ|゙/   私は只の数ヲタなんかとは付き合わないわ。
.     |   \ ∠イ  ,イイ|    ,`-' |      頭が良くて数学が出来てかっこいい人。それが必要条件よ。
     |     l^,人|  ` `-'     ゝ  |        さらに Ann.of Math に論文書けば十分条件にもなるわよ。
      |      ` -'\       ー'  人          一番嫌いなのは論文数を増やすためにくだらない論文を書いて
    |        /(l     __/  ヽ、           良い論文の出版を遅らせるお馬鹿な人。
     |       (:::::`‐-、__  |::::`、     ヒニニヽ、         あなたの論文が Ann of Math に accept される確率は?
    |      / `‐-、::::::::::`‐-、::::\   /,ニニ、\            それとも最近は Inv. Math. の方が上かしら?
   |      |::::::::::::::::::|` -、:::::::,ヘ ̄|'、  ヒニ二、 \
.   |      /::::::::::::::::::|::::::::\/:::O`、::\   | '、   \
   |      /:::::::::::::::::::/:::::::::::::::::::::::::::::'、::::\ノ  ヽ、  |
  |      |:::::/:::::::::/:::::::::::::::::::::::::::::::::::'、',::::'、  /:\__/‐、
  |      |/:::::::::::/::::::::::::::::::::::::::::::::::O::| '、::| く::::::::::::: ̄|
   |     /_..-'´ ̄`ー-、:::::::::::::::::::::::::::::::::::|/:/`‐'::\;;;;;;;_|
   |    |/::::::::::::::::::::::\:::::::::::::::::::::::::::::|::/::::|::::/:::::::::::/
    |   /:::::::::::::::::::::::::::::::::|:::::::::::::::::::::O::|::|::::::|:::::::::::::::/
66132人目の素数さん:2012/08/19(日) 14:04:45.67
Pが線分AB上にあってかつAP:PB=m:nであるときPを"ABをm:nに内分する点"というらしいんですが、"AP:PB=m:n"の意味がわかりません。
適当な数を代入してみればわかると思います
例えばAが5、Bが7だとすればABを1:1に内分する点は6ですが
上の理論でいくとこれが30:42=1:1になるじゃありませんか
67132人目の素数さん:2012/08/19(日) 14:09:40.76
>>66
ゆかいゆかい
68132人目の素数さん:2012/08/19(日) 14:10:41.71
>>66ですがここでいうmはAP間の距離、nはPB間の距離を示しています
誰か助けてくれ
69132人目の素数さん:2012/08/19(日) 14:15:34.78
いま気付いたんですがもしかしてAPとかPBとか言うのはA×Pの意味ではなくA-Pの長さという事なのかな?
70132人目の素数さん:2012/08/19(日) 14:18:45.11
>>66
君の書いている「線分AB」に現れるAやBはなんなんだい?
71132人目の素数さん:2012/08/19(日) 14:20:50.75
          __ノ)-'´ ̄ ̄`ー- 、_
        , '´  _. -‐'''"二ニニ=-`ヽ、
      /   /:::::; -‐''"        `ーノ
     /   /:::::/           \
     /    /::::::/          | | |  |
     |   |:::::/ /     |  | | | |  |
      |   |::/ / / |  | ||  | | ,ハ .| ,ハ|
      |   |/ / / /| ,ハノ| /|ノレ,ニ|ル' 
     |   |  | / / レ',二、レ′ ,ィイ|゙/   私は只の数ヲタなんかとは付き合わないわ。
.     |   \ ∠イ  ,イイ|    ,`-' |      頭が良くて数学が出来てかっこいい人。それが必要条件よ。
     |     l^,人|  ` `-'     ゝ  |        さらに Ann.of Math に論文書けば十分条件にもなるわよ。
      |      ` -'\       ー'  人          一番嫌いなのは論文数を増やすためにくだらない論文を書いて
    |        /(l     __/  ヽ、           良い論文の出版を遅らせるお馬鹿な人。
     |       (:::::`‐-、__  |::::`、     ヒニニヽ、         あなたの論文が Ann of Math に accept される確率は?
    |      / `‐-、::::::::::`‐-、::::\   /,ニニ、\            それとも最近は Inv. Math. の方が上かしら?
   |      |::::::::::::::::::|` -、:::::::,ヘ ̄|'、  ヒニ二、 \
.   |      /::::::::::::::::::|::::::::\/:::O`、::\   | '、   \
   |      /:::::::::::::::::::/:::::::::::::::::::::::::::::'、::::\ノ  ヽ、  |
  |      |:::::/:::::::::/:::::::::::::::::::::::::::::::::::'、',::::'、  /:\__/‐、
  |      |/:::::::::::/::::::::::::::::::::::::::::::::::O::| '、::| く::::::::::::: ̄|
   |     /_..-'´ ̄`ー-、:::::::::::::::::::::::::::::::::::|/:/`‐'::\;;;;;;;_|
   |    |/::::::::::::::::::::::\:::::::::::::::::::::::::::::|::/::::|::::/:::::::::::/
    |   /:::::::::::::::::::::::::::::::::|:::::::::::::::::::::O::|::|::::::|:::::::::::::::/
72132人目の素数さん:2012/08/19(日) 14:25:16.65
事故解決しますた
APとかPBとかいう書き方には改善の余地がありますね
どうもありがとうございました
73132人目の素数さん:2012/08/19(日) 14:43:24.57
> 事故解決しますた

こういうのしたり顔で書いてるの見ると夏休みだなあと思う
74132人目の素数さん:2012/08/19(日) 16:13:07.88
>>73
俺がしたり顔だとどうして思ったの?
俺が夏休みだとどうして思ったの?
根拠のない邪推はよくないよね?
75132人目の素数さん:2012/08/19(日) 17:09:48.29
しますたとかふざけた言い方で謝意はない
誤字は天然かもしれんが
76132人目の素数さん:2012/08/19(日) 17:23:50.78


次の2次不等式を満たす整数xをすべて求めよ。
(1)2x^2+3x-9≦0

(2)x^2-2x-4<0
77132人目の素数さん:2012/08/19(日) 17:31:44.26
>>76
まず、この不等式を満たすxを求めてみたらどうだろう。
78132人目の素数さん:2012/08/19(日) 17:51:07.59
http://www.math.tohoku.ac.jp/exam/questions/10zenki-com.pdf

これの大門4の模範回答をよろしくお願いします。
79132人目の素数さん:2012/08/19(日) 18:27:06.93
すみません、書き忘れましたが(1)(2)は解けましたので(3)だけ教えていただければ十分です。
80132人目の素数さん:2012/08/19(日) 18:29:45.06
          __ノ)-'´ ̄ ̄`ー- 、_
        , '´  _. -‐'''"二ニニ=-`ヽ、
      /   /:::::; -‐''"        `ーノ
     /   /:::::/           \
     /    /::::::/          | | |  |
     |   |:::::/ /     |  | | | |  |
      |   |::/ / / |  | ||  | | ,ハ .| ,ハ|
      |   |/ / / /| ,ハノ| /|ノレ,ニ|ル' 
     |   |  | / / レ',二、レ′ ,ィイ|゙/   私は只の数ヲタなんかとは付き合わないわ。
.     |   \ ∠イ  ,イイ|    ,`-' |      頭が良くて数学が出来てかっこいい人。それが必要条件よ。
     |     l^,人|  ` `-'     ゝ  |        さらに Ann.of Math に論文書けば十分条件にもなるわよ。
      |      ` -'\       ー'  人          一番嫌いなのは論文数を増やすためにくだらない論文を書いて
    |        /(l     __/  ヽ、           良い論文の出版を遅らせるお馬鹿な人。
     |       (:::::`‐-、__  |::::`、     ヒニニヽ、         あなたの論文が Ann of Math に accept される確率は?
    |      / `‐-、::::::::::`‐-、::::\   /,ニニ、\            それとも最近は Inv. Math. の方が上かしら?
   |      |::::::::::::::::::|` -、:::::::,ヘ ̄|'、  ヒニ二、 \
.   |      /::::::::::::::::::|::::::::\/:::O`、::\   | '、   \
   |      /:::::::::::::::::::/:::::::::::::::::::::::::::::'、::::\ノ  ヽ、  |
  |      |:::::/:::::::::/:::::::::::::::::::::::::::::::::::'、',::::'、  /:\__/‐、
  |      |/:::::::::::/::::::::::::::::::::::::::::::::::O::| '、::| く::::::::::::: ̄|
   |     /_..-'´ ̄`ー-、:::::::::::::::::::::::::::::::::::|/:/`‐'::\;;;;;;;_|
   |    |/::::::::::::::::::::::\:::::::::::::::::::::::::::::|::/::::|::::/:::::::::::/
    |   /:::::::::::::::::::::::::::::::::|:::::::::::::::::::::O::|::|::::::|:::::::::::::::/
81132人目の素数さん:2012/08/19(日) 21:34:34.49
同一直線上にない異なる3点A、B、Cに対し、条件K>−5
途中計算を省く
AP↑=4/K+5・AB↑+1/4/K+5・AC↑ である。

問題
BA=BCであるとき、線分PBが∠ABCを2等分するようなKの値を求めよ。


どのように考えていけば分かりません、よろしくお願いします。
82132人目の素数さん:2012/08/19(日) 21:38:42.51
>>81
> AP↑=4/K+5・AB↑+1/4/K+5・AC↑
なんだかよくわからん。
83132人目の素数さん:2012/08/19(日) 21:47:05.51
>>81
式が意味をなさんから
http://uni.2ch.net/test/read.cgi/math/1344601397/1-3
でも見てからにするんだな
84132人目の素数さん:2012/08/19(日) 21:51:19.70
>>81
> AP↑=4/K+5・AB↑+1/4/K+5・AC↑ である。
L=4/(K+5)とおく
AP↑=L・AB↑+(1/L)・AC↑ 
ということか

> 問題
> BA=BCであるとき、線分PBが∠ABCを2等分するようなKの値を求めよ。
>
>
> どのように考えていけば分かりません、よろしくお願いします。
二等辺三角形BCAの頂角∠Bを2等分する直線BPは底辺ACの中点を通る。
85132人目の素数さん:2012/08/19(日) 22:03:33.53
中点を通るっつうか、底辺と直交する。
86132人目の素数さん:2012/08/19(日) 22:03:44.26
 千石さんはなぜ命を狙われているんですか?!
87132人目の素数さん:2012/08/19(日) 22:05:22.54
>>84

そこまでは理解してるんですけど、どう式をつくっていけばいいのかが、よく分かりません
88132人目の素数さん:2012/08/19(日) 22:08:25.94
>>85
81です

直行することにきずけば、式は立てれますね
わざわざありがとうございます
89132人目の素数さん:2012/08/19(日) 22:11:25.35
20代の、ニートの、女性恐怖症の、頭デッカチの虚弱児・ひ弱の、関西の、ゴミ・クズ・カス・無能・虫けらのクソガキ! 

 お前、高校野球見ているか? 10代が大活躍だ! オマエ、恥ずかしくないか?

90132人目の素数さん:2012/08/19(日) 22:13:20.56
クソ暑い中、球遊び興行に無償で協力、御苦労なことだ
91132人目の素数さん:2012/08/19(日) 22:20:54.31
>>81
途中計算を省いたAP↑の式、正しいのか?
92132人目の素数さん:2012/08/19(日) 22:47:45.09
次の放物線と直線に共有点があれば、その座標を求めよ。
(1)y=x^2、y=-x+6
(2)y=-x^2-3x-7、y=3x+2
(3)y=x^2+2、y=2x-6

教えてください><
93132人目の素数さん:2012/08/19(日) 22:48:59.18
>>77
わかりました。
94132人目の素数さん:2012/08/19(日) 22:56:57.14
>>85
この問題(解けるように作られた問題として)では直交条件は上手く使えないんじゃないかな。
95132人目の素数さん:2012/08/19(日) 23:13:30.04
>>92
グラフの交点が求められない
連立方程式が解けない

数Uか
9664です:2012/08/20(月) 03:18:29.46
以下のように考えてみたのですが。

0<x<y より
1<y/x
そこで
y/x=1+s
s>0
とおく。
y=x(1+s)
x^y=y^xに代入して、
x^(x(1+s))=(x(1+s))^x
(x^x)(x^(xs))=(x^x)(1+s)^x
x^x≠0より
x^(xs)=(1+s)^x
x≠0より
x^s=1+s
よって
x=(1+s)^(1/s)
y=(1+s)^((1+s)/s))
すると、
y=x^(1+s)=x^(x^s)
なので、sをxで表せばよいと考えました。

続きます
9764です:2012/08/20(月) 03:19:34.11
続きです

x=(1+s)^(1/s)
より
log(x)=log(1+s)/s
log(1+s)=tとおくと、
s=-1+e^t
で、
log(x)=t/(-1+e^t)
-log(x)+(e^t)log(x)=t
-(e^(-t))log(x)+log(x)=te^(-t)
log(x)=(t+log(x))e^(-t)
-log(x)/x=(-t-log(x))e^(-t-log(x))
よって、ランベルトのW関数を使って
W(-log(x)/x)=-t-log(x)
ところが、
W(-log(x)/x)=-log(x)なので
t=0
つまり、
s=0
になってしまいます。
どこに間違いがあるか教えてください。
98132人目の素数さん:2012/08/20(月) 03:40:27.70
運営乙か
99132人目の素数さん:2012/08/20(月) 07:57:22.31
Π(1+a[n])は収束、Σa[n]は発散
なる複素数列(a[n])を挙げろ

問題文には書いてないですが、a[n]=-1とかしてΠを0に収束させるのはナシな雰囲気です
100132人目の素数さん:2012/08/20(月) 08:19:01.84
雰囲気とか馬鹿か?
101132人目の素数さん:2012/08/20(月) 10:20:12.12
>>100
a[n]=-1
で済ますよりはましかと
102132人目の素数さん:2012/08/20(月) 11:40:19.38
a[n]=-1 こそが一発めの想定解答やないの?
これがダメってどんな雰囲気やw
103132人目の素数さん:2012/08/20(月) 12:24:48.51
Σ_{j=0}^{l} 1/j! 1/(l-j)! (-1)^j
これを一般の整数lで解く方法を教えてください。
104132人目の素数さん:2012/08/20(月) 12:26:25.35
>>103
書き方を勉強して来い、糞蟲くん! カーッ(゚Д゚≡゚д゚)、ペッ
105132人目の素数さん:2012/08/20(月) 12:50:23.71
>>99
Π(1+a[n])が絶対収束なら b[n]=log(1+a[n]) で logΠ(1+a[n])=Σb[n] も絶対収束
正数列とすると適当に大きい n で b[n]<1 として
Σa[n]=Σ(exp(b[n])−1)=Σ(Σb[n]^m/m!;m≧1)=Σb[n]+Σ(Σb[n]^m/m!;m≧2)
≦Σb[n]+(1/2)Σ(Σb[n]^m;m≧2)=Σb[n]+(1/2)Σb[n]^2/(1−b[n])
も絶対収束。
だから絶対収束でない例を探さねばならない
106132人目の素数さん:2012/08/20(月) 20:01:05.55
1/x+1/y+1/z=1

x≦y≦z

自然数であるxとyとzを求めてください
107132人目の素数さん:2012/08/20(月) 20:05:42.32
2,3,6
108132人目の素数さん:2012/08/20(月) 20:13:08.74
解き方お願いします
109132人目の素数さん:2012/08/20(月) 20:32:44.68
女神が教えてくれるのです。
110132人目の素数さん:2012/08/20(月) 21:26:58.77
x≦y≦z より 1/z≦1/y≦1/x.
よって 1/x+1/y+1/z=1≦3/x.
xは自然数だから1または2または3.

あとはそれぞれの場合について調べる。
111132人目の素数さん:2012/08/20(月) 21:37:56.54
10!を計算していて気づいたのですが

1×2×3×4×5×6=8×9×10
7を除いた前後の積が等しい

このような例はもっと大きな数で存在しないでしょうか?

112132人目の素数さん:2012/08/20(月) 21:39:39.26
コンピュータにやらせてみたら?
113132人目の素数さん:2012/08/20(月) 22:33:15.94
7は素数
114132人目の素数さん:2012/08/20(月) 22:40:31.61
>>111
例が一個で規則を要求する

大胆だな
115132人目の素数さん:2012/08/20(月) 22:54:28.08
とりあえず素因数分解
116132人目の素数さん:2012/08/20(月) 23:04:28.72
無理っぽい気がするな
117132人目の素数さん:2012/08/20(月) 23:23:48.82
>>111
200!までしらみつぶしに調べたけどそれしかないな
118132人目の素数さん:2012/08/20(月) 23:36:01.20
(p-1)! ≡ -1(mod p) , p: prime
119132人目の素数さん:2012/08/21(火) 00:30:10.09
11がでてくると22までは11をどけなきゃいけないが7をどうするんだという話になって
7のために14まで入れると今度は13をどうするんだという話になって

そんなこんなで考えなきゃいけない素因数がどんどん増えていく
120132人目の素数さん:2012/08/21(火) 00:48:44.36
素数生成アルゴリズムですから
121132人目の素数さん:2012/08/21(火) 08:18:52.61
これから〜を示そう。
って数学の本にあるじゃないですか。
これって俺と一緒に〜を示そうってことですか?それとも
〜を示してみせてやろうってことですか?
122132人目の素数さん:2012/08/21(火) 10:49:43.17
数学の論理は誰がやっても同じ。
もちろん、思いつくか否かとか、理解できるか否かは個人差があるが、それはまた別の話。
123132人目の素数さん:2012/08/21(火) 12:16:22.79
>>121
英語だと普通は主語が we だから前者の意味かと
124132人目の素数さん:2012/08/21(火) 12:36:31.67
weっていうのは形式な主語だから意味は無いとおもうんですけど。
125132人目の素数さん:2012/08/21(火) 13:17:38.84
それは日本語にした場合
126132人目の素数さん:2012/08/21(火) 14:10:08.34
ちょっと質問なんですがexp(2iπτ)って(exp(2iπ))^τってなるので
τの値に関わらず解は1になるのでしょうか?
127132人目の素数さん:2012/08/21(火) 14:22:38.90
The Seven is our prime in a whole life.
128132人目の素数さん:2012/08/21(火) 14:38:59.47
>>126
τが整数なら。
129132人目の素数さん:2012/08/21(火) 14:55:40.17
τは少数にもなりますが、少数だったら1以外になるんですか?
exp(2iπ)=1だから何乗しても1と思ってましたが…
130132人目の素数さん:2012/08/21(火) 14:58:53.52
-1 * -1 = ?
131132人目の素数さん:2012/08/21(火) 15:04:08.25
-1*(-1) もしくは (-1)*(-1)と書くところだな
132132人目の素数さん:2012/08/21(火) 15:12:47.32
>>129
(a^b)^c=a^(bc)はa,b,cの値によっては成り立たない
133132人目の素数さん:2012/08/21(火) 15:13:25.27
>>130-131
exp(2iπ)=(-1)*(-1)ってことですか?
ごめんなさい、違いが全くわかりません…
134132人目の素数さん:2012/08/21(火) 15:18:34.31
>>132
そうなんですか?
ちなみにτの値はf/fs(f,fs:整数)となっていて
exp(2iπ・f/fs) なので {exp(2iπ)}^(f/fs) = 1^(f/fs) = 1
と思ってましたが{exp(2iπ)}^(f/fs)が成り立たないんですか?
135132人目の素数さん:2012/08/21(火) 15:22:50.48
ド・モアブル(1667年5月26日 - 1754年11月27日)はフランスの数学者である。
主な業績としてド・モアブルの定理を証明したことが知られている。また負の二項分布、二項分布の極限としての正規分布、今日スターリングの公式として知られる近似式なども彼の研究成果である。

ドモアブルの定理(ドモアブルの公式)とは整数 n に対して、
(cos θ + i sin θ)n = cos nθ + i sin nθ
が成り立つという複素数に関する定理である。証明には三角関数の加法定理が利用される。
非整数次の指数に関しては一般に成り立たない。
それは、複素数の非整数次の指数は複数の異なる値をとる多価関数だからである(failure of power and logarithm identities参照)。そして、複素数のn 次の指数計算を含むことから、n が整数でないときその値はその中の可能な値のひとつを表しているにすぎない。

a が整数ならば 2akπ は常に2πの倍数となり式が成立している。 逆に a が整数でないとき、(e^{i \theta})^aは e^{i 2ak\pi} がとりうる値の数だけ異なる値をもつことになる。
2akπ が2πの倍数になるようなk は必ず存在する(K=0)ので定理はこれに対応する値を導くが、それ以外の値を考慮できていない。
136132人目の素数さん:2012/08/21(火) 15:23:58.52
>>134
exp(iπ)=-1
137132人目の素数さん:2012/08/21(火) 15:41:57.07
>>136
exp(2iπ) = {exp(iπ)}^2 = (-1)^2
もしかしてこれが間違ってますか?
138132人目の素数さん:2012/08/21(火) 15:45:11.19
>>126
ところで、こんなことに疑問をもつなんて、どの教科書で勉強してるんですか?
139132人目の素数さん:2012/08/21(火) 15:47:44.67
>>137
その式は正しい
140132人目の素数さん:2012/08/21(火) 15:54:14.49
韓国+竹島=日本の領土
日本の領土-韓国=竹島
この式の検証をお願いします。
141132人目の素数さん:2012/08/21(火) 16:00:22.75
>>138
教科書から見つけたわけでなく出された課題にあって疑問に思っただけです

>>139
なんだかよくわからなくなってきました…

手元の参考資料を見つつもう少し考えてみます
考えてくれた皆さんありがとうございました
142 【関電 86.0 %】 :2012/08/21(火) 17:14:44.95
虚数軸には意味がある
でも、数学科の人間がそれを見つけることはないだろう
おそらく物理学科の人間がそれを見つけると思われる。

143 【関電 86.0 %】 :2012/08/21(火) 17:17:40.22
>>126
ヒント ド・モアブルの定理の一般化
144132人目の素数さん:2012/08/21(火) 17:18:35.22
数学科の人間は自然数の「意味」すら考えていないからな。
145132人目の素数さん:2012/08/21(火) 17:47:25.02
>>142
>>144
はあ?
オマエラは自分が生きている意味を考えてみたら?
146132人目の素数さん:2012/08/21(火) 18:00:18.66
2iπって書き方が気持ち悪い
147132人目の素数さん:2012/08/21(火) 18:11:46.78
普通、実数・虚数の順で書くよなあ
148132人目の素数さん:2012/08/21(火) 18:17:24.24
aが正の実数の時、
「a^(1/a)」がとりうる値の範囲を教えて下さい
149132人目の素数さん:2012/08/21(火) 18:23:56.75
>>148
0 < a^(1/a) ≦ e^(1/e)
150132人目の素数さん:2012/08/21(火) 18:26:45.59
即答ありがとうございます
151132人目の素数さん:2012/08/21(火) 19:00:15.31
して礼は?
152132人目の素数さん:2012/08/21(火) 19:30:45.11
ねたが同じ問題ばかり
15364,97,98:2012/08/22(水) 02:10:58.46
W関数を最後に使ったところで小さい方と一致してしまうようですね。この方法のままではダメなのはわかりましたが、どうすれば良いかがわかりません。どなたか教えてください。
15464,96,97でした:2012/08/22(水) 02:12:45.28
難しいです
155132人目の素数さん:2012/08/22(水) 02:45:42.36
すみません、質問です。
0≧x≧-1、y≧1のとき、
(1+x)^y ≧ e^(x*y) + 0.5x
の解き方を教えてください。
お願いします。
156132人目の素数さん:2012/08/22(水) 03:02:34.08
>>155
「解く」とは?
157132人目の素数さん:2012/08/22(水) 03:12:32.48
>>156
不等式がなぜ成り立つのかを証明することです。
誤った言葉の使い方をしてしまい、申し訳ないです。
158132人目の素数さん:2012/08/22(水) 12:46:10.85
球面上に直線はひけるか
159132人目の素数さん:2012/08/22(水) 12:59:35.94
(f[a+h] - f[a])/h
160132人目の素数さん:2012/08/22(水) 13:12:01.14
A D - B C
161132人目の素数さん:2012/08/22(水) 13:17:20.07
a x + b y
162132人目の素数さん:2012/08/22(水) 13:25:12.39
AC / DC
163132人目の素数さん:2012/08/22(水) 14:45:52.82
質問お願いします。二重積分の問題です
∬exp(-x^2-xy-y^2)dxdy
積分の領域はx≧0,y≧0です。

領域が曲者で全空間であれば次の変数変換で解けています
u=x+(1/2)y
v=(√3/2)y
ヤコビアンは2/√3となります
しかし、x≧0,y≧0の範囲ではexp(-x^2)の不定積分が存在しないため解けなくなってしまいました

お願いします
164132人目の素数さん:2012/08/22(水) 16:42:49.79
u=rcosθ、v=vsinθって変数変換すればいい。定番問題だよ。
165132人目の素数さん:2012/08/22(水) 16:47:54.78
vは、v=rsinθのミス
dudv=rsinθdrdθと、rが出てくるのがミソ。
re^(-r^2)なら積分できるからね。
166132人目の素数さん:2012/08/22(水) 16:56:14.63
なるほどそこからさらに極座標に変換するのですね
答えはπ/(3√3)となりました。
本当にありがとうございました
167132人目の素数さん:2012/08/22(水) 17:03:42.66
質問ですが、
√z は、z=0 が「特異点」だという理解でよろしいですか?
また、特異点の種類としては、そもそも、「定義自体が出来ない」と
いう種類の物という理解であってるでしょうか?

仮にz=0で無理矢理ローラン展開するために、z=0の周りの周回積分を
考えてみると、ローラン展開のzの負の次数の係数は全て0になりそう
です。

なのに、zが実数の場合のグラフを想像すると、微分が無限大になって
いる必要があり、矛盾します。

この点から考えても、
「z=0では、複素関数としての√zは、定義自体が出来ない」
という事のようですが。
168132人目の素数さん:2012/08/22(水) 17:09:17.32
>>167
分岐点。値は0と定義できる。
169132人目の素数さん:2012/08/22(水) 17:09:55.24
王者乙
170132人目の素数さん:2012/08/22(水) 17:35:17.08
>>163
くせーな
回転するのがjkだろう
171132人目の素数さん:2012/08/22(水) 17:36:12.68
>>167
どんな本読んだの?
172167:2012/08/22(水) 17:37:35.08
>>168
あー
173132人目の素数さん:2012/08/22(水) 17:39:50.90
>>171
実はろくに読んでいません
174167:2012/08/22(水) 17:42:06.59
z=0では定義出来るが、z=0を通るとある一本の直線は必ず定義出来ない、
というように理解出来るのでしょうか?

あるいは、その直線上では正則ではない、と言う事ですか?


>>171
つまみ読みです。
175132人目の素数さん:2012/08/22(水) 17:59:40.66
>>174
疑問は不要、留数計算できればいいのだろう
176132人目の素数さん:2012/08/22(水) 18:47:55.66
100円賭けて1/2で勝負して勝てば相手から400円で払い戻されるとき、
賭けた側の期待値は+いくらになりますか?
177132人目の素数さん:2012/08/22(水) 19:21:22.42
>>174
意味不明な事言ってないでちゃんと読め
178132人目の素数さん:2012/08/22(水) 20:59:56.09
>>174
> つまみ読みです。

馬鹿が!
179 【関電 81.3 %】 :2012/08/23(木) 21:25:57.06
>>145
それも物理学が与えるよ。残念だったな。
180132人目の素数さん:2012/08/24(金) 00:24:17.59
数学板で物理学にデタラメ言うな
181132人目の素数さん:2012/08/24(金) 12:20:54.80
a,b∈Z
a^2-5b^2=2
の解の求め方を教えてください。
182132人目の素敵さん:2012/08/24(金) 13:22:48.78
オレンジジュースが3.5L りんごジュースが2.1L。リンゴジュースの量はオレンジジュースの量の何倍でしょう?って割り算になりますか?かけ算になりますか?教えてください!!
183132人目の素数さん:2012/08/24(金) 13:28:49.83
>>181
変形して、
(a^2-2)/5=b^2
平方数の1の位は0,1,4,5,6,9だけなので、a^2-2が5の倍数になる事はない
解なし
184132人目の素数さん:2012/08/24(金) 13:31:25.86
>>182
「リンゴ÷オレンジ」の割り算になる
185132人目の素数さん:2012/08/24(金) 13:39:18.39
>>183
ありがとうございます。ちなみに
平方数の一の位がそれだけってどうやって分かるんですか?
逆に2を他の数Xにおきかえてa^2-Xが5の倍数になったときは
解はあるのですか?
186132人目の素敵さん:2012/08/24(金) 14:37:48.87
>>185
> 平方数の一の位がそれだけってどうやって分かるんですか?
0から9までそれぞれ2乗してみれば分かる。
187132人目の素敵さん:2012/08/24(金) 14:41:18.05
>>185
a^2-5b^2=X を満たす整数 a, b, X ならいくらでも見つかる
188132人目の素敵さん:2012/08/24(金) 15:15:18.63
>>184

助かりました!!本当に有り難うございます!
頑張って勉強します!
189132人目の素数さん:2012/08/24(金) 15:17:08.67
>>186-187
コマプスムニダ
190132人目の素数さん:2012/08/24(金) 15:49:08.30
リンゴをオレンジで割ったら酸っぱい
191132人目の素数さん:2012/08/24(金) 17:24:04.05
∇の公式を証明せよという問題がよくわかりません
例えば∇(φψ)=φ∇ψ+ψ∇φとか∇・(a↑+b↑)=∇・a↑+∇・b↑といったものです
調べてはみたものの、公式だからといってちゃんと説明してくれている所が無くて困ってます
192132人目の素数さん:2012/08/24(金) 17:29:45.81
いくら何でも、成分で書いて比較すれば何とかなるだろ
193132人目の素数さん:2012/08/24(金) 17:34:52.85
成分で書くというのがよくわかりません
例えば∇(φψ)はどのようにすればいいんですか?
194132人目の素数さん:2012/08/24(金) 17:40:21.05
諦めた方がいいよ、マジで
195132人目の素数さん:2012/08/24(金) 17:41:31.96
>>193
そもそも∇はどう定義されているのだ?
196132人目の素数さん:2012/08/24(金) 17:48:49.64
大学で勉強するための基礎がないんだから質問に答えたところで意味がない
197132人目の素数さん:2012/08/24(金) 17:50:59.11
文字での書き方がわからなかったので画像ですみません
http://i.imgur.com/oh28q.jpg
こういうことですか?でもこの後どうすればいいのかわからないです
198132人目の素数さん:2012/08/24(金) 17:54:27.75
だっふんだあー
199132人目の素数さん:2012/08/24(金) 18:36:13.88
>>197
偏微分を勉強する
200132人目の素数さん:2012/08/24(金) 18:38:31.70
>>197
高校で関数の積の微分は習っただろ。
偏微分でも同じだ。
201132人目の素数さん:2012/08/24(金) 18:38:56.11
そもそも常微分からアヤシいな
202132人目の素数さん:2012/08/24(金) 18:40:49.51
>>199-200
これは偏微分というものを使わないとできないものなんですか?
すみません、勉強してきます
203132人目の素数さん:2012/08/24(金) 20:24:06.99
http://i.imgur.com/fsSKH.jpg
http://i.imgur.com/hQoJ7.jpg

この問題の解説をよろしくお願いします。
204132人目の素数さん:2012/08/24(金) 20:32:33.91
>>203
その図を切り抜いて棒につけて回してみる
205132人目の素数さん:2012/08/24(金) 20:33:19.80
マルチ
206132人目の素数さん:2012/08/24(金) 21:08:26.95
t=x^2+4xの値域は(ア)であるから、y=(x^2+4x+3)(x^2+4x+5)+2x^2+8x+3をtの関数として表すことによって、
yの最小値が(イ)であることが分かる

(ア)(イ)を求めよ


どうやればいいんでしょうか><
207132人目の素数さん:2012/08/24(金) 21:21:51.64
>>206
問題文の通りにやればいい、としか言いようがないんだけど
208132人目の素数さん:2012/08/24(金) 21:28:42.52
>>204
もう少しくわしくおしえてください。お願いします
209132人目の素数さん:2012/08/24(金) 21:44:52.34
>>208
相似
210132人目の素数さん:2012/08/24(金) 22:09:46.60
>>207
わかりました><
がんばります
211132人目の素数さん:2012/08/24(金) 23:27:56.09
│x^2-4│>-3x 
をグラフを利用して次の不等式を解け


グラフゎどのような形になりますか?
212132人目の素数さん:2012/08/24(金) 23:28:42.63
なかなかのルアーだな
213132人目の素数さん:2012/08/24(金) 23:29:47.55
>>211
y=|x^2-4|
y=-3x

これは描けるだろう 
214132人目の素数さん:2012/08/24(金) 23:37:04.43
>>213
xの範囲は
x≦-2、2≦xのとき
-2<x<2のとき

でいいんでしょうか?
215132人目の素数さん:2012/08/24(金) 23:40:42.56
68種類の食べ物があって、その中で自分が好きな食べ物は2つだけ。あとは全部嫌いで食べられない。
自分の好みなど知らない第3者が68種の中からランダムに5種の食べ物を選ぶ。
その5種の中に自分が食べられるものが2つとも入る確率を知りたいです。
できれば計算式もお願いします
216132人目の素数さん:2012/08/24(金) 23:46:27.89
食わず嫌いも考えてください。
217132人目の素数さん:2012/08/24(金) 23:52:30.22
嫌いでも68種類すべて食べ物なので食べられます
218132人目の素数さん:2012/08/25(土) 00:29:54.03
>>214
ま、それでもいいけど。

y=x^2-4のグラフを描いて、それをじっと眺めて y=|x^2-4|のグラフが思い描けないか?
219132人目の素数さん:2012/08/25(土) 02:56:15.39
>>215
(66C3)/(68C5)
220132人目の素数さん:2012/08/25(土) 06:00:26.39
>>218
あっ!!わかりました!


ありがとうございます><
221132人目の素数さん:2012/08/25(土) 06:06:25.36
t=x^2+4xの値域は(ア)であるから、y=(x^2+4x+3)(x^2+4x+5)+2x^2+8x+3
をtの関数として表すことによって、
yの最小値が(イ)であることが分かる


tの値域の求め方がわかりません
222132人目の素数さん:2012/08/25(土) 08:30:05.24
平方完成
223132人目の素数さん:2012/08/25(土) 12:07:17.86
マルチ
224132人目の素数さん:2012/08/25(土) 15:53:48.83
「(2+√3)^2009を小数で表した時、1の位の数を求めよ」という問題なんですが、
何から手をつけたら良いでしょうか?
225132人目の素数さん:2012/08/25(土) 16:10:55.43
a[k]=(2+√3)^k+(2-√3)^kとすると
a[k+2]=4a[k+1]-a[k]で、一の位は周期を持つ。
|2-√3|<1だから0<(2-√3)^2009<1
(2+√3)^2009の一の位はa[2009]の1の位から1引けばいい
226132人目の素数さん:2012/08/25(土) 16:13:21.10
2^n * (1+sqrt(3)/2)^n
227132人目の素数さん:2012/08/25(土) 16:43:10.11
k=3の倍数 ⇔ a[k] の一の位は2
k≠3の倍数 ⇔ a[k] の一の位は4
228132人目の素数さん:2012/08/25(土) 17:11:48.17
なぞなぞ 

車の運転が遅いとき→131
できたてのわたあめ→5
笑顔のとき→41
嬉しい期待→80

では、「調べれば答えが分かるくいず」は?
229224:2012/08/25(土) 19:21:25.59
>>225>>227
ありがとうございます。理解できました。

>>226
これは別解となる考え方でしょうか?
230132人目の素数さん:2012/08/25(土) 20:53:31.11
>>228
マルチ
231132人目の素数さん:2012/08/26(日) 20:43:08.75
0でないベクトルu↑,v↑について2|u↑|=3|v↑|で,u↑とv↑のなす角は60度とする.
u↑+v↑と7u↑+tv↑が垂直である時,tの値を求めよ

これお願いしますorz
232132人目の素数さん:2012/08/26(日) 22:20:18.77
2つのベクトルが垂直の時それらのベクトルの内積は0になる事を使う
233132人目の素数さん:2012/08/27(月) 05:26:22.83
4(x^2-y^2-17/4x-51/4y+18/4)=0
これ因数分解できるんでしょうか?
ちなみにx=9 y=3なのですが(x≧0, y≧0)
234132人目の素数さん:2012/08/27(月) 05:33:55.28
すみません>>233ですが質問を変えます
x>0, y>0 で

26/(x+y)+26/(x-y)=6.5
26/(x+y+1)+26/(x-y-2)=8.5
この解がx=9 y=3なのですが、どのような計算で解を出せばよいのでしょう?
この2式を色々動かした結果が>>233の式なのですが、ここで行き詰まり解が導き出せません
上手な計算方法を教えて下さい
235132人目の素数さん:2012/08/27(月) 06:47:46.47
地道に計算すれば解ける

26/(x+y)+26/(x-y) = 6.5
をyについて整理すると
y^2 = x^2-8x
y = √(x^2-8x)  ・・・・@

これを
26/(x+y+1)+26/(x-y-2) = 8.5
に代入して整理すると
5x+6 = 17√(x^2-8x)

両辺を二乗して整理すると
66x^2-593x-9 = 0
(66x+1)(x-9) = 0
x = -1/66,9

x>0なので、x = 9
@に代入して、
(x,y) = (9,3)
236132人目の素数さん:2012/08/27(月) 12:00:11.42
記憶があいまいなんですけど。
Z[√-5]の2を含む極大イデアルが(2,1-√-5)を証明するところで
A=Z[√-5], k=Z/2Zとおくと
A/2A=k[X]/(X-1)^2
だからわかると著者はいうんですけど。
A/2A=k[X]/(X-1)^2がどう関係して上のが極大イデアルなのかさっぱりわからにんですけど
詳しい人詳しく手取り足取り教えてくれると嬉しいです。
ちなみに=記号と=の上の線が〜担っている記号が間違って使われているのは
もともとです。
237132人目の素数さん:2012/08/27(月) 13:56:45.49
(X+1)^2に(X-1)^2を訂正します。
すみません、記憶がほとんどないので間違えました。
238132人目の素数さん:2012/08/27(月) 14:03:36.31
A=Z[√-5]=Z[X]/(X^2+5)
Z→k=Z/2Z で Z[X]→k[X], X^2+5→X^2-1=(X+1)(X-1)=(X-1)^2=(X+1)^2
k[X] で (X-1)^2 を含む極大イデアルは (X-1)
X^2+5=0 は X=√-5, X-1≡0 は 1-√-5≡0
239132人目の素数さん:2012/08/27(月) 14:07:39.50
>>238
ありがとうございます。
すみません、さっぱりわかりません。
もうちょっと初心者でも分かるように書いてくれませんか?
240132人目の素数さん:2012/08/27(月) 14:27:15.39
http://www.amazon.co.jp/Commutative-Cambridge-Studies-Advanced-Mathematics/dp/0521367646
の中身検索の6ページ目に書いてあります。
よろしくお願いします。
ちなみに5ページ目も分かりません。
241132人目の素数さん:2012/08/27(月) 15:11:55.63
A=Z[√-5]=Z[X]/(X^2+5)と書いたとき、k=Z/2Zとおくと、
A/2A=Z[X](2,X^2+5)=k[X]/(X^2-1)=k[X]/(X-1)^2を
得るからP=(2,1-√-5)は2を含む極大イデアルです。

これを詳しく証明してください。
飛びすぎていてわかりません。
242132人目の素数さん:2012/08/27(月) 15:18:51.11
書き忘れました。
PはZ[√-5]の極大イデアルです。
243132人目の素数さん:2012/08/27(月) 16:26:42.96
>>238が詳しく書いてくれているじゃん
これ以上何を?
244132人目の素数さん:2012/08/27(月) 16:30:39.10
>>243
ありがとうございます。
式だけだとさっぱり理解できないので式が何を言いたいのか
>>238の間に日本語で式の説明をいれてくれませんか?
先生お願いします。
245132人目の素数さん:2012/08/27(月) 16:36:05.15
あと≡とか→もわかりません。
コンマが続いているところもどこが区切りなのかとかもわかりません。
それ以前に何をやっているのかも分かりません。
日本語で解説よろしくお願いします。
246132人目の素数さん:2012/08/27(月) 16:38:19.17
なるほど、そのレベルなら無理
247132人目の素数さん:2012/08/27(月) 16:58:43.55
(X-1)←(X-1)
Z[X]←k[X]
Z←k
A/(2,1-√-5)←Z[X]/(2,X-1)
もしかして、こういうことですか?
もし当たってたら、これを詳しく説明してくれませんか?
248132人目の素数さん:2012/08/27(月) 17:00:07.02
埋め込みかつ微分同相写像でないものって何がありますか?
249132人目の素数さん:2012/08/27(月) 17:15:05.53
>>245
勉強しなきゃ説明も無意味
250132人目の素数さん:2012/08/27(月) 17:34:21.99
じゃあ、もういいです。
さようなら。
251132人目の素数さん:2012/08/27(月) 23:08:35.67
>>235
26/(x+y)+26/(x-y) = 6.5をyについて整理するのかあ
なんか無理矢理色々してたな…ありがとうございます!
252132人目の素数さん:2012/08/28(火) 00:51:32.35
X,Yが独立な標準正規分布においてU=2X-Y, V=2Y-XとしたときのUとVの相関係数を求めろ。という問題です

UとVの分散はすぐに出そうなので共分散を求めればいけると思うのですがE[UV]の計算方法がわかりません。
お願いします。
253132人目の素数さん:2012/08/28(火) 09:22:25.95
>>252
とりあえず U,V の分散を計算してみ。
E[U^2]が計算できるなら、E[UV]だって計算できるだろ。
254132人目の素数さん:2012/08/28(火) 11:08:12.01
>>253
やっぱりできませんでした。
E[U^2]=∫u^2 f_U(u)du
とやって、f_U(u)ってなんだ?状態に。
f_U(u)=2f_X(x)-f_Y(y)ってやるのは違いますよね?
255132人目の素数さん:2012/08/28(火) 11:10:37.03
>>252
UV=(2X-Y)(2Y-X) の展開が出来ない?
256253:2012/08/28(火) 11:31:41.80
>>254
f_U(u) ってなんなの?確率密度関数?
いま与えられている確率密度関数は f_X(x) と f_Y(y) だから、これを使って書かないと混乱するよ。

XとYの同時試行における確率密度関数は f_X(x)・f_Y(y) (独立だから)
よって E[U^2]=∬u^2 f_X(x)・f_Y(y) dxdy
257132人目の素数さん:2012/08/28(火) 12:13:31.10
>>254
>f_U(u)=2f_X(x)-f_Y(y)ってやるのは違いますよね?
いや、正にそれでいいんだよ。
展開してE[X^2]、E[Y^2]、E[XY]で表すんだ。
258132人目の素数さん:2012/08/28(火) 13:55:17.38
>>257
σ_U^2=E[U^2]-E[U]^2
=E[(2X-Y)^2]-E[2X-Y]^2
E[X]=E[Y]=0であるから、
=4E[X^2]-E[Y^2]
同様に、
σ_V^2=4E[Y^2]-E[X^2]
σ_UV^2=-2[X^2]-2[Y^2]

こうですか?これだとE[X^2]とE[Y^2]を求めればいけそうですが
E[X^2]=∫x^2 f(x)dx=x E[X]-∫E[X]dx=0
となってしまいました。どこがおかしいんでしょう?
259132人目の素数さん:2012/08/28(火) 13:58:39.90
なんか盛り上がってるな。
260132人目の素数さん:2012/08/28(火) 14:01:54.38
部分積分の計算
261253:2012/08/28(火) 14:10:24.15
>>258
> こうですか?これだとE[X^2]とE[Y^2]を求めればいけそうですが
> E[X^2]=∫x^2 f(x)dx=x E[X]-∫E[X]dx=0
> となってしまいました。どこがおかしいんでしょう?
わるいけど、どんな変形したのかよくわからない。
というか、定積分なのになんで変数 x が飛び出すの?

E[X^2]の値は、Xが標準正規分布であることから察してくれ。

あと前半にも微妙に typo あるね。
262132人目の素数さん:2012/08/28(火) 14:18:15.23
E[XY]=0になるのなんで?
263132人目の素数さん:2012/08/28(火) 14:47:04.27
>>260
そうです。部分積分を使いました。

モーメント母関数M(θ)=e^(θ^2/2)をつかって
M''(θ)=e^(θ^2/2)+θ^2・e^(θ^2/2)
E[X^2]=M''(0)=1
であってますか?

>>262
E[XY]=E[X]・E[Y]=0
とやったんですが
264132人目の素数さん:2012/08/28(火) 15:48:02.42
>>263
そこでモーメント母関数が出てくるあたり、エライなーと思うけど、
標準正規分布ならそもそも分散1じゃろ。
265132人目の素数さん:2012/08/28(火) 15:58:48.13
>>258
σ_X^2=E[X^2]-E[X]^2 を変形すると E[X^2]=E[X]^2+σ_X^2 になることは分かる?
266132人目の素数さん:2012/08/28(火) 16:33:21.46
なぜいきなり中学生扱いwww
267132人目の素数さん:2012/08/28(火) 17:04:07.64
質問したいのですが、例えば、サイコロをふって、1が出たときだけ1万円もらえて、一回の参加費が
千円かかるギャンブルがあった場合、やればやるほど得しますが、3回未満ぐらいだと損する確率の方が
高いです。

僕は組み合わせなどの式を使って計算しているのですが、例えばこのギャンブルを6万回などの多数の試行を
した場合に、1は、大体1万回前後出ていると予想されますが、
これが、
@9千回程度出ている確率
A9千五百回程度出ている確率
B丁度1万回程度出ている確率
C1万5百回程度出ている確率
D1万1千回程度出ている確率

など、細かく場合わけしたりして、このギャンブルの損得具合を考えたいと思います。あと、どれぐらいの回数を試行すれば、儲け具合が安定してくるかなどがわかるようにしたいのですが、組み合わせなどを使って確率を考えた場合、計算が大量になりすぎます。

正確な数値でなくとも、大体これぐらいの数値など、おおよその出し方でもいいので、簡単にこういう場合分けのときの
おのおのの確率を出したりできないでしょうか?また、このようなギャンブルの問題があったときに、大体どれぐらいの回数で損得が安定してくるかなどの計算ができるようになりたいのですが、どのように考えれば、あるいはどういう数学の分野を勉強すればいいでしょうか?
268132人目の素数さん:2012/08/28(火) 17:12:39.99
>>267
やるほど大儲けじゃないかそれwww
とりあえず二項分布・大数の法則あたり調べてみてよ。
269132人目の素数さん:2012/08/28(火) 17:16:59.09
統計でそんなのがあった
270132人目の素数さん:2012/08/28(火) 17:22:19.85
>>264
そうでした。無駄に計算してしまいました。

計算しなおしてみましたが>>258の前半の計算のどこが違うのかわかりません
E[XY]=0ではない?
271132人目の素数さん:2012/08/28(火) 17:40:14.91
>>270
>>260は「>>258の部分積分の計算が間違ってる」という意味では
272132人目の素数さん:2012/08/28(火) 17:45:31.08
x^23-1=0
の根をべき乗根だけで表現したいのですが、やり方をご存知でしょうか?
オネガイします。

なお アラシ 無能者のふざけた答えはお断りします。
273132人目の素数さん:2012/08/28(火) 18:05:05.59
知ってるよ
274132人目の素数さん:2012/08/28(火) 18:51:14.96
やっとわかった。
>>258は展開を間違ってました
σ_U^2=4E[X^2]+E[Y^2]=5
σ_V^2=4E[Y^2]+E[X^2]=5
σ_UV=-2E[X^2]-2E[Y^2]=-4
相関係数は-4/(√5√5)=-4/5
でおわり。あってますよね?
275132人目の素数さん:2012/08/28(火) 19:16:08.81
>>254
じゃ、ついでに x = (1,0), y = (0,1) という 2つのベクトルとして、新しいベクトル
u = 2x-y と v = 2y - x のなす角 θの cosθを、内積か余弦定理で求めてごらん。
276132人目の素数さん:2012/08/28(火) 19:17:09.97
アンカーミス
>>254 じゃなくて、 >>274 ね。
277132人目の素数さん:2012/08/28(火) 19:55:51.19
>>275
さすがにそれぐらいは馬鹿にしないでくださいよって
cosθ=-4/5…!?
どういうことです?相関係数が内積?
278132人目の素数さん:2012/08/28(火) 19:58:59.65
因数分解の問題で、

x2+5/6x+1/6

xのあとの2は2乗と、とって下さい。
279275:2012/08/28(火) 19:59:44.51
実はそうなんだよね。内積をメンドーくさく計算してるだけ。
280132人目の素数さん:2012/08/28(火) 20:04:03.67
281280:2012/08/28(火) 20:05:59.11
282132人目の素数さん:2012/08/28(火) 20:12:02.26
>>277
統計がベクトルで、共分散が内積。相関係数は角度の余弦で、独立は直交。
283275:2012/08/28(火) 20:14:37.74
ついでに言うと、標準偏差はベクトルの長さね。
284267:2012/08/28(火) 20:35:27.06
>>268>>269
二項分布・大数の法則と統計あたりを勉強していけばいいのですね。
やってみます。

ありがとうございました。
285132人目の素数さん:2012/08/28(火) 22:56:08.68
>>282
なるほどー。勉強になりました。
286132人目の素数さん:2012/08/28(火) 23:31:47.39
解き方を教えてください。

問題)√がつかない形にせよ。
  
   √a^2b^2   (a>0,b<0)


287132人目の素数さん:2012/08/28(火) 23:33:55.82
>>286
ルートの定義を思い出す
288132人目の素数さん:2012/08/28(火) 23:34:01.23
2乗するとa^2b^2になるような正の数を探す
289132人目の素数さん:2012/08/28(火) 23:34:13.49
√a^2b^2=|ab|=|a||b|=a*(-b)=-ab
290132人目の素数さん:2012/08/28(火) 23:35:14.13
この手の質問するやつはルートの定義なんて初めから知らないだろどうせ
291132人目の素数さん:2012/08/28(火) 23:38:05.96
>>287
解けました。ありがとうございます。
書き込む前に根号の性質を確認するべきでした。
292132人目の素数さん:2012/08/29(水) 06:00:39.37
-3x2+6x=0
これのxの値を教えてください。おねがいします。
293132人目の素数さん:2012/08/29(水) 06:20:20.69
>>292
-3x(x-6)=0

-3x=0
x=0

x-6=0
x=6

x=0,6
294132人目の素数さん:2012/08/29(水) 06:27:58.51
>>293
助かりました。ありがとうございます!
295132人目の素数さん:2012/08/29(水) 07:22:58.93
お願いします。
仕入れた商品にたいして五割を利益として定価販売したが売れなかった。
そのため定価の3割引きで販売したとき、販売額は42000だった。
仕入れ値はいくらか
296132人目の素数さん:2012/08/29(水) 08:23:01.82
>>295
仕入れ寝をxとおくと
x * 1.5 * 0.7 = 42000
x * 1.5 = 42000 / 0.7
x = 42000 / 0.7 / 1.5 quite easily done
297132人目の素数さん:2012/08/29(水) 08:41:02.44
Q.E.D.はラテン語だぞ
298268:2012/08/29(水) 11:48:49.22
>>284
確率統計の基礎をある程度勉強しないといけないから、あまり簡単ではないかも知らんが
ガンバレ!(・∀・)
299132人目の素数さん:2012/08/29(水) 13:04:36.84
仕入れ寝て・・・
300132人目の素数さん:2012/08/29(水) 17:36:54.75
1/(1-x)^2のマクローリン展開を求めるという問題で、剰余項が0に収束しなくて困ってるんですけど
剰余項は(n+2)*x^(n+1)で合ってますか?
301132人目の素数さん:2012/08/29(水) 18:18:17.04
>剰余項が0に収束しなくて
してるだろ
収束半径がいくらだと思っているのさ
302132人目の素数さん:2012/08/29(水) 18:21:20.65
>剰余項は(n+2)*x^(n+1)で合ってますか?
(n+2)*x^(n+1)は第n+1項であって、そこから先の項を全部足したものを「剰余」と呼ぶのではないのか?フツー
303132人目の素数さん:2012/08/29(水) 18:22:13.25
収束半径r=1って出ました
計算の仕方を間違えてるのかな、よくわからないです
304132人目の素数さん:2012/08/29(水) 18:23:45.77
ある仕事を1人で完成させるのに、Aは5時間、Bは4時間、Cは3時間かかる。
この仕事をA、Bの2人で午前10時から始め、途中からCが1人で行ったところ
午後0時46分に終了した。Cがこの仕事を行った時間は何分か?


という問題で自分は
9/20X+1/3(166−X)=1

という式を立てたのですが解答には結びつきませんでした
解答を見てみると正しい式は
9/20×X/60+(166−X)/180=1

というものでした
馬鹿な自分は何故上の式じゃ駄目なのかが分かりません
どなたか解説お願いします
305132人目の素数さん:2012/08/29(水) 18:36:11.65
>>303
収束半径 |x|<1 で収束する。|x|=1 つまり x = ±1 で収束するかどうかは、別途確かめてみないといけない
が、この場合はだめだね。
306132人目の素数さん:2012/08/29(水) 18:47:20.61
>>304
3600個の商品の箱詰めを、Aは毎分12個の割合でやるので 300分、Bは 15個の割合で
240分、Cは 20個の割合で 180分で仕上げる。今回 3600個の箱詰めを Aと Bでx分やり、
その後 C単独で166-x分、やったところ、仕上がった。xとは何分か。
(12+15)x + 20(166-x) = 3600。これを解いて x = 40.
307305補足:2012/08/29(水) 19:04:19.64
>>303
1/(1-x)^2 = 1 + 2x + 3x^2 + …において、x = -1 とした場合、左辺は 1/4。よって
その右辺 1 - 2 + 3 - 4 + … も収束しているんじゃないかという説もあり、
くわしくは
http://ja.wikipedia.org/wiki/1-2%2B3-4%2B%E2%80%A6
参照。
308132人目の素数さん:2012/08/29(水) 22:09:06.23
>>307
いいかげんな
309132人目の素数さん:2012/08/29(水) 22:11:06.83
>>307
それは収束じゃない
310132人目の素数さん:2012/08/29(水) 22:33:16.68
>>309
「『…』という立場からは、『…』という理由で、これは収束でない」、というくらいのこと
でしょ。推論の途中で 1-2+3-…という形の出てきた場合、収束しないからと思考を打ち切るより、
これを 1/4とおいてもう少し進めてみるほうが、普通は有益と思うが。
311132人目の素数さん:2012/08/29(水) 22:37:36.55
>>310
>収束しているんじゃないかという説もあり
数学者じゃねーだろ
312132人目の素数さん:2012/08/29(水) 22:39:31.64
>その右辺 1 - 2 + 3 - 4 + … も収束しているんじゃないかという説もあり
この「説もある」という言い回しが気色悪いんじゃないかな
「こういう考え方もある」と書けば反発はなかったと思う
313132人目の素数さん:2012/08/29(水) 22:41:44.42
数学者でなくても説くらい主張してかまわんではないか?数学者の反論するのは勝手だが。
314132人目の素数さん:2012/08/29(水) 22:44:12.96
うん、どちらかというと、筋の通った反論を聞きたい。
315132人目の素数さん:2012/08/29(水) 22:44:47.60
ド直球の数学的プラトニストなら、「説」と表現することもあるんじゃない?知らんけど
316132人目の素数さん:2012/08/29(水) 22:50:59.91
>>313

>>311は素人が数学に関する主張をすることを責めているのではなく、
数学では「説もある」というような言い回しはしない、という意味かと。
それは現実の現象を相手にする科学で使われる言葉だから。
317132人目の素数さん:2012/08/29(水) 22:57:10.42
ほう、では数学は「現実の現象を相手にする科学」ではないのか。それは面白い。
318132人目の素数さん:2012/08/29(水) 22:58:37.61
>>317

>>315を読んでね
ついでに>>312
319132人目の素数さん:2012/08/29(水) 23:13:31.70
「収束」って括弧をつければ良かったんだよ。
数学者は厳密な定義から絶対に外れることはできないんだから。
320132人目の素数さん:2012/08/29(水) 23:14:59.23
>>318
やれやれ。
http://en.wikipedia.org/wiki/Theorem
の "Relation with Scientific Theories" の節でも読んでおいてね。まあ、こういう「説」
もあるということで。
321132人目の素数さん:2012/08/29(水) 23:16:00.96
そういう問題ではないでしょ
大体、収束の種類なんていくつもあるんだから、厳密でないわけでもないし
322132人目の素数さん:2012/08/29(水) 23:17:07.46
その「厳密な定義」が、時代によって、立場によって、いろいろ変わるんよ。
323132人目の素数さん:2012/08/29(水) 23:23:07.10
>>320
結局、立場によるってだけの話だ
どの立場からも反論されにくい言い回しとしては、例えば>>312がある
324132人目の素数さん:2012/08/29(水) 23:26:35.13
学術的議論の場では、多くの場合、反論の出るほうが有益だけどね。今回もそういうこと
かな?
325132人目の素数さん:2012/08/29(水) 23:27:44.28
数学の議論で、ただの言い回しに反論が出ることが有益…?
326132人目の素数さん:2012/08/29(水) 23:36:42.20
数学における「説」とは何か、について、考察を深めて、少しは他に対して
寛容になり、真人間に近づく。
327132人目の素数さん:2012/08/29(水) 23:37:35.86
かなり苦しいコジツケですな
328132人目の素数さん:2012/08/30(木) 01:33:07.10
>>307
今は実数位相での収束定義は決まってるんだから
別の位相を定義するか、収束じゃなく別の「級数和」の定義にしなきゃダメだろ
329132人目の素数さん:2012/08/30(木) 01:37:36.08
例えば?
330132人目の素数さん:2012/08/30(木) 02:36:46.84
具体的な定義は307説の側が構成すべきだろ。
収束しない側は普通の収束の定義で済むんだから。
331132人目の素数さん:2012/08/30(木) 03:18:29.55
>>307自体が別の「級数和」の定義の1つになってる
「冪級数化解析接続」とでも言うか
332132人目の素数さん:2012/08/30(木) 03:21:09.45
素数pを比とする等比級数ならp-進位相で収束するな
333132人目の素数さん:2012/08/30(木) 07:31:47.80
>>331
それは定義になっているのか?
パラメータ付きの級数に置き換える方法は無数に有りそうだが、well definedに定まるのか?
334132人目の素数さん:2012/08/30(木) 10:28:49.88
だから1つなのさ
335132人目の素数さん:2012/08/30(木) 16:18:40.07
胎児にダウン症などの染色体異常がないかどうか、妊婦の血液から99%の確率でわかるという新たな出生前診断が、 国内の2つの病院で導入されることがわかった。日本ダウン症協会は、出生前診断が安易に行われることに反対している。
http://headlines.yahoo.co.jp/videonews/nnn?a=20120829-00000050-nnn-soci
(日本テレビ系(NNN) 8月29日(水)17時10分配信 )速報なんで、見る時間によってはURLの変更有り

ここで質問なんですが、その出産前診断の反対理由が↓なんですが・・・正直、しっくりきません
この論法、正しいですか?

◆確率99%は意外に低い精度なんです。

「99%の確率でダウン症の子が生まれる」と診断された場合に、
生まれてくる子供がダウン症である確率は何パーセントかというと…

まずダウン症の子が生まれる確率は1/1000程度と推定して、
1000人の胎児を検査したと仮定しよう。

平均的には1000人に対して、1人がダウン症で、999人がダウン症ではない。
検査の精度が99%ならば、ダウン症ではない999人を検査すると、
ほぼ10人(9.99人)の誤診(1%)が見込まれる。

1000人の胎児を検査して11人が99%の確率でダウン症と診断されて、
実際にはダウン症の胎児は1人だ。10人の胎児はダウン症ではない。

●結論
「99%の確率でダウン症の子が生まれる」と診断された場合に、
生まれてくる子供がダウン症である確率は1/11で約9%だ。

問題は11人も堕胎して、本当にダウン症であるのは1人だけ。
10人の命を無駄にするんですか?

336132人目の素数さん:2012/08/30(木) 16:22:37.25
正しいです
そもそもダウン症の子が生まれる確率がかなりすくないためこのようなことが起こります
337132人目の素数さん:2012/08/30(木) 17:03:35.30
>>335
倫理や具体的な数字はともかく、理屈としては正しい。
一般論として誤診率よりも発生率のほうが低い病気は、
陽性が出ても誤診である可能性のほうが高い。
338132人目の素数さん:2012/08/30(木) 17:18:18.05
>>335 「99%の確率でわかる」があいまい。

正常体を、正常と判断する確率がa、(当然異常と誤審する確率は、1-a)
異常体を、異常と判断する確率がb、(当然正常と誤審する確率は、1-b)
とすると、aとbが一致するとは限らない。
これが両方とも0.99だという前提で、335は書かれている

この99%は、bの事なのではないか?
339132人目の素数さん:2012/08/30(木) 17:22:53.78
補足
だって、検査をしたふりだけで、実際は何もせず、
「99.9%、ダウン症ではありません」
と言っておけば、正しいことを言っていることになる。
(ダウン症発症率が1/1000の場合)

それなのに、99%とより精度の低い数値が出てくるのは、これが、bの数字であることを強く示唆している。
340132人目の素数さん:2012/08/30(木) 17:27:13.79
8個の区別できないリンゴを赤、青、緑、黄の4つの袋に分けて入れるやり方は何通りあるか?
ただし、1個もリンゴが入ってない袋があってもよい。



計算方法も加えてお願いいたします。
341132人目の素数さん:2012/08/30(木) 17:39:40.56
リンゴを8個横に並べ、リンゴとリンゴの間、または、はじっこに棒(3本)をおき、
一番左の棒より左のグループ
一番左の棒と真ん中の棒の間のグループ
真ん中の棒と一番右の棒の間のグループ
一番右の棒より右のグループ
と、四つのグループに分けること考えればよい。
11個のもの中から3個を選ぶ方法C(11,3)=11*10*9/(3*2*1)が答え
342132人目の素数さん:2012/08/30(木) 17:59:03.66
>>317
君の専攻はなに?
343132人目の素数さん:2012/08/30(木) 18:18:25.23
>>340
4つの袋から重複を許して8回選ぶ。4H8。
344132人目の素数さん:2012/08/30(木) 20:30:29.97
345132人目の素数さん:2012/08/30(木) 21:16:40.53
スレチかもしれないけど、
他に見当たらないので(>_<)

f(x,y)=e^xsinyを
原点の回りで二次の項まで展開せよ


これがわかりません(´・ω・`)
お願いします(>_<)

346132人目の素数さん:2012/08/30(木) 22:03:59.44
教えてください。

数がある規則にしたがって並んでます。このとき□にあてはまる数字
を入れなさい。

-1 -2 -4 -8 □ -20 -22 □・・・



347132人目の素数さん:2012/08/30(木) 22:05:12.25
もう一問あります

1 2 4 7 14 19 □ 45 90 □ ・・・

お願いします。
348132人目の素数さん:2012/08/30(木) 22:56:19.89
>>345
それぞれ展開して掛ける
349132人目の素数さん:2012/08/31(金) 01:12:38.05
>>347

 a_1 = 1,
 a_k = a_(k-1) + k-1,  k:偶数
 a_k = 2*a_(k-1),    k:奇数
よって
 a_n = 7*2^(n/2 - 1) -n-3, n:偶数
 a_n = 2*a_(n-1),      n:奇数
350132人目の素数さん:2012/08/31(金) 02:29:23.43
x = a * (b ^1.5 / y ^1.5) / 2
1350 < (b ^1.5 / y ^1.5) / 2 * b / z

x = 10
y = 3000
z = 760
のとき、aとbは範囲を求めよ

↑これを解く式を教えてください。
351132人目の素数さん:2012/08/31(金) 04:03:18.89
a+bi複素数の絶対値って√(a^2+b^2)だったとおもいますが
exp(a+bi)ってなったときのexp(a+bi)絶対値ってどういう値になるの?
352132人目の素数さん:2012/08/31(金) 04:05:47.85
exp(a)
353132人目の素数さん:2012/08/31(金) 04:33:53.63
>>351
exp(bi) = ?
|exp(bi)| = ?
354132人目の素数さん:2012/08/31(金) 07:21:38.94
>>351
exp(a+bi)=exp(a)・exp(bi)=exp(a)・(cos(b)+i・sin(b))
と変形すれば分かるだろ
355132人目の素数さん:2012/08/31(金) 09:51:41.08
x^2+y^2=z^2
でx,y,zが全部整数になるものはどうやってみつけますか?
356132人目の素数さん:2012/08/31(金) 10:10:12.37
>>355
そのような組み合わせをピタゴラス数といいます
ピタゴラス数でググってみてください

(a, b, c) = (m2 − n2, 2mn, m2 + n2) or (2mn, m2 − n2, m2 + n2)
すべてのピタゴラス数はこれをみたす

あとすべてのピタゴラス数はカバー出来ないけど単純にピタゴラス数を見つける方法として

底辺を奇数aとし
斜辺c=(a^2+1)/2
もう一つの辺b=(a^2-1)/2
という組み合わせはピタゴラス数となります
例えばa=11とすると
c=(121+1)/2=61
b=(121-1)/2=60
11^2+60^2=3721
61^2=3721で成り立ちます



357132人目の素数さん:2012/08/31(金) 10:10:21.91
>>355
恒等式
  ( m + n )^2 − ( m − n )^2 = 4mn
に着目する
4mn が平方数になればよい
そうなるのは m ,n が平方数のとき
358132人目の素数さん:2012/08/31(金) 10:24:48.60
>>356-357
カムサハムニダ
359132人目の素数さん:2012/08/31(金) 12:48:53.35
lim_n_→_∞ a_n/b_n=1
のときのΣa_nが収束する十分条件ってb_nが
なんのときでしたっけ?
360132人目の素数さん:2012/08/31(金) 13:04:29.83
必要十分条件じゃなくって?

十分条件だったら「b_n=a_n」とかもアリになっちゃうけど
361132人目の素数さん:2012/08/31(金) 13:08:19.27
a_nもb_nも任意じゃなくて特定のa_n,b_nっすよ。
362132人目の素数さん:2012/09/01(土) 07:17:13.33
>>360
b_n=a_nだけじゃ十分じゃないだろ
363132人目の素数さん:2012/09/01(土) 12:46:37.08
>>362
十分条件の意味が分かってない。
364132人目の素数さん:2012/09/01(土) 13:02:45.82
b_n=a_nならΣa_nが収束する、といいたいのか?
365335:2012/09/01(土) 15:28:01.23
>>344
ありがとう
でも、具体的数字を見て、さらに分からなくなりました
表を転記すると

検査結果が
陽性で、実際ダウン症だった 209
陽性で、実際ダウン症でなかった 3
陰性で、実際ダウン症だった 3
陰性で、実際ダウン症でなかった 1468

だから、陽性反応が出た人のダウン症である確率は、209/212=0.985 (98.5%)
単純に、そういうことでいいんじゃないでしょうか?

無理やり
「まれな病気で、集団に0.1%しか存在しないとしましょう。1000人からなる集団を仮定します。」
というように、この0.1%という仮定の数字と1000人という数字をわざわざ持ってくるから、変な誘導になるのでは?

どう考えても
>>335
>「99%の確率でダウン症の子が生まれる」と診断された場合に、
>生まれてくる子供がダウン症である確率は1/11で約9%だ。」

この論法より、 上に書いた
>陽性反応が出た人のダウン症である確率は、209/212=0.985 (98.5%)
の方が実際の数字から、正しい気がしてなりません
366132人目の素数さん:2012/09/01(土) 15:49:44.28
367132人目の素数さん:2012/09/01(土) 16:19:08.75
>>365
そのデータのダウン症率12.6%は、常識の1/1000より多いがどういうこと?
368132人目の素数さん:2012/09/01(土) 16:24:27.08
>>363
こんなところでカミングアウトか?
369132人目の素数さん:2012/09/01(土) 16:57:34.84
>>367
普通に考えると、検査を受けた全ての人の数値ではなく、たまたま入手した被験者データ
陽性だった人の中の212人
陰性だった人の中の1471人
の結果を書き出しただけで、全受診者が1683人というわけではないんじゃないの?

極端な話、同じ数字(陽性・陰性ともに200人ずつ)という数字でも、なんら問題無いように
370132人目の素数さん:2012/09/01(土) 17:58:52.11
>>369
ダウン症率と診断精度の仮定を、>>365のデータ通りにすれば
ベイズ確率で>>365の直感に合うんじゃないか?
371132人目の素数さん:2012/09/01(土) 20:03:49.40
>>365
一番最初に引用された記事では、ダウン症発生率は1/1000位だったはず。
それが、別の記事では、212/1683(≒1/8)のデータを扱っています。

最初の記事では、
>> 正常体を、正常と判断する確率がa、(当然異常と誤審する確率は、1-a)
>> 異常体を、異常と判断する確率がb、(当然正常と誤審する確率は、1-b)
これらを区別することなく議論しています。
区別する必要がないならば、信用できますが、別の記事によると、間違える確率は
7倍もの違いがあり、同じとして議論するには無理があります。

一方別の記事の方では、扱われているデータのダウン症発生率が1/8になっており、
何らかの別のデータの抽出データを流用したものと思われ、純粋にその数値を信用
することは出来ません。wikiによると、
>> 25歳で1/1200、30歳で1/880、35歳で1/290、40歳で1/100、45歳で1/46という高齢出産で多発する研究報告がある。
とあり、明らかに、不自然なデータです。
372132人目の素数さん:2012/09/02(日) 08:57:20.44
8個の区別できないリンゴを赤、青、緑、黄の4つの袋に分けて入れるやり方は何通りあるか?
ただし、1個もリンゴが入ってない袋があってもよい。

d^8(x^0+x^1+...x^8)^4(0)=d^8f^4(0)




計算方法も加えてお願いいたします。
373132人目の素数さん:2012/09/02(日) 08:59:51.36
374132人目の素数さん:2012/09/02(日) 09:25:12.26
ある数xの4倍から1引いた数は、xの6倍よりも15小さい。ある数xを求めなさい。

「ある数xの4倍から1引いた数」は4x−1ですよね?
「xの6倍」は6xで・・・
「15小さい」ってどういうことですか?
375132人目の素数さん:2012/09/02(日) 09:33:55.33
お前の歳の6倍よりも15歳若いひとってことだよ。
376132人目の素数さん:2012/09/02(日) 10:34:48.30
>>374
4x-1=6x -15
14=2x
x =7
377132人目の素数さん:2012/09/02(日) 17:25:01.95
群Gが、正規部分群Nと部分群Hを持つとき、NHはGの部分群であることを証明せよ
378132人目の素数さん:2012/09/02(日) 18:05:17.57
012345678
111111111
0111111111
00111111111
000111111111
0000111111111
00000111111111
000000111111111
0000000111111111
00000000111111111
12345678987654321
012345678987654321
012345678987654321
...
9+8.2+7.3+...i*(10-i)+...+1*9
2(9+16+21+24)+25=2(70)+25=140+25=165

4H8=11!/8!3!=11.10.9/3.2.1=55.3=165
379132人目の素数さん:2012/09/02(日) 18:14:41.25
>>377
いやだ
380132人目の素数さん:2012/09/02(日) 18:19:26.80
NH(NH)^=NH(H^N^)=NH(HN)=NHN=NNH=NH
381132人目の素数さん:2012/09/02(日) 18:21:19.67
NHK

犬HK

特別アジア大好き放送局
382132人目の素数さん:2012/09/02(日) 18:35:28.49
>>380
NHN=NNH
が言えるのは
HN=NH
だからですよね?それが言えるのは何故?
383132人目の素数さん:2012/09/02(日) 18:55:14.91
>>382
そこまで聞くか
384132人目の素数さん:2012/09/02(日) 19:09:39.96
NHN^=N
385132人目の素数さん:2012/09/02(日) 19:10:47.36
GNG^=N
HNH^=N
HN=NH
386あのこうちやんは始皇帝だった:2012/09/02(日) 19:20:35.02
 
 20代と60代の、ニート・無職の虫けらども!!!!!!!!!!!!!!

 早く定職に就け!!!!!!!!!!!!!!!!!!!!!
387132人目の素数さん:2012/09/02(日) 21:15:39.80
>>385
ごめん。
^の意味がわからんかった。で、
http://homepage3.nifty.com/rikei-index01/daisu/seikibubungun.html
に証明が書いてあって、こっちで理解できたわ。悪かったな。
388132人目の素数さん:2012/09/03(月) 00:14:57.67
なんだかんだで^で通じただけ、そこそこの方の質問でした
もっと低レベルな方の質問をお待ちしています
389132人目の素数さん:2012/09/03(月) 01:11:48.05
n枚一列に繋がった切手のシートを折りたたむことを考える
全ての繋ぎ目を山折か谷折したとき一枚目の表面が一番上にくる折り方は何通りか、nの式で表せ

n=2の時は1通り
n=3の時は2通り
n=4の時は4通り
n=5の時は10通り
n=6の時は24通り
n=7の時は64通り

見た目に反してかなり難しい……
誰かお願いします
390132人目の素数さん:2012/09/03(月) 01:24:39.41
1枚目を固定して考えた場合、
1枚目と2枚目の間を山折りにして1枚目が表に出るようにすれば、
そこから先はどっち向きに折ってもよさそうな気がするんだけれど、違うの?
巻き込むようにして1枚目を隠すこともできるけれど、
山折り谷折りはそのままで重ね方を変えるだけで対応できるよね?
391132人目の素数さん:2012/09/03(月) 01:31:28.25
>>390
例えばn=5で山谷山山だと折り方2通りあるんですよねー
その発想だと最低でも2^(n-2)通りあるとしか言えないですね
392132人目の素数さん:2012/09/03(月) 02:11:59.09
>>389
ごめんなさい未解決問題でした
393132人目の素数さん:2012/09/03(月) 02:58:11.79
マジで未解決なのこれ?
394132人目の素数さん:2012/09/03(月) 04:14:26.88
重ね方も数えるのかな
395132人目の素数さん:2012/09/03(月) 07:07:54.31
>>389 >>392

n=7の時は66通り
n=8の時は174通り
n=9の時は504通り
n=10の時は1406通り


http://logsoku.com/thread/science.2ch.net/math/1062880030/

http://space.geocities.jp/fantsy10/study1.html

n≦45
http://www.ms.unimelb.edu.au/~iwan/meanders/series/semi.meanders.ser

山本幸一「郵便切手の問題」
 数セミ増刊「数学100の問題」p.48-50, (1984) 日本評論社
396132人目の素数さん:2012/09/03(月) 08:15:24.92
重ね方を問題にするなら
「一枚目の表面が一番上にくる」という条件がなくても十分に面白い問題だと思うけれど
この条件の意義は?
端が中に折り込まれる場合も含めると数が爆発的に増えてしまって難しすぎるとか、
逆に、端が表に出ている場合の問題に帰着できるから(例えば適当なところで分割する)、
この条件付きで考えるのが解く筋道だとか。
397132人目の素数さん:2012/09/03(月) 09:08:18.73
5階建てのマンションに5枚のパネルを配置する。
パネルがない階があっても、パネルが一つの階に集中しても構わない。



この場合、5枚のパネルを4つの仕切りで分けるという考え方をした場合は9C4=126通りですが。

5枚のパネルを5つのフロアに配置するから5^5=3125通りとい考え方も出来るのですが、どちらが正しいんでしょうか?
398132人目の素数さん:2012/09/03(月) 09:35:17.37
>>397
パネルに区別がつかない場合は9C4、つく場合は5^5
それにしても階を跨ってパネルを設置したりしちゃいかんのだろうか
399132人目の素数さん:2012/09/03(月) 10:08:36.41
>>372
公務員試験の問題だな
400132人目の素数さん:2012/09/03(月) 10:17:56.26
>>398
ありがとうございます。
階を跨ぐのは無しです
401132人目の素数さん:2012/09/03(月) 12:18:25.55
>>397
パネルはすべて区別がつくの?
つかないなら9C4
つくなら5^5
402132人目の素数さん:2012/09/03(月) 12:31:29.98
できたら、他に答えた人がいないかをみてから答えてほしい
403132人目の素数さん:2012/09/03(月) 13:03:32.14
すまんこ
404132人目の素数さん:2012/09/04(火) 00:18:15.86
スマートフォンのセキュリティで使われる9つの点のうち4つ以上を結ぶパターンは何通りあるか。
405132人目の素数さん:2012/09/04(火) 00:32:26.07
さんこ
406132人目の素数さん:2012/09/04(火) 06:45:24.00
一応数学・算数に分類されていたのですが…お願いします

【問】
視野を遮るものが何もない野原の一本道を歩いていったところ、最初にA電柱とB電柱との頭が一直線上に見え、
その右側にC電柱とD電柱との頭がやはり一直線上に見えた。次にしばらく歩いてから後ろを振り返ってみると、
今度はC電柱とA電柱の各々の頭が一直線上に見え、しかもB、Dの両電柱はその右側にあった。
以上のことから正しく言えることはどれか。
1. B電柱が最も高く、A電柱は最も低い。
2. A、B、Cの電柱の高さは変わらない。
3. D電柱はB電柱よりも低い。
4. B電柱は一番歩道に近い。
5. C電柱が最も低い。
【解答】
5. C電柱が最も低い。
----------------------------------------------------------------------------------------
模範解答には左図のような位置関係が書かれていましたが、
右図のように考えました。条件からは、右図も成立すると思うのですが、これでは駄目なんでしょうか?
この場合正答の「C電柱が最も低い」は成立せず、選択肢に解答がないことになりますが…
ttp://ranobe.sakura.ne.jp/src/up104705.jpg
407132人目の素数さん:2012/09/04(火) 08:25:53.94
>>406
問題文に間違いが無いとすると、疑問の通りのように思える。

ただ、「C電柱とA電柱の各々の頭が一直線上に見え」とアルファベット順ではない表記があるので、
順番が電柱の前後を表していることは明らかで、「A電柱とB電柱との頭が一直線上に見え」という表現では、
BよりもAの方が手前にあるという意味を含むということなのかも知れない。
たしかにその方が自然な表現だとは思うが、若干の疑問は残る。
408132人目の素数さん:2012/09/04(火) 09:58:54.07
京大のぱくり?
409132人目の素数さん:2012/09/04(火) 12:42:58.25
>>406
近い方を先に言うと言う前提がある訳でもないから欠陥問題だな
410132人目の素数さん:2012/09/04(火) 13:11:53.44
複数の電柱の名を言う時は、近い電柱から順に言う
遠い電柱からは言わない、ましてやランダムに言うなど言語道断。

というテスト。

411132人目の素数さん:2012/09/04(火) 13:20:46.74
>>406
たとえ左図でも「5. C電柱が最も低い」 を正しいというためには
「電柱は歩行者よりも十分高い」という仮定が必要。

電柱が、放牧家畜の脱走防止などに使う背の低い位置の電線を
支える電柱だったりしたら(こういう電柱は広い野原にはいかにも有りそう)
成立しない。

つまり出題者は、論理的思考能力などの高い人材よりも
自分と常識を共有する人材を必要としている。

412132人目の素数さん:2012/09/04(火) 13:25:35.37
いわゆる空気が読めるかどうかのテスト

数学ではないが、数学の問題をたくさん見ると時々紛れている
413132人目の素数さん:2012/09/04(火) 13:43:23.87
アスぺ殺し用問題だろ
414132人目の素数さん:2012/09/04(火) 13:48:01.56
「振り返って見る時に振り返る軸が地面に垂直」
という仮定も必要だな。
415132人目の素数さん:2012/09/04(火) 13:52:38.71
>>411
歩行者がキリンではないという仮定も必要
416132人目の素数さん:2012/09/04(火) 13:54:04.25
>>414
それは思いつかなかったw
417132人目の素数さん:2012/09/04(火) 14:00:16.71
野原が平らで地球が平らで象が支えていてその下に亀が支えているという仮定も必要だな。
418132人目の素数さん:2012/09/04(火) 14:42:25.50
イデアルと環ってあるじゃないですか
それはR/Iみたいに書くじゃないですか?
これの分数の計算みたいな規則ってあるんですか?
あるのなら教えてください。
419132人目の素数さん:2012/09/04(火) 14:55:14.99
>>417
支えているのはヘラクレスでも構わないので
その仮定は必要条件ではない
420132人目の素数さん:2012/09/04(火) 16:07:11.37
>>418
計算規則じゃなく表記法
同値類の集合は普通そう書く
421132人目の素数さん:2012/09/04(火) 16:09:38.00
計算規則って言うのは
R/I+P/J=P+R+I+J
見たいなのですよ?
上のは例なので成り立た津かどうかはわかりませんけど。
422132人目の素数さん:2012/09/04(火) 16:17:30.45
>>421
その数式みたいなのは何?
そんなのあるの?
423132人目の素数さん:2012/09/04(火) 16:25:00.03
>>418
教科書読めよ
424132人目の素数さん:2012/09/04(火) 17:56:49.43
>>418
ある。イデアル算という。
425132人目の素数さん:2012/09/04(火) 18:39:05.88
1000^1/5
近似値を出せって問題なんだがどうやるの?
工業大学一年です

2^10=1024だから
1000≒1024で 答え 4 って提出したら怒られた
426132人目の素数さん:2012/09/04(火) 18:56:02.35
>>425
桁数指定がないんならそれでいいんじゃね。
427132人目の素数さん:2012/09/04(火) 19:08:33.98
>>425
1000^1=1000 だから1000/5=200.0000000 くらいでいいんじゃね?

>>426
授業でお手本やってんの聞いてなかったんだろう。
4*(1000/1024)^(1/5)=4*(1-24/1024)^(1/5) を評価するのだろう。
428132人目の素数さん:2012/09/04(火) 19:34:33.63
誤差0.5%やないの
429132人目の素数さん:2012/09/04(火) 19:46:17.85
誤差少なくするにはどうすればいいの?
430あのこうちやんは始皇帝だった:2012/09/04(火) 20:04:30.81

 お前たちは、定職に就くのが先決だろがあああああああああああああ!!!!!!!!!!!!!!!!

 ニート・無職の、ゴミ・クズ・カス・無能・虫けらのクソガキどもがあああああああああああああああああ!!!!!!!!!!!!!!!!!!!!!
431132人目の素数さん:2012/09/04(火) 20:28:31.34
誤差を圧縮すれば
432132人目の素数さん:2012/09/05(水) 02:42:06.08
>>404もいい問題だと思うけど誰も答えられない?
433132人目の素数さん:2012/09/05(水) 02:44:24.75
>スマートフォンのセキュリティ
これを知らない人にとっては問題文が意味不明
434132人目の素数さん:2012/09/05(水) 04:41:51.36
>>432
お前が答えろよ

答えられないのに上から目線
435132人目の素数さん:2012/09/05(水) 09:02:39.41
http://beust.com/weblog2/archives/000497.html
ググればすぐ出るやん
436132人目の素数さん:2012/09/05(水) 15:59:47.29
>>425
1000を 1024で代用するのは、とてもよい着眼である。これに基づき、次のように計算できる。
1000^(1/5) = (1024 - 24)^(1/5) = (1024)^(1/5)・(1-24/1024)^(1/5) = 4 (1+h)^(1/5).
ここで h = -0.0234だ。(1+h)^a = 1 + ah + (1/2)a(a-1)h^2 + … (a=1/5)と展開できるから、
このあたりまで拾って、1000^(1/5) ≒ 4×0.9952685547 = 3.981074219。ちなみに
真値は 3.981071706
437436:2012/09/05(水) 16:01:47.59
って、>>427 がもうやってたか。
438436:2012/09/05(水) 16:10:53.34
じゃ、x^5 - 1000 = 0 をニュートン法で解く。 x' = x - (x^5-1000)/(5x^4)という
漸化式で、初期値 4から出発して 2回くりかえせば、
x'' = 131095482626957/32929696272200 = 3.98107172.
439436:2012/09/05(水) 16:32:50.29
お、x^3 - 1000/x^2 ならニュートン法で 3次の収束だ。 x' = (2x^5 + 3×1000)x / (3x^5 + 2×1000)。
x = 4から出発して、1回で 6桁、あう。 (x' = 1262/317 = 3.981072555)。
2回で 20桁、 3回で 60桁、合う。これ、最強。
440132人目の素数さん:2012/09/05(水) 16:33:44.62
計算機使えばすぐ出るだろ。
441436:2012/09/05(水) 16:51:53.25
> 計算機使えばすぐ出るだろ。
その計算機の、効率のよいプログラムを書く訓練、あるいは出してきた
答を評価する訓練には、こういう課題は大切だと思うよ。
442132人目の素数さん:2012/09/05(水) 17:01:09.37
授業で習ったのは二項定理による1次の近似だと想像。
443132人目の素数さん:2012/09/05(水) 17:27:40.22
分数を小数点にするところで計算機使っているようにしか思えないんですけど。
444132人目の素数さん:2012/09/05(水) 17:37:31.73
こういうのは、暗算の練習じゃないんだから、計算機使っていいんだよ。
関数電卓の√xとか、 x^y みたいな機能を効果的に実現するのに、どんな
方法があるかを学習するのよ。「数値計算」というのよ。
445132人目の素数さん::2012/09/06(木) 01:47:33.41
0の複素数乗,0^z=0 (但し,z∈C\Q)となるのでしょうか?
分かりやすく教えてください。
446132人目の素数さん::2012/09/06(木) 01:49:35.43
zが無理数なら0^z=0?
zが複素数なら0^z=exp(zln(0))なのでどう定義する?
447132人目の素数さん:2012/09/06(木) 01:53:41.49
くだらない問題で暇つぶしできていいね
448132人目の素数さん::2012/09/06(木) 01:57:26.81
是非,教えてください。
449132人目の素数さん:2012/09/06(木) 06:02:51.55
いやです
450132人目の素数さん:2012/09/06(木) 10:05:12.09
>>418
「じゃないですか」って言い回しを使う奴は嫌い
451132人目の素数さん:2012/09/06(木) 10:06:21.98
つまんねーねた
452132人目の素数さん:2012/09/06(木) 12:46:27.21
数列 a_n=m^n (mは正の整数) を素数pで割った余りの数列って必ず周期的になるのでしょうか?
453132人目の素数さん:2012/09/06(木) 12:51:53.39
>>452
なるよ。
454132人目の素数さん:2012/09/06(木) 12:53:51.30
>>453
自明だった.逆に周期的にならないような数列ってどんなものがあるのでしょうか?
455132人目の素数さん:2012/09/06(木) 12:54:31.13
>>454
等差数列とか。
456132人目の素数さん:2012/09/06(木) 13:00:21.88
>>454
0,1,0,0,1,0,0,0,1,0,0,0,0,1,0,0,0,0,0,1,0,0,0,0,0,0,1,……
457132人目の素数さん:2012/09/06(木) 13:00:42.19
ん?a_n=a*n+b≡a*(n+p)+b mod.p だから最長でも周期pを持つのでは?
458132人目の素数さん:2012/09/06(木) 13:05:04.89
>>457
等差数列そのものは公差0以外なら周期的にならないだろ。
質問文をきちんと書いてくれよ。
459132人目の素数さん:2012/09/06(木) 13:11:17.56
>>454
a[n]=nの約数の個数 とか
460132人目の素数さん:2012/09/06(木) 13:19:04.93
例だけなら a_n=乱数の100倍の整数部分 mod p とかでいいじゃん
初頭関数でそういうのはあるかってのはわからん.
461132人目の素数さん:2012/09/06(木) 13:21:48.54
>>460
擬似乱数は周期的ry
462132人目の素数さん:2012/09/06(木) 13:21:59.59
乱数を持ち出すのはアリ?
463132人目の素数さん:2012/09/06(木) 13:58:08.68
素数列とか
464132人目の素数さん:2012/09/06(木) 16:50:42.16
a[n]=f(a[n-1],a[n-2],…,a[n-m])の形に表せるなら
必ず周期を持ってその長さはp^m以下
465132人目の素数さん:2012/09/06(木) 17:00:35.07
a[n]=Floor(π*10^n) とかは?
466132人目の素数さん:2012/09/06(木) 17:48:28.90
>>464
mod p をfの計算より先に出来る場合って制限が必要かな?
467132人目の素数さん:2012/09/06(木) 17:57:27.10
どういうこと?
468132人目の素数さん:2012/09/06(木) 22:55:54.92
http://www.47news.jp/47topics/e/223273.php

ここの数学2bの(2)の問題なんですけど、解説見てもさっぱりわかりません
数学2b習ってないので詳しく教えてほしいのです
469132人目の素数さん:2012/09/06(木) 22:57:10.49
ファイバーゼーションってどういう意味でしょうか?
470132人目の素数さん:2012/09/06(木) 23:00:43.03
前後関係がないとさっぱりわかりません><
和訳なら、たぶんファイバー化
471132人目の素数さん:2012/09/07(金) 02:54:01.23
>>468
どの問題のことなのかチンプンカンプンで〜す。
472132人目の素数さん:2012/09/07(金) 02:59:49.90
>>468
47newsは「年度」の意味を分っていないようだ。
473132人目の素数さん:2012/09/07(金) 07:55:52.72
>>472
474132人目の素数さん:2012/09/07(金) 09:40:49.04
>>471
数学2、数学Bの2問目のsin,cosのやつです
475132人目の素数さん:2012/09/07(金) 11:02:48.17
非可換群ってあるじゃないですか?
ab≠baですよね。
これを可環群にするために元に順番をつけれるようにして
順番を並び替えてから計算するってやると可換じゃないですか?
a_1 b_0=b_0 a_1=b a
でも同じ番号が出てきた場合どうしたらいいですか?
やりかたおしえてください。
476132人目の素数さん:2012/09/07(金) 12:31:07.85
可環群は可換群の誤記とみなすとしてもだな…

"元に順番"というのは、論ずる非可換群を構成する二項演算において
左からの出現順に元へ番号をふっていくということか?

順番を並び替えるってどうやって?
非可換群だからそのままじゃ順番は入れ替えちゃいけないぞ?
たとえ番号をふったとしてもだ

同じ番号?左からの出現順に番号をふったとして
どうして同じ番号が現れるんだ?

それとも俺がおかしいのか?何か理解できていないのだろうか
477132人目の素数さん:2012/09/07(金) 13:23:26.84
この方程式はどうやって解くのでしょうか?
手順を詳しく教えて下さい、よろしくお願いします!
http://iup.2ch-library.com/i/i0736981-1346991786.jpg
478132人目の素数さん:2012/09/07(金) 13:24:49.55
>>476
b_1 a_0 c_3=a b c
(a_0とa_1とは別物として)
にするんですよ。
479132人目の素数さん:2012/09/07(金) 14:10:35.43
結合則が成り立たないから群ではない
(a0c2)b1≠a0b1c2
480132人目の素数さん:2012/09/07(金) 14:12:40.38
>>477
ただの3次方程式
481132人目の素数さん:2012/09/07(金) 14:16:16.17
>>480
ありがとうございます。解き方が分かりません。解答までの途中式が見たいです
482132人目の素数さん:2012/09/07(金) 14:19:37.92
>>481
分母を払う。
483132人目の素数さん:2012/09/07(金) 14:27:58.09
分母のはらいかたが分からないです…すみません
484132人目の素数さん:2012/09/07(金) 14:31:17.70
>>479
結合法則って始めのカッコと後ろのカッコを取り替えても
答えは同じってことだからみたしていますよ。
(a0c2)b1=a0(c2b1)=abc
485132人目の素数さん:2012/09/07(金) 14:34:22.65
問題なのはa0b0の計算だと思うんですけど
a0b0の場合はアルファベットの小さい順ってことで
いいですかね?
486132人目の素数さん:2012/09/07(金) 14:45:49.97
>>475
何を言っているのかよくわからない
例としてルービックキューブの操作で説明してくれ
487132人目の素数さん:2012/09/07(金) 14:49:41.54
「非可換群を可換群にする」 とは、別の演算を定義するということか?
そうして出来上がったものは、元の非可換群とは何の関係もない代物ではないのか?
そんなものを定義して何か意味があるのか?
488132人目の素数さん:2012/09/07(金) 14:53:36.47
数字を時間で言うと
時間が無いとすると
abcdefghi
みたいに順番に書かないといけないけど
時間0にaが発生したことをa0とかくと
a0b1c2
ってやればバラバラに計算しても元に戻せる意味があるじゃないですか?
489132人目の素数さん:2012/09/07(金) 14:58:18.21
>>488
名前を書き換えることで非可換な群が可換になる、と主張しているの?
490132人目の素数さん:2012/09/07(金) 15:02:03.00
もういいです。さようなら。
491132人目の素数さん:2012/09/07(金) 15:51:42.70
どなたか>>477をお願いします
492132人目の素数さん:2012/09/07(金) 15:52:29.52
例えば非可換群となる集合Gと二項演算Xがあり
Gの任意の元a,bにおいてa X b = b Y a となる二項演算Yがあるとしよう

いまGの任意の元a,b,cにて
c X b X a = c X (b X a) = c X (a Y b) = (a Y b) Y c = a Y b Y c
(最後の=は確認が必要だが成立する)…なるほど

Gの任意の元g1,g2,g3,…,gnにおいて例えば
g5 X g7 X g2 X … X g3 と同値な
Z1 g1 Z2 g2 Z3 … Zn gn Zn+1
という表記は得られるだろうか
ただしZnには「幾つかの閉じ括弧〜XかY〜幾つかの開き括弧」が入る

…のようなことを考えていたのだろうか?

だけど正直、そのような表記を得られるとしても、果たして旨みがあるのだろうか
XとYでは演算そのものが違うから結合法則が成立しない以上(*)
結局計算順序は入れ換えられないし

* c X b X a = c X (b X a) = c X (a Y b) ≠ (c X a) Y b = b X (c X a) = b X c X a
493132人目の素数さん:2012/09/07(金) 16:03:53.65
>>491
http://www.wolframalpha.com/input/?i=958766%3D30000%2F%281%2Br%29%2B30000%2F%281%2Br%29^2%2B1030000%2F%281%2Br%29^3
元問題は手計算でrを求めろって問題じゃないだろ、いくら何でも
494132人目の素数さん:2012/09/07(金) 16:08:02.80
回答って書いてあるからrは答えなくていいんだよ。
495132人目の素数さん:2012/09/07(金) 16:25:21.62
>>493
これは、rを求める問題だとしたら、「 100万円を年利率 rの元利均等(毎年定額 3万円ずつ返済)
で借り、3年目で早期解約したら、95万8千円請求された。年利率を求めよ」かな?
496132人目の素数さん:2012/09/07(金) 16:31:22.18
>>493
これって手計算では難しいんですか?
497132人目の素数さん:2012/09/07(金) 16:33:07.62
>>494
この式を作るのが問題になっていて、そこは分かるんですが、もしこういう式を解く問題が試験に出るなら出来ないとまずいかなと思いまして…
498132人目の素数さん:2012/09/07(金) 16:34:19.30
普通は rはわかっていて、「3年で早期返済するには、いくら返さなければいけ
ないか」という設問だよね。
499132人目の素数さん:2012/09/07(金) 16:34:56.83
電卓持ち込みでもこんな問題は出ないんじゃないかな
500132人目の素数さん:2012/09/07(金) 16:37:29.76
>>499
そうなんですか!すみません…お騒がせしました!
501132人目の素数さん:2012/09/07(金) 17:23:58.91
wolfram alphaにも「ググれ」みたいな決まり文句が欲しいな。
502132人目の素数さん:2012/09/07(金) 21:22:16.96
>>474
第1問の(2)のことか?
sinα=cos2β 0≦β≦πより0≦2β≦2π
α=π/6のときsinα=1/2だから、cos2β=1/2 から 2β=π/3、5π/3。
これから β=π/6、5π/6。

0≦α≦π/2のとき、2β=π/2-α、2π-(π/2-α)、即ち β_1=π/4-α/2、β_2=3π/4+α/2
このとき α+β_1/2+β_2/3=11α/12+3π/8

π/2≦α≦πのとき、2β=π/2-(π-α)=-π/2+α、2π-(-π/2+α)=5π/2-α
このとき α+β_1/2+β_2/3=13α/12+7π/24

よって、

0≦α≦π/2のとき、 3π/8≦α+β_1/2+β_2/3≦5π/6
π/2≦α≦πのとき、 5π/6≦α+β_1/2+β_2/3≦11π/8

まとめて 3π/8≦α+β_1/2+β_2/3≦11π/8。

yが最大になるのはα+β_1/2+β_2/3=π/2=3π/6のときだから
11α/12+3π/8=π/2から α=3π/22。

yの最大値は1

503132人目の素数さん:2012/09/08(土) 02:28:51.13
>>501
使い方が難しいからそういう時は黙って式を貼り付けるのがよろし
504132人目の素数さん:2012/09/08(土) 13:22:08.90
>>501
wfaれ
505132人目の素数さん:2012/09/08(土) 13:45:19.00
小学生でも解けるのに、大人は間違えやすい算数の問題

 新人タレント・柑乃美優(かんのみゆう)ちゃんはかわいいルックスとは裏腹に、
実は大学で数学を専攻し、数学の塾講師もしているバリバリの理系女子。
現在、“かわいすぎる数学講師”として活動中だ。
そんな美優センセイ得意の数学から、小学生でも解けるのに大人は間違えやすい話題の算数問題を出題してもらった。
「就職活動のSPI以来、数学なんてやっていないよ!」という人も、ぜひチャレンジを!

【問1】
たろう君は自宅から花子さんの家までを、
行きは時速4km、帰りは時速6kmで往復しました。
このとき、往復の平均速度は時速何kmでしょうか?

「道のり・速さ・時間の問題は小学校で習う問題。
10秒くらいでパッと答えられたらカッコいいな♪」(美優センセイ)

http://nikkan-spa.jp/276626

506132人目の素数さん:2012/09/08(土) 18:47:31.62
>>505

【正解】
 正解は『時速4.8km』です。

【解説】
『時速5km』……じゃないですよ!

正解は『時速4.8km』でした。

お弁当箱の形をした「み・は・じ」を覚えていますか?
速さは「道のり/時間」で求められるので、それぞれをちゃんと計算するのがポイントです♪
今回は道のりがわからないので、仮に Y (km) とします。<注1>

すると、
●行き …… Y を時速4kmで歩くと、Y/4 時間かかる。

●帰り …… Y を時速6kmで歩くと、Y/6 時間かかる

⇒ 往復2Y(km)を計 (Y/4 + Y/6) 時間かかって歩いたので、

2Y/(Y/4 + Y/6) =4.8(km/時)となります♪
507132人目の素数さん:2012/09/08(土) 19:40:11.33
速度なのにベクトル無視していいの?
508132人目の素数さん:2012/09/08(土) 19:52:29.06
アスぺのいちゃもんか?
509132人目の素数さん:2012/09/09(日) 01:33:44.56
>>506
調和平均という便利な言葉しらないのか?
510132人目の素数さん:2012/09/09(日) 01:38:19.19
「小学生でも解けるのに」 て問題に調和平均を持ち出さない
511132人目の素数さん:2012/09/09(日) 04:08:01.12
>>506
小学生でも みはじ とかで覚えてるのは最初だけ
512132人目の素数さん:2012/09/09(日) 06:38:38.62
>>510
アホ。
この機会にその概念と言葉を教えろということだよ。
513132人目の素数さん:2012/09/09(日) 10:47:25.60
小学生には難しすぎる
小学生の範囲ではない
514132人目の素数さん:2012/09/09(日) 11:33:38.17
出題文をみれば分るとおり、対象は小学生ではない。
515132人目の素数さん:2012/09/09(日) 12:23:10.65
問題文のどこに小学生が対象でないと書いてある?

もちろん大人が解くことを禁止はされないが
一般に「算数」の問題は小学生が対象

516132人目の素数さん:2012/09/09(日) 12:25:04.21
「対象に小学生が含まれる」 と 書かないと 余計な反論が来そうだな

論点は、対象に小学生が含まれるか含まれないかであって
その他が対象になっているかいないかではない。
517132人目の素数さん:2012/09/09(日) 12:54:27.06
>>507
「速さ」か。
確かに算数の問題集ではすべて「速さ」かも
美人すぎる出題者が速度と速さの違いを理解していないのかも
518132人目の素数さん:2012/09/09(日) 14:01:58.65
>>506

「平均」速度の定義にもよるが....

時間平均: ∫vdt/∫dt = 2Y /∫(1/v)dx = 4.8

距離平均: ∫vdx/∫dx = ∫vdx / 2Y = 5.0
519132人目の素数さん:2012/09/09(日) 14:03:54.76
小学生には難しすぎる
小学生の範囲ではない
520132人目の素数さん:2012/09/09(日) 14:38:50.10
>>515
小学生が対象なら「小学生でも解ける」とは言わない
521132人目の素数さん:2012/09/09(日) 14:44:48.26
>>515
問題文、かwww
522132人目の素数さん:2012/09/09(日) 15:01:55.30
でも算数ってことは小学生の習う範囲でってことだからな
わかってて4.8と答えるのが正解
523132人目の素数さん:2012/09/09(日) 15:34:00.03
>>520
小学生が対象でないなら「数学」と言うのではないだろうか?

小学生が対象でない「算数」を認めるかどうかの違いだな。
「算数とは小学生が対象の数学である」以外の「算数」の定義が必要になるわけだ。
524132人目の素数さん:2012/09/09(日) 16:13:55.68
Aは隣町の郵便局まで出かけました。
Aが出かけて五分後、弟は兄の忘れ物に気付いて自転車で追いかけました。
自転車は二等辺三角形です。
等差数列の忘れ物は、兄に何分後に届く?
525132人目の素数さん:2012/09/09(日) 16:15:39.32
Aは隣町の郵便局まで出かけました。
Aが出かけて五分後、弟は兄の忘れ物に気付いて自転車で追いかけました。
自転車は二等辺三角形です。
弟は兄に5/8の確率で追いつきます。追いついた時、十分条件は30分後でした。
忘れ物は、兄に何分後に届くでしょうか。
526132人目の素数さん:2012/09/09(日) 16:17:47.96
おもしろくない
527132人目の素数さん:2012/09/09(日) 16:27:27.88
急に発狂するのはなぜなんだろう?
528132人目の素数さん:2012/09/09(日) 19:08:44.13
>>523
算数に「数を算える」以外の意味があるのか?
小学生対象の数学を算数というのは定義じゃなく言葉の使い方だろ
529132人目の素数さん:2012/09/09(日) 19:49:57.36
>>505

元NHKアナの神田愛花もいますけど。

かわいさでは神田うのかな、年増だが。
530132人目の素数さん:2012/09/09(日) 19:55:15.51
531132人目の素数さん:2012/09/10(月) 02:47:58.23
8 桁の整数 N が見えたのでとりあえず素因数分解を試みようと思った.
たまたま(?)近くにあった平方数 A で近似したら差 A-N が平方数になり瞬殺だった.
と思いきや,よく考えると A は平方数ではなく平方数の 2 倍であった.
しかし A-N が N を割り切ったので結局瞬殺だった.

→ 調べたら,全然たまたまではなかった.
問:N は何か.
532132人目の素数さん:2012/09/10(月) 03:17:10.27
>>531
解こうとした奴がバカ
これが完全解
533132人目の素数さん:2012/09/10(月) 04:35:34.61
これがどうしても解けません。ここになら天才がいると聞いてきました。

ttp://www.illuminatiorder.info/Spez/square.htm

たぶん、こんなかんじで数式に当てはめれば良いんだと思うんだけど...ちょっと私には頭脳が足りないみたいでだれかご協力お願いします。
http://ja.wikipedia.org/wiki/魔方陣
534533:2012/09/10(月) 04:56:34.19
粗悪ルアーで遊ぶかは、皆さん次第
535132人目の素数さん:2012/09/10(月) 10:49:43.20
リー群の話にかんするおそらく単純な質問です
左不変微分作用素のつくる環は、左不変微分作用素と
定数関数で生成される。
Eijについて、XEij=Σx(il)∂x(lj)
に対して、対応する多項式が
PXEij(u)=u(ij)
となるのだそうです。
これがなんでかわかりません。

536132人目の素数さん:2012/09/10(月) 11:23:42.91
定義をググったが見つからんなー
537132人目の素数さん:2012/09/10(月) 11:58:35.94
微分作用素がX(f(ax))=Xf(ax)のとき左不変だそうで
多項式は点を固定した上で偏微分記号を
その微分回数を累乗に微分する変数を変数に
したもので、主に最高次を頭部としてあって
その考察にかかれています
538132人目の素数さん:2012/09/10(月) 12:04:42.24
おそらく前に書かれていることの"ワン•ステップ"
であることは確かだとおもいます
539132人目の素数さん:2012/09/10(月) 12:36:08.36
>>535
>左不変微分作用素のつくる環は、左不変微分作用素と
>定数関数で生成される。

それが定義なら自明
540132人目の素数さん:2012/09/10(月) 13:42:55.06
おそらく定義をまともに説明できたら自明に分かる事だろう
>>537で説明したつもりではダメだが
541132人目の素数さん:2012/09/10(月) 13:50:18.06
その内容を証明するために、帰納法をつかうため
同じ頭部を生成できることを示していると
思うのですが、最初の式から、二番目の多項式に
なることがわかりません。
当然、PXEij=Σx(il)u(lj)だと思いますが
なぜこれが、u(ij)に等しいのか分かりません
542132人目の素数さん:2012/09/10(月) 13:55:49.88
全く理解していないことは良く分かった
543132人目の素数さん:2012/09/10(月) 15:17:26.18
ど素人乙
544132人目の素数さん:2012/09/11(火) 03:41:58.27
a,b∈Cについてa∈{□×b,□×b,□×b}はa^3=b^3であるための必要十分条件である

□に当てはまるものを埋めなさい


この問題教えてください
545132人目の素数さん:2012/09/11(火) 04:04:05.69
(二つのlimは自由に入れ替えれるか)

(α,β)∈(R×{∞})^2とする時,
lim_{x→α}f(x,y)とlim_{y→β}f(x,y)とも収束するならば
lim_{x→α}lim_{y→β}f(x,y)=lim_{y→β}lim_{x→α}f(x,y)
は成立ちますでしょうか?

もし成立たないならそのような例を教えていただけたら助かります。
546132人目の素数さん:2012/09/11(火) 04:05:19.40
a^3=b^3 をaについて解けばいい
(a-b)(a^2+ab+b^2)=0 と変形する
547132人目の素数さん:2012/09/11(火) 09:10:39.61
age
548132人目の素数さん:2012/09/11(火) 11:12:49.75
〜の補題っていう名前の有名なやつが、どんな定理の補題かは
どうやって分かるんですか?
549132人目の素数さん:2012/09/11(火) 11:39:58.78
いろんな定理に使えるから有名なんだ
550132人目の素数さん:2012/09/11(火) 11:47:06.10
なるほど、俺はてっきり何かの定理を証明するために使った補題が
補題だけ有名になったのかと思っていたよ。
でもそういうのもあるんじゃないんですか?
551132人目の素数さん:2012/09/11(火) 11:58:15.37
ドラクエで竜王を倒すことが最終目的なのに
世界中に散らばっているコイン集めがネタになり有名になってしまい
しまいにゃ、コイン集めが面白くなって
それに奔走するようなものだ
552132人目の素数さん:2012/09/11(火) 12:01:13.93
それを本末転倒という
553132人目の素数さん:2012/09/11(火) 12:01:46.89
コインはドラクエ4からだろ。
554132人目の素数さん:2012/09/11(火) 12:09:24.06
細けぇこたぁいいんだょ
555132人目の素数さん:2012/09/11(火) 12:17:18.31
あ、1nの頭部だった。
556132人目の素数さん:2012/09/11(火) 12:27:13.90
コインはカジノで世界中で集めるのはメダルだよ。
557132人目の素数さん:2012/09/11(火) 12:30:12.30
細けぇこたぁいいんだょ
558132人目の素数さん:2012/09/11(火) 12:42:38.32
ドラクエをあまり知らない先生が
生徒たちに分かりやすい例えを言ったつもりだが
かえってつっこみを受け
よけい訳が分からなくなってしまったという悲(喜)劇の例であった
559132人目の素数さん:2012/09/11(火) 13:17:46.88
前の問題は解決しました。
Banach sp c0の点列{xn}nについて、xnは複素数列とする。
これが0に弱収束するための必要十分条件は
sup(n)||xn||<∞かつ、全てのkで、xnk→0となることである
これを証明したいのですが、十分はε論法を二回適用で
容易に示せるようですが、必要のためsupの条件を
否定したときがわかりません。
560132人目の素数さん:2012/09/11(火) 14:04:36.67
質問です。
lim┬(n→∞)⁡〖1/n(1/(√(n^2+n)-√(n^2+2n))+1/(√(n^2+3n)-√(n^2+4n))+⋯1/(√(〖3n〗^2-n)-√(3n^2 )))〗
区分求積を使うと思うのですが、有理化などをしたあとに、√の中にk/n以外に1/nが残ってしまいます。
561132人目の素数さん:2012/09/11(火) 14:09:58.47
>>560
>有理化などをしたあとに
結果を詳しく
562132人目の素数さん:2012/09/11(火) 14:17:37.22
>>545
(α,β)=(0,0)
f(x,y)=x/(x+y) (原点では定義しない)
563132人目の素数さん:2012/09/11(火) 14:46:46.06
559です
とても馬鹿に思われてしまいそうですが、
たとえば、
xn=(0,0,・・・,logn(n番目),0, ・・)
は0に弱収束しませんか?
564132人目の素数さん:2012/09/11(火) 15:18:17.18
560です。
lim┬(n→∞)⁡〖1/n 農(k=1)^n▒(√(n^2+(2k-1)n)+√(n^2+2kn))/(-n)〗
分母の1/nをルートの中に入れるとk/n以外に1/nが残ってしまいます。
565132人目の素数さん:2012/09/11(火) 15:45:51.64
>>559
これで意味が伝わると思ってるのか?
566132人目の素数さん:2012/09/11(火) 16:18:40.81
>>564
和の最初の方を何項か書き出してみて
567132人目の素数さん:2012/09/11(火) 16:39:02.03
>>566
なるほどです!項を書き出してからもう一度シグマの形にすれば、できました!
ありがとうございます!
568132人目の素数さん:2012/09/11(火) 19:08:01.31
xnは複素数列{xnk}であるでした。おねがいします
569132人目の素数さん:2012/09/11(火) 19:10:49.16
すみません{xnk}kでした。後は間違いがなければ、
書き方以外元の問題と一緒です
570132人目の素数さん:2012/09/11(火) 19:14:02.76
xnk なんて意味不明の書き方なんだよ。
571あのこうちやんは始皇帝だった:2012/09/11(火) 19:52:37.68

 また、お前たちか! 20代と60代の、ニート・無職の、女性恐怖症の、ゴミ・クズ・カス・無能・虫けらのクソガキども!

 死ね!!!!!!!!!!!!!!!!!!!!!!!!!!!!!!!!!!!!!!!!!!!!
572132人目の素数さん:2012/09/11(火) 19:53:54.10
x以下添字ですが、ただし、kはnの添字で部分列という意味ではなく
xnがc0の元である。つまり、そのdual space はl1であるといったいみでの
弱収束と題意をとりました。
573132人目の素数さん:2012/09/11(火) 22:00:49.65
>>572
x以下添字ですが、なんて他人にはつたわらんよl。
x_nとかx(n)とか、その書き方に対してkの意味を明らかにする記号の使い方を工夫しろよ。
574132人目の素数さん:2012/09/11(火) 22:49:18.83
575132人目の素数さん:2012/09/12(水) 01:22:11.83
>>562訂正
原点では定義しない→直線x+y=0上では定義しない
576132人目の素数さん:2012/09/12(水) 10:20:58.81
すみません。xnは複素数列{xnk}kとする。
577132人目の素数さん:2012/09/12(水) 10:24:32.73
559です。576は間違えたので574のものが元の問題なのでお願いします。
578132人目の素数さん:2012/09/12(水) 13:53:32.25
>>571
チネ
579132人目の素数さん:2012/09/12(水) 19:58:38.96
>>571
ちゃんとくそがきらしくくそするんで、
みんななんかエレガントな回答おねがいします。
580132人目の素数さん:2012/09/12(水) 20:21:28.59
581132人目の素数さん:2012/09/12(水) 23:46:38.43
x+2y+i(−2x+y)を、0→1→1+i→i→0の正方形で積分

@0→1:1/2+i
A1→1+i:2−(3/2)i
B1+i→i:−3/2
Ci→0:−1-(1/2)i
@〜C:−3i
与式は正則なはず、、だからこの積分は0になるはず、、

どこかで計算間違いまたは勘違いしてますでしょうか、、
582132人目の素数さん:2012/09/12(水) 23:58:30.49
>>581
全部
583132人目の素数さん:2012/09/13(木) 01:44:23.39
>>581
0→1:1/2-i
1→1+i:1/2-i
1+i→i:-3/2+4i
i→0:1/2-2i
(1/2-i)+(1/2-i)+(-3/2+4i)+(1/2-2i)=0
584132人目の素数さん:2012/09/13(木) 07:41:41.71
>>581
1と3、2と4を立式して足し合わせれば、
積分計算が楽になり、正則でならなければ
ならない意味も一目瞭然です。
585581:2012/09/13(木) 19:46:41.80
ありがとうございます!
でも>>583は違うような、、
0→1:1/2-i
1→1+i:3/2+2i
1+i→i:-5/2
i→0:1/2-i
でないですか?


586あのこうちやんは始皇帝だった:2012/09/13(木) 19:50:56.56

 お前たちは、定職に就くのが先決だろがああああああああああああああ!!!!!!!!!!!!!!!!!!!!!!!!

 ニート・無職の、ゴミ・クズ・カス・無能・虫けらのクソガキどもがあああああああああああ!!!!!!!!!!!!!!!!!!!!!!!!!!!!!
587132人目の素数さん:2012/09/13(木) 21:51:28.56
えーん、おれの>>574だれか答えかいてよー
588132人目の素数さん:2012/09/13(木) 23:55:53.37
貼ってあるリンクなど見ない人は多い
589132人目の素数さん:2012/09/14(金) 00:14:28.76
小寺平治のクイックマスター微分積分の平面と点の最短距離で間違った答えかいてる。

正解はv*(p-q)/|v|です。
590132人目の素数さん:2012/09/14(金) 00:18:12.50
>>587
xnkwww
591132人目の素数さん:2012/09/14(金) 00:20:07.04
完備だから当然だろ。 QED
592132人目の素数さん:2012/09/14(金) 02:58:53.26
>>587
答えはかいてあるだろう
593132人目の素数さん:2012/09/14(金) 09:10:36.30
>>591
それは自明ではなさすぎるのでno q.e.d.です。
なにか上位の定理でもあるのでしょうか?
>>592
答えはありません。
>>590
これは大失態でした。申し訳ありません。
>>588
その通りでございます。
594592:2012/09/14(金) 20:24:03.06
>>593
ないのは証明

馬鹿乙
595132人目の素数さん:2012/09/14(金) 21:07:01.95
>>594
その通りですね。証明の模範解答のための略解とくに、
sup||x(n)||<∞の否定についての考察が欲しいです。
596132人目の素数さん:2012/09/14(金) 21:23:59.01
空間ベクトルで(ベクトル記号省略)

OR=(2/3)a+(1-k/3)c+kd
OR=(1-t)a+(1-s)c +(s+t)d

まぁ、数値や、条件はいいんですが
比較するときに
a,c,d(ベクトル記号省略)は同一平面上にないから...となり比較してますが、どういう意味があるのですか?
確かにa,c,dは同一平面上にありませんが...

597132人目の素数さん:2012/09/14(金) 21:32:57.54
>>596
a,c,dがどういう条件をみたせば対応する係数が等しい、といえるのかを考えてみよ。
598132人目の素数さん:2012/09/14(金) 21:47:42.42
>>597
ゼロベクトルでなくて平行でない?
599132人目の素数さん:2012/09/14(金) 21:54:05.48
ベクトルの一次独立性、というのは聞いたことがないか?
600132人目の素数さん:2012/09/14(金) 21:54:13.95
まちがい><
平面ならせいかい(・∀・)
601132人目の素数さん:2012/09/14(金) 21:59:44.79
>>599
ありますが、詳しくは分かりません。
>>600
ですよね
空間になるといまいち分かりません。
602132人目の素数さん:2012/09/14(金) 22:10:02.65
↑OA,↑OB,↑OC が同一平面上にあると
↑OC は ↑OA,↑OB を用いて表せるので
↑OP = s↑OA + t↑OB + u↑OC 以外にも
↑OA,↑OB,↑OC の係数を考えることができる
つまり複数の表現ができることになるので係数比較が意味を成さない

「ひと通りに決まりますよ」と保証しているのが1次独立性であり
このおかげで見かけが違っている表現で「実は等しいので」と式を立てることができる
603132人目の素数さん:2012/09/14(金) 22:12:41.37
>>602
なるほど。
ならこの場合
0ベクトルでなく平行ではないというのは意味ないですか?
604132人目の素数さん:2012/09/14(金) 22:21:48.17
意味ないです><
605132人目の素数さん:2012/09/14(金) 22:22:56.19
>>603
平面ベクトルの1次独立性は「0ベクトルでなく平行ではない」という表現もできるが
空間はこれでは表現できない
あえて表現すれば「3つのベクトルで4面体が決まる」ということだが
「1次独立」という用語を使うほうが簡潔だ
最近の教科書には出てないらしいけど使って怒られるころはあるまい
606132人目の素数さん:2012/09/14(金) 22:51:26.92
>>605
表現できないってどういうことですか?
607132人目の素数さん:2012/09/14(金) 22:59:24.10
正しい命題にはならない、ということ。
608132人目の素数さん:2012/09/14(金) 23:08:10.91
3次元ベクトルなら、質問元にもあるように「同一平面上にない」と表現されることも多いと思うけどな
609132人目の素数さん:2012/09/14(金) 23:13:05.66
その通りだよ。だれもそこは否定していない。
610132人目の素数さん:2012/09/14(金) 23:22:48.86
ふつうに
a,c,dベクトルが1次独立ではないのでと書いてもいいですか?
611132人目の素数さん:2012/09/14(金) 23:42:20.25
とか尋ねるということは、1次独立が魔除けの呪文とでも思っているということだな
612132人目の素数さん:2012/09/15(土) 06:46:33.55
いや、意味が分かりません。

いいんですか?
613132人目の素数さん:2012/09/15(土) 10:36:07.36
誰かお願いします

d/dx F(x,y)=0 の場合

F(x, -1) = 3, F(x, 1)= -3 が全てのxにおいて可能な解は存在するか
614132人目の素数さん:2012/09/15(土) 10:36:48.80
問題中に与えられている条件次第。
615132人目の素数さん:2012/09/15(土) 10:39:05.60
F(x,y)=-3y
616132人目の素数さん:2012/09/15(土) 10:47:33.95
>>615
xが存在しなくてもいいんですか
617ななし:2012/09/15(土) 12:51:57.65
3a2乗-3a-1=0の解説をお願いします
618132人目の素数さん:2012/09/15(土) 13:39:13.21
三次元ベクトル r
g(r)=1/ ||r|| の場合
△gは?
619132人目の素数さん:2012/09/15(土) 14:13:52.01
一般の5次方程式が代数的に解けないことは有名ですが
この事実から以下のことが成り立ちますか?
「4則演算と根号の組み合わせで表現できない数αと、有理数係数の1次以上の多項式f(x)について
αをうまく選んだとき、f(α)が有理数となるような、f(x)の次数の最小値は5である」
620132人目の素数さん:2012/09/15(土) 14:20:33.21
>>619
何を任意に与えて、何を適当に選ぼうとしているのか、はっきりさせてよ。
621132人目の素数さん:2012/09/15(土) 17:00:47.91
http://www.youtube.com/watch?v=Q4gTV4r0zRs&feature=player_embedded

これってどうやって計算するの?
20×20はいくつになるの?
622132人目の素数さん:2012/09/15(土) 17:36:35.20
総当たり
623132人目の素数さん:2012/09/15(土) 17:37:35.14
>>619
代数的に解けないことと、解が代数的数でないことは
同値ではないとおもうんだが
624132人目の素数さん:2012/09/15(土) 17:39:38.97
そこでいう「代数的」ってどういう意味ですか?
625132人目の素数さん:2012/09/15(土) 19:28:38.85
αをうまく選んだときは なんでもいえるから deg f = 5 は最小。 >>619



626132人目の素数さん:2012/09/15(土) 21:10:23.83
誰か>>618お願いします
627132人目の素数さん:2012/09/15(土) 21:24:12.59
g(・)って何?
Δの定義ってなんだっけ?昔のことで忘れてしまった。
定義を聞けば思い出して分るかも。
628132人目の素数さん:2012/09/15(土) 22:24:49.15
          __ノ)-'´ ̄ ̄`ー- 、_
        , '´  _. -‐'''"二ニニ=-`ヽ、
      /   /:::::; -‐''"        `ーノ
     /   /:::::/           \
     /    /::::::/          | | |  |
     |   |:::::/ /     |  | | | |  |
      |   |::/ / / |  | ||  | | ,ハ .| ,ハ|
      |   |/ / / /| ,ハノ| /|ノレ,ニ|ル' 
     |   |  | / / レ',二、レ′ ,ィイ|゙/   私は只の数ヲタなんかとは付き合わないわ。
.     |   \ ∠イ  ,イイ|    ,`-' |      頭が良くて数学が出来てかっこいい人。それが必要条件よ。
     |     l^,人|  ` `-'     ゝ  |        さらに Ann.of Math に論文書けば十分条件にもなるわよ。
      |      ` -'\       ー'  人          一番嫌いなのは論文数を増やすためにくだらない論文を書いて
    |        /(l     __/  ヽ、           良い論文の出版を遅らせるお馬鹿な人。
     |       (:::::`‐-、__  |::::`、     ヒニニヽ、         あなたの論文が Ann of Math に accept される確率は?
    |      / `‐-、::::::::::`‐-、::::\   /,ニニ、\            それとも最近は Inv. Math. の方が上かしら?
   |      |::::::::::::::::::|` -、:::::::,ヘ ̄|'、  ヒニ二、 \
.   |      /::::::::::::::::::|::::::::\/:::O`、::\   | '、   \
   |      /:::::::::::::::::::/:::::::::::::::::::::::::::::'、::::\ノ  ヽ、  |
  |      |:::::/:::::::::/:::::::::::::::::::::::::::::::::::'、',::::'、  /:\__/‐、
  |      |/:::::::::::/::::::::::::::::::::::::::::::::::O::| '、::| く::::::::::::: ̄|
   |     /_..-'´ ̄`ー-、:::::::::::::::::::::::::::::::::::|/:/`‐'::\;;;;;;;_|
   |    |/::::::::::::::::::::::\:::::::::::::::::::::::::::::|::/::::|::::/:::::::::::/
    |   /:::::::::::::::::::::::::::::::::|:::::::::::::::::::::O::|::|::::::|:::::::::::::::/
629132人目の素数さん:2012/09/15(土) 22:25:03.92
>>627
gはただの関数です。
△は▽・▽f
630132人目の素数さん:2012/09/15(土) 22:41:23.93
ラプラシアンの極座標表示を使うと楽なんじゃない?
631132人目の素数さん:2012/09/15(土) 22:42:46.35
携帯から失礼します
どなたか解き方教えてください

流れの速さが
毎時3qの川の上流には甲市
下流には乙市がある。
Aは船で乙市から
甲市へさかのぼり
Bは甲市から乙市へ下る。
静水時の速さを比較すると
Bの船はAの船の3倍であり
またそれぞれの所要時間を比較すると
AはBの5倍の時間を要するという。
Aの船の静水時の時速を求めよ。

よろしくお願いします
632132人目の素数さん:2012/09/15(土) 22:49:03.59
B+3=5(A-3)
B=3A
633132人目の素数さん:2012/09/15(土) 23:06:29.76
634132人目の素数さん:2012/09/15(土) 23:14:26.44
>>632
解けました!
ありがとうございました!
635132人目の素数さん:2012/09/15(土) 23:30:55.78
>>629
演算子の定義のままに計算するだけか。
r=(x,y,z)に対して g(r)=1/|r|=1/√(x^2+y^2+z^2)
定義に従えば Δg=(∂^2/∂x^2+∂^2/∂y^2+∂^2/∂z^2)g
(∂/∂x)g=(∂/∂x)(x^2+y^2+z^2)^(-1/2)
=(-1/2)(2x)(x^2+y^2+z^2)^(-3/2)=-x(x^2+y^2+z^2)^(-3/2)
これを更にxで偏微分すると
(∂^2/∂x^2)g=-(x^2+y^2+z^2)^(-3/2)+(-x)(-3/2)(2x)(x^2+y^2+z^2)^(-5/2)
=-(x^2+y^2+z^2)^(-3/2)+3(x^2)(x^2+y^2+z^2)^(-5/2)
これをy,zについても同様に計算すると
(∂^2/∂x^2+∂^2/∂y^2+∂^2/∂z^2)g
=-3(x^2+y^2+z^2)^(-3/2)+3(x^2+y^2+z^2)(x^2+y^2+z^2)^(-5/2)
=-3(x^2+y^2+z^2)^(-3/2)+3(x^2+y^2+z^2)^(-3/2)
=0

こんなんで合っているのか?
636132人目の素数さん:2012/09/15(土) 23:52:26.86
>>623
代数的数は必ず有理数の根号と四則演算で表現できるのですか?
637132人目の素数さん:2012/09/16(日) 01:58:23.32
できない
638132人目の素数さん:2012/09/16(日) 02:12:31.80
反例は?
639132人目の素数さん:2012/09/16(日) 02:17:58.43
じゃあ逆に、有理数の根と四則演算で書けるけど代数的数じゃない数はあるの?
640132人目の素数さん:2012/09/16(日) 03:10:32.69
>>635
チェイン・ルールは必要ないんですか?
641132人目の素数さん:2012/09/16(日) 04:05:55.41
運営乙
642132人目の素数さん:2012/09/16(日) 04:54:38.33
>>638
有理係数5次方程式X^5-2=0の1つの根2^{1/5}は、有理数の根号で表せないような代数的数の一例。

>>639
有理数の根号と四則演算で書ける数はすべて代数的数。
643132人目の素数さん:2012/09/16(日) 05:40:37.57
>>619
「」内がなにいいたいのか分からないから、こちらで手直しするが、
>一般の5次方程式が代数的に解けないこと
に関係なく
>任意の有理係数の1次以上の多項式f(x)に対して、4則演算と根号の組み合わせで表現できない数αをうまく選べば、
>f(α)が有理数となる、という条件を満たすような有理係数多項式f(x)の次数の最小値は5以下である
ことが正しいのは、既にエルミートが証明してる。
エルミートは、有理係数5次方程式に対する楕円関数を用いた解法を見つけた。
前者から後者が証明出来るのかどうかは知らない。また、「」内を
>或る4則演算と根号の組み合わせで表現できない数αが存在して、任意の有理数係数の1次以上の多項式f(x)について
>f(α)が有理数となる、という条件を満たすような有理係数多項式f(x)の次数の最小値は5以下である
と解釈するのはちょっとムリ。
644132人目の素数さん:2012/09/16(日) 06:15:46.26
>>619
任意の有理係数の1次以上の多項式f(x)に対して、4則演算と根号の組み合わせで表現できない数αをうまく選べば、
f(α)が有理数となる、という条件を満たすような有理係数多項式f(x)の次数の最小値は3だな。
有理係数多項式X^3-2の1つの根は2^{1/3}で、これは根号では表せない。
根号ってベキ根を含んでいないただの√のことだろ?
ベキ根も含んで考えるなら、上の条件を満たす有理係数多項式の次数の最小値は5になる。
645132人目の素数さん:2012/09/16(日) 07:17:52.45
重力ポテンシャルは鉛直ベクトルで渦がないから,rot・gradはrotで0になる。
宇宙工学のイロハです。
646132人目の素数さん:2012/09/16(日) 07:31:31.04
div・grad=laplace
重力ポテンシャルは湧き出し0だからdivで0になる。
647132人目の素数さん:2012/09/16(日) 07:49:19.51
(x-a)^5=0
648132人目の素数さん:2012/09/16(日) 08:24:14.91
四角形を自由に作ってください。

この四角形の辺の長さは縦横がaとbで、ともに整数です。
面積sは a * bです

また、面積sは

s = 16+45n 

を満たします(nは自然数)

あなたが作った四角形のたてよこ a,b の長さを答えてください。

面積は900以下で
なるべく正方形に近い形の四角形だとありがたいです。
それと nは14前後くらいの値だと嬉しいです。
649132人目の素数さん:2012/09/16(日) 08:27:56.61
n の値を1から順番に増やしていけばいいんだけど、
そのサイズの四角形の辺の長さがわからん。

面積が 331だったとして、
そのサイズの四角形を作ることのできる辺 a,b が何通りあるのかが分からん。

素数にバラしてあれこれするやり方が分からん。
650132人目の素数さん:2012/09/16(日) 08:35:35.83
n=19 1*871=871 13*67=871
n=18 1*826=826 2*413=826 7*118=826 14*59=826
n=17 1*781=781 11*71=781
n=16 1*736=736 2*368=736 4*184=736 8*92=736 16*46=736 23*32=736
n=15 1*691=691
n=14 1*646=646 2*323=646 17*38=646 19*34=646
n=13 1*601=601
n=12 1*556=556 2*278=556 4*139=556
n=11 1*511=511 7*73=511
n=10 1*466=466 2*233=466
n=9 1*421=421
n=8 1*376=376 2*188=376 4*94=376 8*47=376
n=7 1*331=331
n=6 1*286=286 2*143=286 11*26=286 13*22=286
n=5 1*241=241
n=4 1*196=196 2*98=196 4*49=196 7*28=196 14*14=196
n=3 1*151=151
n=2 1*106=106 2*53=106
n=1 1*61=61
n=0 1*16=16 2*8=16 4*4=16
651132人目の素数さん:2012/09/16(日) 08:36:22.44
a*b<=900 だから、そんなにたくさんないでしょ
出題形式といい(数学的に)真面目に解くより、プログラム組んだ方が早そう
652648:2012/09/16(日) 09:05:14.63
>>650
あざっす!!!!!

>>651
そんなプログラム組めないポ・・・
Excelで素因数分解とかよく分からんし・・・
653132人目の素数さん:2012/09/16(日) 09:45:56.40
>>640
難しいことは聞かないでくれ。
それは何のことだ?
654132人目の素数さん:2012/09/16(日) 10:56:05.06
ずいぶんレベルが下がるけどわからないので教えてください。

赤色、青色、黄色の3種類の色の玉がかごの中に入っています。
玉は全部で10個入っています。
赤色は青色よりも多いですが、黄色の2倍よりも少ないです。
次の推論の中で、この条件を必ず満たすことを言っている人は誰ですか。
全て選びなさい。

A君 青色は3個以下。
B君 赤色は5個以上。
C君 一番多い色は赤色。

正解はA君

と書かれているのだが、A君の場合でも
青色が1個、黄色が1〜2個、残り全部が赤色だった場合、
赤色が黄色の2倍より多くなるので必ず満たすとはいえないと思います。
よって私の答えは全員、必ず満たすとはいえないので「該当者無し」なのです。

問題または答えが間違ってるのか私の考えが間違ってるのか教えてください。
655132人目の素数さん:2012/09/16(日) 11:22:55.58
うっかりしてる
656132人目の素数さん:2012/09/16(日) 12:16:27.35
>>654
> 次の推論の中で、この条件を必ず満たすことを言っている人は誰ですか。
本当にそういう問題文なの?
問題文の条件の時に必ず言えることを言っている人は誰なのかを問う問題じゃないの?
657132人目の素数さん:2012/09/16(日) 12:34:16.13
          __ノ)-'´ ̄ ̄`ー- 、_
        , '´  _. -‐'''"二ニニ=-`ヽ、
      /   /:::::; -‐''"        `ーノ
     /   /:::::/           \
     /    /::::::/          | | |  |
     |   |:::::/ /     |  | | | |  |
      |   |::/ / / |  | ||  | | ,ハ .| ,ハ|
      |   |/ / / /| ,ハノ| /|ノレ,ニ|ル' 
     |   |  | / / レ',二、レ′ ,ィイ|゙/   私は只の数ヲタなんかとは付き合わないわ。
.     |   \ ∠イ  ,イイ|    ,`-' |      頭が良くて数学が出来てかっこいい人。それが必要条件よ。
     |     l^,人|  ` `-'     ゝ  |        さらに Ann.of Math に論文書けば十分条件にもなるわよ。
      |      ` -'\       ー'  人          一番嫌いなのは論文数を増やすためにくだらない論文を書いて
    |        /(l     __/  ヽ、           良い論文の出版を遅らせるお馬鹿な人。
     |       (:::::`‐-、__  |::::`、     ヒニニヽ、         あなたの論文が Ann of Math に accept される確率は?
    |      / `‐-、::::::::::`‐-、::::\   /,ニニ、\            それとも最近は Inv. Math. の方が上かしら?
   |      |::::::::::::::::::|` -、:::::::,ヘ ̄|'、  ヒニ二、 \
.   |      /::::::::::::::::::|::::::::\/:::O`、::\   | '、   \
   |      /:::::::::::::::::::/:::::::::::::::::::::::::::::'、::::\ノ  ヽ、  |
  |      |:::::/:::::::::/:::::::::::::::::::::::::::::::::::'、',::::'、  /:\__/‐、
  |      |/:::::::::::/::::::::::::::::::::::::::::::::::O::| '、::| く::::::::::::: ̄|
   |     /_..-'´ ̄`ー-、:::::::::::::::::::::::::::::::::::|/:/`‐'::\;;;;;;;_|
   |    |/::::::::::::::::::::::\:::::::::::::::::::::::::::::|::/::::|::::/:::::::::::/
    |   /:::::::::::::::::::::::::::::::::|:::::::::::::::::::::O::|::|::::::|:::::::::::::::/
658132人目の素数さん:2012/09/16(日) 12:58:22.65
d/dx F(x,y)=0 の場合
F(x,a)=b
a,bは自然数は存在するか?
また、F(a,y)=0 は存在するか?
659132人目の素数さん:2012/09/16(日) 16:44:00.51
>>642
> 2^{1/5}は、有理数の根号で表せないような代数的数の一例。 

書けているようだが? 
もしかして近頃は5乗根は根号ではないのか?
660132人目の素数さん:2012/09/16(日) 17:45:25.75
>>650
あざっす

N=14 19*34 で配置パズル、解けそうです

http://i.imgur.com/2zFgK.png
661132人目の素数さん:2012/09/16(日) 17:53:11.28
有理数係数の5次方程式が代数的に解けないとき、
その方程式の解は有理数の四則演算と根号で表現できない数なのですか?
それとも、有理数の四則演算と根号で表現できる数だけれども、それを導出する方法(公式)がないってことですか?
662654:2012/09/16(日) 17:56:52.37
>>656
原文から変更してるのは果物を色の玉に変えてるだけで問題文はそのまま、
あと、子供向け算数コーナーの物なので3人の条件の提示が口語調だったのを
直してるぐらいです。
>問題文の条件の時に必ず言えることを言っている人は誰なのかを問う問題
だとしたら問題文が説明不足ですね。
ありがとうございます。
663592:2012/09/16(日) 18:16:14.14
>>626
微分のことは自分でしなさい
664132人目の素数さん:2012/09/16(日) 18:17:33.60
学校の宿題きくやつ、ワロスw
大学生にもなって知能低すぎわろしゅw
665132人目の素数さん:2012/09/16(日) 18:26:01.13
二次関数y=x^2-5x+6のグラフをx軸方向にどれだけ平行移動すれば原点を通るようになるか。
666132人目の素数さん:2012/09/16(日) 18:26:34.15
>>825
高校生の質問板からのマルチいいですか?
朝から頑張ってるんですけど全然わかりません
667132人目の素数さん:2012/09/16(日) 18:28:21.24
安価はミスです
668132人目の素数さん:2012/09/16(日) 18:30:29.39
>>665
x軸方向にa平行移動した曲線の方程式をまず求める。
669132人目の素数さん:2012/09/16(日) 18:31:16.87
>>666
B={x-2≧0},C={x^2+ax+b<0}でC⊂Bになるa,bの条件は何か
670132人目の素数さん:2012/09/16(日) 20:38:14.69
二次不等式ax^2+2x+a>0がすべての実数xに対して成り立つように、定数aの値の範囲を定めよ。
671132人目の素数さん:2012/09/16(日) 20:49:26.24
(1+√2)^7+(1-√2)^7

解き方教えて下さい
(1±√2)^7と答えでは変形してるのですが、出来ないですよね?
672132人目の素数さん:2012/09/16(日) 21:09:47.59
>>671
上手い方法が思いつかないなら、地道に2項展開を実行する。
√2と-√2の奇数次項が打ち消しあうから、それほど大変な計算ではない。
673132人目の素数さん:2012/09/16(日) 21:11:38.79
>>671

 a_n = (1+√2)^n + (1-√2)^n,
これは初期値
 a_0 = a_1 = 2
と漸化式
 a_n = 2*a_(n-1) + a_(n-2),
を満たす。

答 478
674132人目の素数さん:2012/09/16(日) 21:27:16.03
ああだこうだ泥臭く一所懸命やってやっと下書きできたと思ったら
673が綺麗な求め方を出してたんで下書きがお蔵入り
675132人目の素数さん:2012/09/16(日) 21:44:00.79
          __ノ)-'´ ̄ ̄`ー- 、_
        , '´  _. -‐'''"二ニニ=-`ヽ、
      /   /:::::; -‐''"        `ーノ
     /   /:::::/           \
     /    /::::::/          | | |  |
     |   |:::::/ /     |  | | | |  |
      |   |::/ / / |  | ||  | | ,ハ .| ,ハ|
      |   |/ / / /| ,ハノ| /|ノレ,ニ|ル' 
     |   |  | / / レ',二、レ′ ,ィイ|゙/   私は只の数ヲタなんかとは付き合わないわ。
.     |   \ ∠イ  ,イイ|    ,`-' |      頭が良くて数学が出来てかっこいい人。それが必要条件よ。
     |     l^,人|  ` `-'     ゝ  |        さらに Ann.of Math に論文書けば十分条件にもなるわよ。
      |      ` -'\       ー'  人          一番嫌いなのは論文数を増やすためにくだらない論文を書いて
    |        /(l     __/  ヽ、           良い論文の出版を遅らせるお馬鹿な人。
     |       (:::::`‐-、__  |::::`、     ヒニニヽ、         あなたの論文が Ann of Math に accept される確率は?
    |      / `‐-、::::::::::`‐-、::::\   /,ニニ、\            それとも最近は Inv. Math. の方が上かしら?
   |      |::::::::::::::::::|` -、:::::::,ヘ ̄|'、  ヒニ二、 \
.   |      /::::::::::::::::::|::::::::\/:::O`、::\   | '、   \
   |      /:::::::::::::::::::/:::::::::::::::::::::::::::::'、::::\ノ  ヽ、  |
  |      |:::::/:::::::::/:::::::::::::::::::::::::::::::::::'、',::::'、  /:\__/‐、
  |      |/:::::::::::/::::::::::::::::::::::::::::::::::O::| '、::| く::::::::::::: ̄|
   |     /_..-'´ ̄`ー-、:::::::::::::::::::::::::::::::::::|/:/`‐'::\;;;;;;;_|
   |    |/::::::::::::::::::::::\:::::::::::::::::::::::::::::|::/::::|::::/:::::::::::/
    |   /:::::::::::::::::::::::::::::::::|:::::::::::::::::::::O::|::|::::::|:::::::::::::::/
676132人目の素数さん:2012/09/16(日) 22:05:16.26
>>650
あざっす!

N = 14の 19*34 で無事にレイアウトが仕上がりました。

http://images.wikia.com/anno2070/images/b/bb/Plot_110.png
677132人目の素数さん:2012/09/16(日) 22:14:11.77
>>671
7乗くらいなら
(1+√2)^7と(1-√2)^2を展開した時偶数番目の項を打ち消せるのに注目するのが1番手っ取り早いと思うよ

(1+√2)^7+(1-√2)^7
=2(7C0+7C2*(√2)^2+7C4*(√2)^4+7C6*(√2)^6)
=2(1+21*2+35*4+7*8)
=2(1+42+140+56)
=2*239
=478
678132人目の素数さん:2012/09/16(日) 22:31:58.41
>>654
青が4個以上と仮定する
青は最低4個
赤は青より多いから
赤は最低5個
黄の2倍は赤より多いから
黄は最低3個
4+5+3=12>10
合計10個なので矛盾
よって仮定は正しくない

この条件を満たす時必ずみたす推論だからといって
推論の条件をみたしたらこの条件を満たすとは限りません
679132人目の素数さん:2012/09/16(日) 22:43:43.26
>>678
そういう疑問じゃないだろ。
680673:2012/09/16(日) 23:40:03.14
>>671 の別解

 sinhβ = 1
とおくと、
 coshβ = √2,
 (1+√2)^n - (√2 -1)^n = 2sinh(nβ)
 = 2(sinhβ)U_n(coshβ) = 2U_n(√2),
と表せる。
ここに U_n(x) は第2種チェビシェフ多項式。

 U_7(x) = 64x^6 - 80x^4 +24x^2 -1,
 (与式) = 2U_7(√2) = 478

>>674
 申し訳ない。死んでお詫びを...(AA略)
681132人目の素数さん:2012/09/16(日) 23:42:51.89
>>680
別に謝る必要はないというか
俺も勉強になったので感謝してる
682132人目の素数さん:2012/09/17(月) 00:04:01.26
誰か>>658をお願いします
683132人目の素数さん:2012/09/17(月) 00:16:22.94
>>682
x,y は独立変数?
∂/∂x F(x,y)=0 でなくて?
何とも答えようというか考えようがない
684132人目の素数さん:2012/09/17(月) 00:31:29.33
>>679
上は余分だったな
ただ下の部分が>>654の疑問に対する返答のつもりなんだが
>>654が何歳かにもよるが高校生以上なら必要条件と十分条件について理解すればよいね
685132人目の素数さん:2012/09/17(月) 00:46:51.16
>>676 こういうパズルに隙間なく
いくつかのパーツを組み合わせていく問題って
解くためのアルゴリズムは見つかってるの?

それともやっぱり人間の直感とかで
やった方が早いの?
将棋も人間の直感の方がまだ強いよね。
686132人目の素数さん:2012/09/17(月) 00:50:19.26
>>683
∂/∂x F(x,y)=0 です
すいません
687132人目の素数さん:2012/09/17(月) 00:51:03.92
2Y>R>B
Y+R+B=10
3+4+3,<6
3+5+2
3+4+3
688654:2012/09/17(月) 01:16:03.29
>>678
先でも書いているとおり、問題文の説明不足でよいでしょう。
ありがとうございます。
689132人目の素数さん:2012/09/17(月) 01:38:10.67
>>684
だからそういう疑問じゃないだろ
690132人目の素数さん:2012/09/17(月) 01:40:52.87
>>685
総当りしていくアルゴリズムそのものはたいしたことない。
しかし総当りでない一般のアルゴリズムは未発見
というか、そのようなアルゴリズムは存在しないことが証明されていたような気もする。
691132人目の素数さん:2012/09/17(月) 04:44:10.72
命題: 空でない任意の集合 A が与えられた場合、func(A) ∈ A なる(選択)関数 func を選ぶ事が可能である。
これって明らかのように思えますが、もしかして選択公理が必要なんでしょうか?
要は添字集合の濃度が 1 の場合ってことです。
A={2,4,6,8,...} みたいに具体的に示された場合は、最小値を採るといった感じで示せますが、
捉えどころの無い未知の集合についても、全てそう主張できるのか言えるのでしょうか。
692132人目の素数さん:2012/09/17(月) 08:35:28.31
>>686
微係数が0とは?
693132人目の素数さん:2012/09/17(月) 10:01:02.23
数学
694132人目の素数さん:2012/09/17(月) 10:33:37.64
>>691
funcの定義域は何?
Aだけなら別に選択公理は不要
Aを含む集合族なら選択公理そのもの
695132人目の素数さん:2012/09/17(月) 11:28:24.45
a>0,b>0,a+b>0で(a+1/a)^2+(b+1/b)^2≧25/2であることを示せ
696132人目の素数さん:2012/09/17(月) 11:29:33.14
>>695間違えました
a>0,b>0,a+b=1で(a+1/a)^2+(b+1/b)^2≧25/2であることを示せ
697691:2012/09/17(月) 11:40:15.30
>>694
funcの定義域は集合Aだけを含む集合族となりますね。添字集合は文字「A」のみ。
具体的でない「捉えどころの無い集合」なんてのはそれこそ選択公理絡みでしか出現しないような予感がしますが、
そこから1個だけ要素を拾い上げるのも超越的な操作に思われ、これも選択公理が前提なのではないでしょうか?
698132人目の素数さん:2012/09/17(月) 16:04:30.40
>>697
選択公理で問題になる「捉えどころの無い集合」てのは添字集合の方。
それから、存在を示すのに具体的に構成する必要はない。
Aが空集合でないことから、Aの元xが存在し、
それに対応してf_x:A→xという写像が存在するので
条件を満たす写像が存在することが示せる。
ここでAの元xを具体的に選ぶ必要はない。
存在するけど具体的には示せないなんて代物も数学の論理では有り。
699132人目の素数さん:2012/09/17(月) 16:46:07.18
>>697
「集合Aは空でない」

「x∈A なる xが存在する」
は同値
具体的だろうが抽象的だろうが最低でも1個の要素の存在が示されているという事なので
それを使って func(A) = x とでも定義すればよい
700132人目の素数さん:2012/09/17(月) 17:29:40.30
¬ &forall x: A[x]
701132人目の素数さん:2012/09/17(月) 19:11:24.02
分問スレ369の145で質問した者です。その後調べてみると
どうやらデルタ関数や超関数というのが関係していることが分かりました
関数の一般性を研究する分野があるらしいです

ホモロジーやコホモロジーも関係していると思いますが難しくて分かりません。
以上、報告でした。
702132人目の素数さん:2012/09/17(月) 19:41:28.26
意味のない報告だなー
703132人目の素数さん:2012/09/17(月) 20:21:56.17
>>696

 Σ[i=1,n] (a_i + 1/a_i)^2 ≧ (1/n){Σ(a_i + 1/a_i)}^2  (コーシー)
   = n(A + 1/H)^2
   ≧ n(A + 1/A)^2,

ここに A = (1/n)Σ[i=1,n] a_i  相加平均
    H = n/Σ[j=1,n] (1/a_j)  調和平均
704132人目の素数さん:2012/09/17(月) 20:42:03.82
>>702
意味がなくはない
と言うよりどう考えても意味はあるので、意味がないと思うのなら
意味がないと思う理由を述べてください。
705132人目の素数さん:2012/09/17(月) 22:19:04.52
145 : 132人目の素数さん : 2012/05/12(土) 16:44:05.16
>>85
はい、そうです。できればまとめて勉強したいのでどの分野になるのか教えてくださると助かります。
自分が何をやっているのかがよく分かっていない状態なので。

たぶんそこで「"10"と表記されるn進数のnは、n進数で表せる整数の数と等しいのか?」と
少数の無限桁の末尾が定義できない事についてです。
「∞進数とした時に、整数として"10"と表記することは出来ない」となるのでしょうか。
一桁で示される整数が無限にあるとすれば、次の位にいつまでも上がらなくなります。

もしも上の桁に上がらないのだとすれば、下の桁に下がることも同様にないイメージがあります。
無限桁にする為にはn進数が有限であることが条件ですか。
それともn進数のnが無限で、桁も無限な構造はあるのでしょうか。
そこで例えば、無限桁の末尾の整数一桁を∞にしてしまった場合はどうなりますか。
706132人目の素数さん:2012/09/17(月) 22:19:54.96
>>88 >>83
その場合に0.aaa…を満たすXは無限にありますか。ありそうなのですが、あるとすると
Xを増やしていくと、0.aaa…は無限に小さくなります。

ここで有理数と無理数についてです。循環しないので無理数ではないです。
有理数の分数で表せる性質からすると、Xを満たす数だけ0.aaa…があることに
なってしまい、分数の表記が一つではなくなってしまいます。これはいわゆる
有理数と同じなのでしょうか。

1.000…=0.999… がイコールなのもx進数の一つがイコールで繋がっているだけだとすると、
1.000…と0.999…は、それぞれが別々の有理数の一つの表示です。

一つの表示に過ぎないとなると、集合論的な無限の濃さを調べる際の、
有理数の数を数えることは一体なにをやっている事になるのでしょうか。


極端な考えとして、aaaa.000…と表示した際に、各桁によってaの進数が違う数学の
体系はあるのでしょうか。あるとすればどの数学になりますか。ないとすると何故ないのでしょうか。
707132人目の素数さん:2012/09/17(月) 22:33:05.55
>>706
>1.000…=0.999… がイコールなのも
その…の意味および‥‥が含まれるとき=の意味(というか定義)を厳密に説明してもらえないか?そこがあいまいだからよくわからない結論がでてくるのでは?
708132人目の素数さん:2012/09/17(月) 22:49:12.73
1/9==0.111....でこれに*3すると1/3==0.333...になります。
左辺は3/9で有理数の約分の規則に従うだけ特に問題はありません。
右辺に注目すると0.111....が無限の桁でありこれに*3して0.333....となるようですが、
0.111.... * 3 == 0.333....が成立しない演算の体系は例えばどういうのがありますか?

それと、難しい議論になるので例では特に触れませんが、*を行うと小数の各桁が10進で収まらないときは、
桁上がりで収まらなかった数値が次の桁の+になりますが(例えば0.333 *4==1.2+0.12+0.012+....)、
この様に*を行っても桁上がりが成立しない、言い換えると左辺式 * (Π)と右辺式 + (Σ)が可換でない演算の体系は例えばどういうのがありますか?
709132人目の素数さん:2012/09/18(火) 00:33:18.44
>>708
>1/9==0.111....
とあっさり書くが、これどういう意味?
電卓の表示の結果から類推しているのですか?

無限に続く、というのが実際どういう意味なのか少し一緒に考えてみませんか?
710132人目の素数さん:2012/09/18(火) 00:38:28.52
>>708
我々が実数と呼んでいるものの演算体系(と無限級数の定義)を離れたところで
そこに書いてあるどこまでも続く数字の列を議論したいのなら、
どこか別のところにスレを立ててやってくれ。
711709:2012/09/18(火) 00:43:51.31
>>710
>どこまでも続く数字の列
そのようなものが実在するのか疑問だし、もっというと概念化すら怪しい、という気がします。
「無限」という言葉自体が有限であることにだまされているような気がします。
712132人目の素数さん:2012/09/18(火) 00:50:44.56
>>711
だから別のところでやってくれと言っている。
713709:2012/09/18(火) 00:58:07.92
終着点としてイプシロンデルタに誘導する意図、であってもですか?
714132人目の素数さん:2012/09/18(火) 01:03:11.38
realでもhyperrealでもsurrealでもp-adicでもいいから、専用スレ建ててそっちでやってよ
715132人目の素数さん:2012/09/18(火) 01:04:10.98
716132人目の素数さん:2012/09/18(火) 01:15:54.74
717132人目の素数さん:2012/09/18(火) 01:28:24.03
>>711
実は有限であるのにそれを「無限」と称して、みんながみんな、実は有限でしかありえないなもの(物体)やこと(概念)を、無限にある・無限である、と勘違いしてるってことですか?
718709:2012/09/18(火) 01:41:09.64
>>714
Intuitionistic infinity は入らないのね、こちらでは嫌われ者ですか‥
719132人目の素数さん:2012/09/18(火) 01:55:32.70
720132人目の素数さん:2012/09/18(火) 02:30:20.68
>>717
有限を対象とした手続きでなければ人間は実行できない、というのが数学の立場。
無限好きがよくやる1/3=0.333・・・に3を掛けて1=0.999・・・のごとき処理は
「・・・」の意味を明確にしない限り戯言でしかない。
721132人目の素数さん:2012/09/18(火) 02:40:49.20
無限好きだとか、そういうのではなくて、知識も根気もない人をあしらうための説明でしょう。
722709:2012/09/18(火) 03:49:49.02
>>720
あれ、それって数学の立場というよりは intuitionistic の立場のように聞こえる‥‥
723132人目の素数さん:2012/09/18(火) 03:58:56.47
だから別のところでや(ry
724132人目の素数さん:2012/09/18(火) 04:02:27.85
粘着はスルー
725132人目の素数さん:2012/09/18(火) 06:00:14.03
0.999・・・ってのは単に1/10を公比とする無限等比級数と考えられるから
高校2年生くらいの数学の知識で和が求められるはずだよね。
あと問題があるとすれば実数の表示として一対一対応をどう考えるか。
一般レベルの人にはやっぱり無理なのかななんて思ったり。
726709:2012/09/18(火) 06:16:23.29
>>725
一般の人ですが、普通に直線を点集合として捉えるのはさすがに困難をかんじますので、ここはデデキントの切断はどうでしょう?
727709:2012/09/18(火) 06:20:11.34
>>725
それが1に収束する、というきちんとした証明というか概念は高校生には困難かも。εδというよりは∀∃の処理が。
でも当時知りたかった概念ではあったのです。
728132人目の素数さん:2012/09/18(火) 06:22:46.98
だから別のところで(ry
729132人目の素数さん:2012/09/18(火) 07:35:51.41
>>703
相加相乗平均は習っているのですがΣの計算方法と調和平均は習ってません。すみません
730132人目の素数さん:2012/09/18(火) 08:34:21.83
習っていないからという表現は
自ら進んで調べる気なんて無い。常に受け身で行くぜ。
といった感じがして、あまりよろしくないね

>>703
1行目はコーシー(シュワルツ)の不等式じゃないよね?
イェンゼンの不等式(凸関数の性質)使ったのなら分かるけど
731132人目の素数さん:2012/09/18(火) 08:59:08.75
小数にうるさい人間がいるようだな
732132人目の素数さん:2012/09/18(火) 11:01:50.14
なんだ。幾何級数の事例で納得しちゃったのか。
大きいエネルギーを多少持ってそうだから桁上がり処理についての代数的概念まで踏み込んでみたけど、やっぱり難しかったか。
733132人目の素数さん:2012/09/18(火) 17:24:15.81
とりあえず分野だけは知っておいた方がいいかなと思った
内容は時間をかけて勉強していく
734132人目の素数さん:2012/09/18(火) 19:19:22.12
デデキントの切断とか出てくるってことはwikiで勉強してるんですか?
エプシロン・デルタ論法やデデキント実数切断の公理など解析を由来として理解することもできますが、あなたの思考方法を見ると解析より代数で理解した方があなたに向いてると思いますよ。
高木先生の解析概論の影響が今でも強いので、日本では伝統的にまずは解析による理解ありきです。
実数までなら解析で理解してもいいんですが、複素数以降さらに行列やベクトル空間で解析となると、例えば複素数なら複素数自体じゃなくて2変数・多変数を勉強することになり実質、高木実数を拡張したものでしか教えません。
例えば日本のほとんどの数学教本は、数の本質や数列の収束など基本的な数学概念について、解析系の方法(エプシロン・デルタとか)の説明でしか説明してません。
複素数体の元 z と2変数写像 f[x,y]がまったく同じ z==f[x,y] とみなすような演算の体系なら解析系でやっても疑問・質問は生じないんですけどね。
あなたは1/3 == 0.333... が解析で考えるとまったく同じと言われて違和感があったんでしょ?
代数か幾何ですかね、現代のコンピュータを当たり前の用に有効利用して、かつ、やってて楽しいのは。
解析は微分も含めて応用範囲も広いですけど、代数は数論・群論・暗号・情報を計算して、幾何はベクトル空間・物理数学・グラフ(トポロジー)・定積分とかの計量を評価して、解析とはまた違った味があります。
735132人目の素数さん:2012/09/18(火) 21:54:44.37
>>727
「限りなく」という形容詞句連用形を情緒的に解釈しなければ、
高校生でも収束の概念は誤解なく理解できると思うよ。
意味するところはε-δ論法の形をとっていないだけで同じことを言っているのだから。
736132人目の素数さん:2012/09/18(火) 22:03:44.52
∀∃の文章が理解できないと高校数学も覚束ないと思うんだけど
極限とか不等式とか関数の最大最小とか
737132人目の素数さん:2012/09/18(火) 22:48:47.10
高校生では、任意のとか、全てのとか、存在するとかそういう言葉を使えないと?
738709:2012/09/18(火) 23:11:01.34
∀∃

∃∀
くらいまでなら日本語による書き下しで処理できたかもしれませんが、
∀∃∀
になると、いったん言語化するステップを踏むのはなかなか大変。

>>734
>wikiで勉強してるんですか?
高木解析を中心に蛾のようにぐるぐる回りながら楽しんでいます、年寄り(といっても30代)の趣味として。本業はコンピュータ屋さん。
代数、といわれて思いつくのは教養課程の線形代数くらいしかないのですが、いろいろと広い世界が待っているのですね。
いろいろなキーワードを教えていただきありがとうございました。
739132人目の素数さん:2012/09/18(火) 23:28:55.56
得意なプログラミング言語は何かありますか?(gnurなど数式統計処理系も含む)
740132人目の素数さん:2012/09/18(火) 23:39:34.44
>>738
どうも高校生を舐めている様だ。
言い換えれば、高校生でなくても分らない奴は分かっていない、とでもいうか。
741132人目の素数さん:2012/09/18(火) 23:43:13.69
文系大学中退じゃないですか多分。
言ってること、書いてること(勉強してること)、が何だか中途半端なようですし。
742132人目の素数さん:2012/09/18(火) 23:58:51.71
高校じゃ習わない。習わないから受験にも出ない。
受験に出ないと予備校でも教わらない。

自分で受験高校数学用以外の勉強を別途してるようなやつじゃないと知らない。

教えれば理解できるかどうかとは全く別の話。
743132人目の素数さん:2012/09/19(水) 00:15:20.60
極限や収束、幾何級数や無限等比級数は、基礎の基礎なので高校教科書でもしっかりとした説明がありますよ。
744132人目の素数さん:2012/09/19(水) 01:18:33.82
>>696 >>729

 x^2 + y^2 = (1/2)(x+y)^2 + (1/2)(x-y)^2 ≧ (1/2)(x+y)^2,
を使って
 (左辺) ≧ (1/2)(a+b + 1/a + 1/b)^2
     ≧ (1/2){(a+b) + 4/(a+b)}^2   (相加・調和平均)
     = (1/2)(1 + 4)^2
     = 25/2,

〔相加・調和平均〕
 1/a + 1/b - 4/(a+b) = {(a+b)^2 -4ab}/{ab(a+b)} = (a-b)^2 /{ab(a+b)} ≧ 0,
745132人目の素数さん:2012/09/19(水) 01:32:04.05
>>740
うーん、多分高校生のころの自分が論理(学)的思考に乏しかった、と読み替えてください。

>>741
理系ですが‥‥
746132人目の素数さん:2012/09/19(水) 01:33:59.01
>>743
んー、イプシロンデルタなみの説明がないと、とてもとても。
自分の頃は困惑しつつ丸呑みしていたようです。
747132人目の素数さん:2012/09/19(水) 01:36:36.24
748132人目の素数さん:2012/09/19(水) 03:00:08.47
質問させてもらいます

中心がOである円周上に、二点ABをとる。
A、Bにおける接線の交点をPとし、
OPとABの交点をCとする。
また、円周上に点Dをとり、直線DCと円の交点をEとする。

角OPA=角OPBを示せ

こんな問題ですが、どうでしょう
749132人目の素数さん:2012/09/19(水) 03:54:31.39
DとEはどこに行った?
もっと長い問題の一部か?

証明には斜辺ともう1辺の等しい直角三角形の合同を使うのが簡単だろう。
750132人目の素数さん:2012/09/19(水) 13:09:21.89
>>744
丁寧にありがとうございました!
調和平均も調べながらやっていろいろと勉強になりました
751592:2012/09/19(水) 13:50:24.07
>>743
どこの高校?
752132人目の素数さん:2012/09/19(水) 14:46:30.88
解析学における実数 [編集]

実数の完備性により、実数に値を持つ関数で様々な近似操作を考えることができ、微積分が定義される。
特定のクラスの関数たちに対して距離の概念などを用いて位相を考えると位相線形空間が得られ,多くの場合に無限次元であるが、考えている位相に関して完備になっている。
位相空間上の関数やその積分の収束を考えるときは、問題にしている関数たちによって指定される位相空間の部分集合が重要になるが、こうして可測集合の概念,
例えば実閉区間 [0, 1] 上の関数を考えるときには一点集合 {t} (0 ? t ? 1) や開集合を含んで、補集合をとったり可算個の合併について閉じていたりするような集合族を考えることになる。
距離を持つコンパクト空間の可測集合のなす構造は、高々可算集合または閉区間 [0, 1] の構造に同型となることが知られている。

17世紀にニュートンとほぼ同時に微分の概念に到達したライプニッツは数の無限小変動(モナド)の考え方によって微分をとらえようとした。彼の考え方は十分に形式化されず、厳密性を欠いたものだった。
18〜19世紀にコーシー・ワイエルシュトラスらによりイプシロン-デルタ論法にもとづく微分の定式化が達成された。これにより数のコーシー列の「収束先」の存在を保証するものとして実数の体系がはっきりとした存在意義を持つようになった。
ライプニッツの無限小の概念はその曖昧さ故にε-δ法の陰に葬り去られていたが、1960年代に超準解析という枠組みのもとで厳密な定式化が達成された。
753132人目の素数さん:2012/09/19(水) 14:47:45.59
幾何学における実数 [編集]

ウリゾーンの補題から正規空間とよばれる広いクラスの位相空間の位相構造,つまりどの部分集合が開集合かはその上の実数値連続関数のなす空間に完全に反映されていることがわかる。
ユークリッド空間は有限次元の実ベクトル空間にその構造と両立するような距離をあたえたものとして定式化される。
実1次元ベクトル空間を平行移動したものが直線を示し、実2次元ベクトル空間を平行移動したものが平面を表していると見なせる。古典的なユークリッド幾何学は2次元や3次元のユークリッド空間とその構造を保つような変換についての研究だと解釈できる。
現代数学における図形の基本的な定式化の方法として多様体の概念が挙げられるが、これは局所的にはユークリッド空間のように見える「端切れ」を張り合わせたものとして定式化される。
したがって多様体の点は局所的にはいくつかの実数の組による座標付けを持ち、多様体上の実数値関数について微分や積分を考えることが可能になる。
多様体は連続的なものとして定義されるので、その連続的な「時間発展」、「変化」、あるいは「変形」を考えることができるが、これはしばしば加法群 R の微分同相による作用と考えることができる。
このような作用は力学系とよばれ、その類似として様々な分野でも R の作用が研究される。

代数学における実数 [編集]

実数の集合 R は体の構造を持っており、実数を係数とした多項式や実数の拡大体を考えることができる。ここで実数が極大順序体であることにより実数係数の多項式は2次以上なら既約にならない。
したがってRの有限次元拡大になっている可換体は R 自身と複素数体 C しかなく、可換性を外してもほかの有限次拡大体は四元数体 H しかない。
数論的に重要と見なされる位相群にQ のイデアル類群があるが、その単位元の連結成分は加法群 R と同型である。
754132人目の素数さん:2012/09/19(水) 14:52:04.27
馬鹿参上
755132人目の素数さん:2012/09/19(水) 15:03:38.51
バカ乙w
756132人目の素数さん:2012/09/19(水) 15:41:03.18
>>748です
>>749さんすいません、最後間違えました

中心がOである円周上に、二点ABをとる。
A、Bにおける接線の交点をPとし、
OPとABの交点をCとする。
また、円周上に点Dをとり、直線DCと円の交点をEとする。

角OPD=角OPEを示せ

これをだれかおねがいします
757132人目の素数さん:2012/09/19(水) 15:44:11.44
独学者用質問スレでも立てる?
758 忍法帖【Lv=16,xxxPT】(1+0:8) :2012/09/19(水) 15:47:49.91
a,b,x,yは0を含めず(a+b)(ax^2+by^2)と(ax+by)^2の大小を比べよ
この問題でニ式の差の正負で比べようとしていてab(x+y)^2になってそれ以上がわかりません
759 忍法帖【Lv=16,xxxPT】(2+0:8) :2012/09/19(水) 15:49:27.20
>>758
日本語がおかしかったかもしれません
0を含めずというのはa,b,x,y≠0という意味です
760132人目の素数さん:2012/09/19(水) 16:06:09.32
>>758
(a+b)(ax^2+by^2)-(ax+by)^2
=abx^2+aby^2-2abxy
=ab(x-y)^2
よって
「a,b>0またはa,b<0」かつx≠yの時(a+b)(ax^2+by^2)>(ax+by)^2
a=0またはb=0またはx=yの時(a+b)(ax^2+by^2)=(ax+by)^2
「a<0<bまたはb<0<a」かつx≠yの時(a+b)(ax^2+by^2)<(ax+by)^2
761132人目の素数さん:2012/09/19(水) 16:07:39.95
>>760
訂正
a=0またはb=0またはx=yの時
→x=yの時

a,b≠0やった
762 忍法帖【Lv=16,xxxPT】(3+0:8) :2012/09/19(水) 16:27:36.56
>>761
場合わけが必要なんですなありがとうございました
763132人目の素数さん:2012/09/19(水) 16:33:58.81
>>758
もっと具体的に書いて。
764132人目の素数さん:2012/09/19(水) 16:34:29.86
ありゃ、リロードしてなかった。
765132人目の素数さん:2012/09/19(水) 17:03:49.05
a,b,cは実数でt≧1を満たす任意の実数に対しての不等式
ax^2+bx+c≦√(t-1)≦bx+cで
c=1になることを示せ
766132人目の素数さん:2012/09/19(水) 17:04:48.88
すみませんt≧-1です
767132人目の素数さん:2012/09/19(水) 17:06:10.45
>>765>>766
全部間違えてました本当にすみません
768132人目の素数さん:2012/09/19(水) 17:08:34.75
a,b,cは実数でt≧-1を満たす任意の実数に対しての不等式
at^2+bt+c≦√(t-1)≦bt+cが成り立っている

c=1になることを示せ
769132人目の素数さん:2012/09/19(水) 17:14:07.53
t≧-1?
770132人目の素数さん:2012/09/19(水) 17:16:39.75
t≧-1だと√(t-1)のルートの中が負になる場合があるぞ
771132人目の素数さん:2012/09/19(水) 17:33:20.75
本当にごめんなさい。レス前によく確認します…√(t+1)でした
772132人目の素数さん:2012/09/19(水) 17:35:01.42
a,b,cは実数でt≧-1を満たす任意の実数に対しての不等式
at^2+bt+c≦√(t+1)≦bt+cが成り立っている

c=1になることを示せ

です。手のつけどころが分かりません
ヒントだけでも良いのでよろしくおねがいします
773132人目の素数さん:2012/09/19(水) 17:48:49.47
任意の実数っていう意味はわかる?
t≧-1の全ての実数tで成り立たないといけない
つまりt=0でも成り立たないといけないってこと
774132人目の素数さん:2012/09/19(水) 17:55:18.01
>>772

t は -1 以上の実数でその不等式が成立するんだぜ。
つまり、 tに 0 とか -1 とか好きな数字を放り込んでも
その不等式が成立するってことだ。

そして、c の値を求めたい訳だけど、
その不等式から cの値を求めるには a や b の存在が邪魔だよな?

a や b は t=0 の時にうまい具合に消えてくれるよな。

とりあえず、t = 0 を代入してみて、不等式をよくみつめろよ。
775132人目の素数さん:2012/09/19(水) 17:56:59.28
受験勉強の時の知識って結構おぼえてるもんだな。
もう30歳なのに。。。

ち、ちなみに神戸卒TOEIC700です… (; '‘ω‘`)
776132人目の素数さん:2012/09/19(水) 17:57:47.65
>>774
ありがとうございました
777132人目の素数さん:2012/09/19(水) 18:26:24.09
質問スレ
http://uni.2ch.net/test/read.cgi/math/1348042247/
709さんこちらで思う存分どうぞ
778132人目の素数さん:2012/09/19(水) 18:32:30.37
神戸卒と現役(私立)立教生を比べるとどちらの人間が(いろいろな意味で)王者なんですか?
779132人目の素数さん:2012/09/19(水) 18:35:50.97
>>778
意味がわからないけど神戸卒の今の職業によるだろjk
780132人目の素数さん:2012/09/19(水) 18:41:19.42
>>779
は?
俺は立教大学理学部数学科だぞ
なめるなよ
781132人目の素数さん:2012/09/19(水) 18:46:47.80
b=1/2を示せも考えているのですが
aを消そうと思ってもうまくいかないのでb=1/2になるように逆から考えていますがうまくいきません。
ご指摘お願いします
782132人目の素数さん:2012/09/19(水) 18:50:40.39
立教に嫉妬するなよおまえらwww

地方に住んでるやつらは現実がわかってないなwww

783132人目の素数さん:2012/09/19(水) 19:23:56.24
>>775
おっさんキモイ><
784A欄既卒 ◆iD93.8lby6 :2012/09/19(水) 20:00:13.31
t = 0を代入すると a と b の両方が一度に消えてしまうからな。
b が消えたら、話が前に進まないんだよな。

t = 0を代入した時に、
a だけがうまい事、その場に残るようにしたい。

両辺を t で微分してみたらどうだろう。

そうすると、 a と b のt に関する次数が1つずれるから、
b が t の値の影響をうけなくなるよね。

この状態で t=0 を代入してa を消すと。
785132人目の素数さん:2012/09/19(水) 20:06:39.27
>>784
すみません一年なんで微分わからないんで他のやり方ないですかねorz
786A欄既卒 ◆iD93.8lby6 :2012/09/19(水) 20:10:21.36
>>783
なんだよ、やるってぇのか
てめぇ… (; '‘ω‘`)

>>785
俺の半分くらいしか生きてなくてワロタwww
ソ連って知ってる?


ところ、不等式の両辺を微分するのって
数学的にアリだっけ?
符号の向きとか、別に変わらない・・・よね?
787132人目の素数さん:2012/09/19(水) 20:12:56.87
>>781
y=√(t+1)のグラフ(横軸t)を書いてみよう
それに
y=bt+1の直線(つまりy切片1の直線)をイメージする
すると(0,1)で共有点をもつ
t≧-1で常に√(t+1)≦bt+1が成り立たないといけないから
y=bt+1はy=√(t+1)の(0,1)における接線でないといけない
あとは微分したらその傾きは1/2とわかる
つまりb=1/2
788132人目の素数さん:2012/09/19(水) 20:19:38.67
>>786
微分しても不等号変わらないとは限らない

>>785
微分だめなのか
789132人目の素数さん:2012/09/19(水) 20:21:05.40
微分って接線の傾き出す奴のですよね
ちょっと考えてみます
790132人目の素数さん:2012/09/19(水) 20:25:53.27
√(t+1)≦bt+1に注目してどちらも正なので両辺の二乗の差を考える
(bt+1)^2-(t+1)=b^2t^2+(2b-1)t=t(tb^2+2b-1)≧0
これがt≧-1の全て(任意)の実数で成り立たないといけない
つまり-1≦t<0ではtb^2+2b-1≦0
0<tではtb^2+2b-1≧0である必要がある
tb^2+2b-1の連続性を考えるとt=0のときtb^2+2b-1=0になる必要がある
よって2b-1=0⇔b=1/2
791132人目の素数さん:2012/09/19(水) 20:26:16.61
>>784
おっさん乙
792132人目の素数さん:2012/09/19(水) 20:34:56.55
連立方程式なんだが、これ解けないよね?
x/2+y=x-y=2x-5y

中2レベルの知識でといて欲しいんだけど
特に条件はついてない
793132人目の素数さん:2012/09/19(水) 20:39:38.19
x=4yをみたす全ての実数
中2レベルなら解不定
794132人目の素数さん:2012/09/19(水) 20:59:30.38
>>790
あ、t の一次式の部分だけで
解けるんだ・・・
左の at2 とか要らんかったんや。
795A欄既卒 ◆iD93.8lby6 :2012/09/19(水) 21:11:23.69
>>790
あのさあ、
これって難しくね?
旧帝大の2次試験レベルじゃね?
796132人目の素数さん:2012/09/19(水) 21:20:20.89
>>790
>つまり-1≦t<0ではtb^2+2b-1≦0
>0<tではtb^2+2b-1≧0である必要がある
>tb^2+2b-1の連続性を考えるとt=0のときtb^2+2b-1=0になる必要がある
ここらへんがよくわからないです。すみません
797132人目の素数さん:2012/09/19(水) 21:28:38.15
t(tb^2+2b-1)≧0なら-1≦t<0で(マイナス)*(マイナス)になるはずだから(tb^2+2b-1)<0ってことですよね
で、そのあとのt=0のとき(tb^2+2b-1)が0になるのか分かりません
連続性っていうのがよくわかってないかもです
798132人目の素数さん:2012/09/19(水) 21:34:08.95
>>796
>つまり-1≦t<0ではtb^2+2b-1≦0
>0<tではtb^2+2b-1≧0である必要がある
ここはわかる?
正になるには正×正か負×負ってこと

>tb^2+2b-1の連続性を考えるとt=0のときtb^2+2b-1=0になる必要がある
多分ここでよくわからなくなったと思うんだけど
上の二行をみるとtが負から正になると(つまり0をまたぐとき)tb^2+2b-1も負から正になると分かったよね?
tb^2+2b-1ってことはt=0の時tb^2+2b-1=0にならないといけないということ
tb^2+2b-1が-2→-1→?→1→2ってなっていくのに"?"が急に100とかになるわけにはいかないからね


799132人目の素数さん:2012/09/19(水) 21:45:40.70
>>798
丁寧にありがとうございますスッキリしました。
あとaの範囲の問題がありますが最後ぐらいは自力でなんとかしようとおもいます
800132人目の素数さん:2012/09/19(水) 21:45:49.11
解けない問題投下

合同な円を二つ用意しぴったり重ねます
その円の半径をRとしたとき、

(1)一つの円の中心をR動かしたときの三日月形の面積
(2)一つの円の中心をR/2動かしたときの三日月形の面積
(3)一つの円の中心がT秒でR動くとしたときのN秒後(0<=N<=T)の三日月形の面積

を求めよ
801132人目の素数さん:2012/09/19(水) 22:09:07.03
>>795
王者の立教なめてるの?
返答しだいではまじでいくが




802132人目の素数さん:2012/09/19(水) 22:09:27.94
>>800
円の重なった部分の弧の両端とその円の中心とを結んでできる扇形の面積
そして弧の両端と二つの中心を結んでできるひし形の面積
これを求めればいいかと

(三日月の面積)=(図形全体の面積)={(二つの円の面積)ー(重なった部分の面積)}×1/2
(重なった部分の面積)=(扇がたの面積)×2−(ひし形の面積)

細かい解答は書くのめんどくさいんでこれで理解できたらいいな←
803132人目の素数さん:2012/09/19(水) 22:11:15.15
扇の中心角は、二つの円の中心間の距離と半径から
なんとか出してくれw
804132人目の素数さん:2012/09/19(水) 22:13:07.85
f(x)=∫[2x,x^2]e^t * cost dt
上の関数をxについて微分せよ。
g(t)=e^t * cost とおいて、その原始関数をG(t)とおく。と始めていいですか?

別にg,Gとする必要なく、f,Fでも問題ないですかね?
805132人目の素数さん:2012/09/19(水) 22:38:34.89
f(x)=〜でつないでいるのにまたfをつかうのはよくない
806132人目の素数さん:2012/09/19(水) 22:38:53.01
>>803
三角形の3辺から角を出すのは余弦定理
807132人目の素数さん:2012/09/19(水) 22:39:53.84
>>800 >>802 >>803
別スレッドで答えたように,問題 (2),(3)の結論は,本質的に逆三角関数を用いなければ表現できない.
(3) の答は
R^2{π-2θ+N/T×√(1-1/4×(N/T)^2)}.
ここで,θは
cos(θ)=1/2×(N/T), 0<θ<π/2
を満たす実数.
808132人目の素数さん:2012/09/19(水) 22:59:02.73
>>805
ならば、最初のgを使ったやつで解答を進めていっても大丈夫ですか?
809132人目の素数さん:2012/09/19(水) 23:02:43.56
>>808
大丈夫
810132人目の素数さん:2012/09/19(水) 23:06:58.49
a,b,cは実数でt≧-1を満たす任意の実数に対しての不等式
at^2+bt+c≦√(t+1)≦bt+cが成り立っている

c=1,b=1/2でaの範囲を求めよ
各辺からbt+c=1/2t+1を引いてat^2≦0よりa≦0がわかりました
また、-1≦t<0ではtを大きくしていくと中辺は小さくなりaは大きくなりました
0≦tでは中辺は大きくなりaは小さくなりました
これよりaが最も大きくなるのはt=0の時だと思うのですがどうでしょう
811132人目の素数さん:2012/09/20(木) 00:46:51.11
812132人目の素数さん:2012/09/20(木) 02:17:56.54
>>811
使ってないトイレットペーパー部分を横から見たときの面積をa
10日間で使った部分の面積をb
トイレットペーパーの厚さをx
トイレットペーパーを使い始めてから使い終わるまでの日数をyとすると

使ったトイレットペーパーを長方形とみると
b=660x
円の面積とみると
b=(6^2-5^2)π

連立して解くとx=π/60

トイレットペーパーは毎日同じ面積ずつ減っていくと考えて
使ってないトイレットペーパーの面積は10日で使う面積のa/b倍だから
y=10*a/b
a=(6^2-2^2)π
より29<y<30
よって使い始めて30日目で交換する
交換日は9月30日
813132人目の素数さん:2012/09/20(木) 02:27:27.61
.>>812
断面積で考えて
1日分=(6~2-5~2)/10*π=11π/10[cm^2]
全体=(6~2-2~2)*π=32π[cm^2]
なので、32π/(11π/10)=320/11=29.09...日分、交換は9月30日
紙の厚さは、(11π/10)/660=0.0053...[cm]≒0.053[mm]
ちなみに、全長は320/11*660[cm]=192[m]
数値が現実離れしているような
814132人目の素数さん:2012/09/20(木) 02:42:14.54
>>813
10日6.6mで約30日192mか
ノロウイルスのしわざだな
815132人目の素数さん:2012/09/20(木) 02:43:30.01
>>812
>>813
助かりました。
ありがとうございます。
816132人目の素数さん:2012/09/20(木) 12:03:50.65
未解決問題を使って証明する定理で
フェルマー予想の変わりに使われ出した
問題って何がありますか?
817132人目の素数さん:2012/09/20(木) 14:00:15.79
問題
818132人目の素数さん:2012/09/20(木) 14:14:45.75
リーマン予想が正しいと仮定するととかそういうやつか?
819132人目の素数さん:2012/09/20(木) 15:04:35.24
xについての2次式f(x)と3次式g(x)が以下をみたしている
(i)f(x),g(x)共に最高次の係数は1である
(ii)g(x)をf(x)で割った余りは -x+3である
(iii)f(3) = 0,g(0) = -3, g(1) = 2
f(x),g(x)を決定せよ

(i)より係数をa,b,c,d,e,と置くと
f(x) = x^2+ax+b, g(x) = x^3+cx^2+dx+e
(ii)の商をh(x)=Ax+Bと置くと
g(x) = f(x)h(x)-x+3 ---@
(iii)よりf(3)で3a+b=-9 ,g(0)でe=-3,g(1)でc+d=4
最高次がどちらも1よりA=1
@でx=3で3c+d=-8,x=0でbB=-6,x=1で4(1+a+b)(1+B)
これ等からc,dで連立方程式を立てc=-6,d=10まで導きました。ここからa,bをどうやって出すのか分かりません
820132人目の素数さん:2012/09/20(木) 15:20:40.04
c+dの値が違う
821132人目の素数さん:2012/09/20(木) 15:24:09.21
すみません…あってる値でどうa,bをだすのか教えてください
822132人目の素数さん:2012/09/20(木) 15:30:12.32
>>819
>@でx=3で3c+d=-8,x=0でbB=-6,x=1で4(1+a+b)(1+B)
ここ4=(1+a+b)(1+B)でした
823132人目の素数さん:2012/09/20(木) 15:35:40.33
c+dはあってるだろ
824132人目の素数さん:2012/09/20(木) 15:36:20.72
g(x)=x^3-5x^2+7x-3=(x-1)^2(x-3)=(x-1)(x^2-4x+3)
825132人目の素数さん:2012/09/20(木) 15:42:11.29
4=(1+a+b)(1+B)の文字を
bB=-6と3a+b=-9を使って全てbに揃えてうんたらかんたら
826132人目の素数さん:2012/09/20(木) 15:43:40.48
>>820 >>824
問題見間違えてた
吊ってくる
827132人目の素数さん:2012/09/20(木) 15:47:16.46
答えは二通り
f(x)=x^2-4x+3か
f(x)=x^2-5x+6
828132人目の素数さん:2012/09/20(木) 16:13:15.51
できました有難うございました。
829132人目の素数さん:2012/09/20(木) 17:24:22.53
>>772 >>781 >>799
t≧-1 において √(t+1)≦bt+1 …@
が成り立つときのbの値を求める自然な方法は〈bを分離〉することです.

bが何であっても,t=0のとき@は成り立つことに注意し,まず@を
「t>0 において b≧(√(t+1)-1)/t」かつ
「-1≦t<0 において b≦(√(t+1)-1)/t」
と分解する.不等式の右辺を,分子を有理化して書き換えると
「t>0 において b≧1/(√(t+1)+1)…A」かつ
「-1≦t<0 において b≦1/(√(t+1)+1)…B」
となる.1/(√(t+1)+1) は t≧-1 で連続な減少関数なので,
Aを満たすbの範囲は b≧1/2, Bを満たすbの範囲は b≦1/2.
したがって,@を満たすbの値は b=1/2.

残された問題
t≧-1 において at^2+(1/2)t+1≦√(t+1)
が成り立つときのaの範囲を求めるときにも〈aを分離〉すれば,気持ちよく解決できます.
830132人目の素数さん:2012/09/20(木) 18:24:00.60
>>829
>bが何であっても,t=0のとき@は成り立つことに注意し,まず@を
>「t>0 において b≧(√(t+1)-1)/t」かつ
>「-1≦t<0 において b≦(√(t+1)-1)/t」
ここでbのあとの不等号の向きがなぜ違うのか分かりません。すみません
831829:2012/09/20(木) 19:25:23.52
>>830
>ここでbのあとの不等号の向きがなぜ違うのか
不等式の両辺に 1/t を掛けるとき,t<0の場合は不等号の向きが変わるでしょう?
だから,「t>0」と「-1≦t<0」の2つの場合に分けたのです.
ちなみに,aの範囲を求めるときには,1/(t^2) を掛けることになるので,
場合分けが不要になります.
832132人目の素数さん:2012/09/20(木) 19:30:28.69
>>831
うっかりしてました。ありがとうございます
833132人目の素数さん:2012/09/20(木) 19:59:30.81
aの方、微分を使わずに手際よく求められるかな
834132人目の素数さん:2012/09/20(木) 20:20:48.22
手際よくは無理そうだな
835829:2012/09/20(木) 20:22:07.62
>>833 >>834
>aの方、微分を使わずに手際よく求められるかな
求められます.aに課せられた条件を
t≧-1 において a≦(-1/4)/(√(t+1)+t/2+1)
と言い換えると,右辺は t≧-1 で連続な増加関数になることが,
分母の形から微分法を用いずに分かるので.
836132人目の素数さん:2012/09/20(木) 20:31:06.19
>>835
分かりました。増加関数なのでt=-1のときがa=(-1/4)/(√(t+1)+t/2-1)なんですね
837132人目の素数さん:2012/09/20(木) 20:34:17.58
また式を写し間違えてる
838132人目の素数さん:2012/09/20(木) 20:38:37.68
ごめんなさい。自分の計算式が間違えてました
839132人目の素数さん:2012/09/20(木) 20:41:04.22
有難うございました。aはマイナスになるはずなのに…って思ってたときに指摘いただいたのですぐに計算間違えを解決できました
840132人目の素数さん:2012/09/20(木) 20:43:12.79
at^2≦√(t+1)-t/2-1を
a≦(-1/4)/(√(t+1)+t/2+1)と変換するのは閃きいるよな?
有理化の逆をしたのはわかるがt^2の項以外が消えたのはたまたまだし(係数によるから)
841132人目の素数さん:2012/09/20(木) 20:52:25.64
at^2+bt+c≦√(dt+e)≦bt+cについて同様の問題を考えたとき、
これが初等的に解けるためのd,eの条件を求めよ、だな。
842132人目の素数さん:2012/09/20(木) 20:59:01.27
B={x-2≧0},C={x^2+ax+b<0}でC⊂Bになるa,bの条件
843829:2012/09/20(木) 21:06:04.43
>>840
>有理化の逆をしたのはわかるがt^2の項以外が消えたのはたまたまだし(係数によるから)
√(t+1)-(1+t/2)=(t^2)×(t=0 で連続な関数) と書き換えられることには,
「1+t/2 が〈t=0における √(t+1)の1次関数〉である」という意味が(一応)あります.
844829:2012/09/20(木) 21:09:07.41
>>843
>「1+t/2 が〈t=0における √(t+1)の1次関数〉である」という意味が(一応)あります.
訂正です.
「1+t/2 が〈t=0における √(t+1)の近似1次関数〉である」という意味が(一応)あります.
845132人目の素数さん:2012/09/20(木) 21:09:12.04
>>841
とくに妙な条件なし。d,e>0のもとで>>829と同じようにやると、解ける。
c=√e、b=d/(2√e)、a≦-d^2/(2e√e)
846132人目の素数さん:2012/09/21(金) 00:28:38.32
複素数αと、それと共役な複素数βがある(ただしどちらも実数ではない)
複素数平面上の円を考える
それが1、−1、αを通るなら、この円は‐1/βも通ることを示せ

いろんな解法が欲しいです
よろしくお願いします
847132人目の素数さん:2012/09/21(金) 01:03:45.07
|-z|=1
848132人目の素数さん:2012/09/21(金) 01:14:42.59
やる気の起こった順に
・(-1,1)弦で円周角の定理
・一次分数変換して一直線上
・(x,y)で力技
849132人目の素数さん:2012/09/21(金) 01:44:34.16
z*でzの共役複素数を示すこととする

円の中心点の座標をpとおく
ただし円は1、-1を通るので(略)p*=-p

円の方程式は|z-p|^2=(z-p){(z-p)*}=r^2
1とαを通るので(α-p)(α*+p)=(1-p)(1-p*)
整理してp=(αα*-1)/(α*-α)

(-1/α*)*=-α*/(αα*)=-1/α

|β-p|^2
=|1/α*-p|^2
=(-1/α*-p)(-1/α-p*)
=1/(αα*)+p(1/α-1/α*)-p^2

=1/(αα*)+{(αα*-1)/(α*-α)}{(α*-α)/(αα*)}-p^2
=1/(αα*)+{(αα*-1)/(α*-α)}{(α*-α)/(αα*)}-p^2
=(αα*-1+1)/(αα*)-p^2

=1-p^2
=(1-p)(1-p*)

でいえてるのかどうかはしらん
850132人目の素数さん:2012/09/21(金) 10:12:38.46
>>846
αの絶対値をr, 偏角をθとおくと,-1/βの絶対値は 1/r, 偏角はθ+π となる.
α, -1/β, 1, -1, 0 が表す点をそれぞれ,A, B, P, Q, O とすると,
OP:OA=OB:OQ, ∠POA=∠BOQ により △OPA ∽ △OBQ である.だから,
∠PAO=∠BQO, 即ち ∠PAB=∠PQB が成り立ち,4点A, B, P, Q は同一円周上にある.

(問題の本質は,上の解答例の1行目に凝縮されています.)
851132人目の素数さん:2012/09/21(金) 10:23:32.55
MがコンパクトでないときC^r位相とW^r位相の一致しないことの具体例がわかりません
ホイットニィ位相についてです
おねがいします
852846:2012/09/21(金) 12:22:47.62
みなさん解答ありがとうございます
個人的には>>850が一番気に入りました
853132人目の素数さん:2012/09/21(金) 13:15:04.51
>>846
>>850と本質的には同じだけど
>>850と同じ記号で、線分ABと線分PQはOで交わる。(P,Qの偏角より)
OA・OB=OP・OQだから、方べきの定理の逆よりA,B,P,Qは同一円周上にある
854132人目の素数さん:2012/09/21(金) 13:18:33.20
| -1/r cis[-b] |^2 == 1
855132人目の素数さん:2012/09/21(金) 14:10:42.49
>>751
教科書1ページぐらい使ってしっかり説明してあるようですが、高校生(16-18才の知能)で収束と極限をはっきりと区別できる人間は滅多にいない(旧帝現役合格余裕レベル)です。
856132人目の素数さん:2012/09/21(金) 14:26:22.11
収束と極限の区別?
問題にするところがおかしくないか
857132人目の素数さん:2012/09/21(金) 14:49:33.18
収束と極限を正確に理解してはっきりと区別できる人間、と書いた方が分かりやすかったですか。
普通の順番で教えるなら、数列や級数、その先に平均変化率と微分係数と続くので、理解することが難しいことをかなり駆け足でやるのでほとんどの人間は理解もできなければ説明されたことも頭に残ってないと思いますよ。
忘れたことろに、0.999...スレとか&epsiron;-δ論法だけで突き進む人間とか出てくるし、このあたりが勘違いにつながってるんじゃないですか。
858132人目の素数さん:2012/09/21(金) 15:12:59.35
>>855
高校では極限の定義をしてない。
独学で高木でも読めば別だが。

知能は関係ない。
859132人目の素数さん:2012/09/21(金) 15:32:00.33
>>858
正確に理解してると「極限』とか「値』とかの言い回しが気になっちゃう(こだわっちゃう)んですよね。
理解するにはそれ相当の努力が必要ですし、獲物を狙うような鋭い目つきも必要ですけど、議論収束予定半径内ならそれを許容する度量も必要かなと思います。
860132人目の素数さん:2012/09/21(金) 16:00:05.64
>>857
いや別に誤解はしてないつもりだよ
それら概念の区別が高校生にとって難しいものだとも思わないし、仮に呼称を間違えたとしても支障ないはず
放射能と放射性物質をまとめて「放射能」と呼ぶようなもの
861132人目の素数さん:2012/09/21(金) 16:04:42.03
論文をよみたいです。
無料で読めるサイト教えてください。
862132人目の素数さん:2012/09/21(金) 16:10:41.26
「知能」というから理解できないんですか?
このあたりが、あなたの「こだわり」ですかね。

微妙な言い回しを使っても使わなくても17才前後で理解できないというわけではなく、17年程度の(普通の)イクスペアリアンスでは何をどうあがいても理解は無理(つまり理解できない)ってことです。
この一文ですら論理的に何を言ってるのか、把握は出来ても理解するのは難しいですよね?
こういうのを「知能』とここでは呼んでます。
863132人目の素数さん:2012/09/21(金) 16:14:41.18
あんた、収束と極限を区別するまでによっぽど苦労したんだな…w
864851:2012/09/21(金) 16:15:57.69
自己解決です。引っかかった人のために
W^r位相の方が、C^r位相より強いのは定義からいえます。
しかし、コンパクトだとすれば、W^r自体も、C^r位相に
入るので、もちろん同じになります。
865132人目の素数さん:2012/09/21(金) 16:28:37.86
>>862
そんな雑なレトリック(虚仮威し)で大丈夫か?
866132人目の素数さん:2012/09/21(金) 16:59:43.88
867132人目の素数さん:2012/09/21(金) 17:17:03.20
>>864
解決したんなら具体例も書いてくれ
有界でない関数でも集めるんか?
定義をググって考えようとしたら頭がごちゃごちゃになって諦めた
868132人目の素数さん:2012/09/21(金) 17:22:10.92
ε-δ論法など高校生でも読めば分かる事だ
なにを大げさに考えてるやら
869132人目の素数さん:2012/09/21(金) 17:42:22.92
>>860
bakaotu
870132人目の素数さん:2012/09/21(金) 17:58:31.63
limit A[k]=>πであるA:{A[1], A[2], A[3],....}についてAが増加数列で上に有界ならばA[k] (k->inf)は常にある値に収束する。ここでπをAの...
871132人目の素数さん:2012/09/21(金) 18:59:34.52
馬鹿乙はまだいたのか馬鹿乙真似
872132人目の素数さん:2012/09/21(金) 20:53:48.64
>>846
清書
>>849さんに倣って * で複素共役を表し、α、1、-1を通る円の中心を p、半径を r とする。
|1-p|=|-1-p|=r から p+p*=0、1+pp*=r^2。
|α-p|=r から r^2=(α-p)(α*-p*)=αα*-pα*-αp*+pp*=αα*-pα*-αp*-1+r^2
これより 0=αα*-pα*-αp*-1 両辺をαα*で除して
0=1-p/α-p*/α*-1/(αα*) すなわち 1=1/(αα*)+p/α+p*/α*。
|-1/α*-p|^2=(-1/α*-p)(-1/α-p*)=1/(αα*)+p/α+p*/α*+pp*=1+pp*=r^2 。
873132人目の素数さん:2012/09/21(金) 21:32:55.03
放物線y=ax^2+bx+cをy=ax+bに対称移動した式を求めよ。
高1にわかる説明でお願いします。
874132人目の素数さん:2012/09/21(金) 21:34:11.33
>>873
対称移動→対称に移動
875132人目の素数さん:2012/09/21(金) 22:05:26.51
>>873
直線y=ax+bで折り返したものの方程式を求めよ、のほうが分りよくないかい?

やることは機械的。
点(x,y)の直線y=ax+bに関する対称点を(u,v)とすると
(y-v)/(x-u)=-1/a  (直交条件) ・・・(1)
中点((x+u)/2,(y+v)/2)はy=ax+b上にあるので
(y+v)/2=a(x+u)/2+b ・・・(2)

(1)、(2)を連立してx,yをu,vで表す。
そのx,y を y=ax^2+bx+c に代入すると求めるu,vの式が得られる。
最後に、uをxに、vをyに書き換えて終了。
876851:2012/09/21(金) 23:04:53.64
>>867
たとえばRからRへのC^∞写像のW^rの開集合の一つは
コンパクトでないR全体の各点のジェットが開集合に
含まれていなければなりません
これはC^rでは有界閉区間はR全体になり得ないから
その領域の外では任意のジェットで良いわけです
つまり上でとったようなW^rの開集合はC^rの開集合に
ならないです。開集合の無限個の共通部分は位相の定義にそぐわず
必ずしも開集合にならないためです
877132人目の素数さん:2012/09/21(金) 23:50:42.38
開集合の無限個の共通部分が必ずしも開集合にならないのは分かるんだが
実際に開集合にならない事の証明は?
あぁ、やっと思考の焦点があってきた
要するに無限区間が有限個のコンパクト集合の和で表わせないだけのことか
サンキュー
878132人目の素数さん:2012/09/22(土) 00:40:21.16

http://images3.wikia.nocookie.net/__cb20120920231618/anno2070/images/6/66/Plot310.ad.png

こういう風にタイルを配置するパズルって
最適なタイル数とかあるんですか?

制約としては、
各工場は中央倉庫と道路でつながっていないといけません。
879132人目の素数さん:2012/09/22(土) 00:46:42.91
どれが工場、中央倉庫かわからんけどw
どうみてもナップザック問題
880132人目の素数さん:2012/09/22(土) 08:54:50.28
初歩的な問題ですがちょっとコンセプトが分からないのでお願いします。

数列 x_n = 1/n の集積点を全て求めよ
881132人目の素数さん:2012/09/22(土) 09:47:16.57
(x,y)=((u,v)+(s,t))/2
((u,v)-(s,t))((x,y)-(0,b))=0
((u,v)-(s,t))(((u,v)+(s,t))/2-(0,b))=0
((u,v)-(s,t))((u,v-2b)+(s,t))=0
(s,t)^2+(0,-2b)(s,t)-(u,v)(u,v-2b)=0
(s^2,t^2-2bt)=(u^2,v^2-2bv)
(s,t)=(+/-u,b+/-(b^2+v^2-2bv)^.5)
=(+/-u,b+/-(v-b))
=(+/-u,+/-v+b-/+b)
882132人目の素数さん:2012/09/22(土) 09:47:23.69
>>880
集積点と予想される点が本当に集積点になっていることと、
それ以外の点が集積点でないことを示す。
883132人目の素数さん:2012/09/22(土) 09:56:23.70
0いがいになにがある。 qed
884132人目の素数さん:2012/09/22(土) 10:02:40.76
アキミュレーションポイントは近傍で無限に数列があるところ。
0以外だと近傍を十分小さくとれば、有理数の稠密せいから、それでも
無限に数列がある。でも、これはオイラー数列だから、r近傍だと、1/m-1/n>r
だと、それ以上ないから、可算数列なので、有限個しかない。
トリビアだからqed
885132人目の素数さん:2012/09/22(土) 10:56:00.89
集合{R線形写像C→C}の次元が4であるのはなぜでしょうか
教えてください
886132人目の素数さん:2012/09/22(土) 11:49:10.33
>>880
0以外が集積点だとしてその点をpと置いてpのp/2の近傍をとると
そこに有限個しか点が入ってないから集積してません
ですが自明で十分でしょう。
887885:2012/09/22(土) 12:03:30.01
自己解決しましたので結構です
888132人目の素数さん:2012/09/22(土) 12:24:47.17
>>879
つまりどうやったら解けるんだってばよ!
889132人目の素数さん:2012/09/22(土) 14:05:23.35
>>880
密着位相なら全ての点が集積点
890132人目の素数さん:2012/09/22(土) 14:06:56.57
>>880
離散位相なら集積点はない
891132人目の素数さん:2012/09/22(土) 14:08:51.37
>>888
しらみつぶし
892132人目の素数さん:2012/09/22(土) 18:15:49.89
すいません、イデアルの基本的なところがあやふやになってしまったので質問します。
イデアルIについてそのべき乗I^2というのは単に各元を2乗していった集合ではなく、I*Iを含むという意味ということで、合っているでしょうか?
整数環Zにおいて2Zを2乗すると4Zですか?
893132人目の素数さん:2012/09/22(土) 18:42:53.77
環 R の部分集合 I が、加法群としての部分群
894132人目の素数さん:2012/09/22(土) 18:45:59.16
I+I^=I
895132人目の素数さん:2012/09/22(土) 21:38:11.96
>>892
そのとおり
896132人目の素数さん:2012/09/22(土) 21:51:37.97
お願いします。
成功の確率がr(0<r<1)のゲームを何回か繰り返す。はじめ9枚以下のコインを
持っていて、各ゲームごとに成功したらコインを一枚もらい、失敗したらコイン
を一枚わたす。
持っているコインが10枚になるか、無くなったらゲームをやめる。
n枚のコインから始めて、コインが10枚になる前に無くなる確率をP(n)(0≦n≦10)で表す
ただしp(0)=1,p(10)=0とする。

問題
p(n+1),p(n),p(n−1)の関係式を求めよ。ただし1≦n≦9とする。
897132人目の素数さん:2012/09/22(土) 21:54:04.21
続き
答えはp(n)=rp(n+1)+(1−r)p(n−1)です
解答はn枚からはじめて10枚になる前にゲームが終了する確率p(n)
は一回目に勝つとき、得点はn+1になる
よってrp(n+1)
一回目に負けるとき得点はn−1になる
よって(1−r)p(n−1)
以上よりp(n)=rp(n+1)+(1−r)p(n−1)
自分はn枚からゲームを始めて10枚になる前にゲームが終了する確率p(n)が
、何故n+1枚からゲームを始めて負ける確率に勝率rをかけて表されるかが分かりません。
連投すみません

898132人目の素数さん:2012/09/22(土) 22:02:05.80
ひどいマルチですwwwwwwwww
回答不用wwwwwww
899132人目の素数さん:2012/09/22(土) 22:05:40.92
>>893
解答がつかないから別のスレでも書き込みしました
すみません
900132人目の素数さん:2012/09/22(土) 22:10:06.44
たかだか2,30分程度でw
901132人目の素数さん:2012/09/22(土) 22:16:25.11
>>900
ゆとりがないゆとりですみませんw
解いてくれる人が都合よくいるだろうと思ってました・・・・
902132人目の素数さん:2012/09/22(土) 22:36:36.22
n枚の状態から移れる次の状態は何かを考えれば、明らか。
903132人目の素数さん:2012/09/22(土) 23:12:17.16
>>902
n-1枚やn+1枚になるってことですね
それが理解できないのでとにかく自分でやってみます
ベタな確率漸化式の問題だと、n+1回目の確率を記述するためにn回目の確率が
用いられるのは納得できるんです。n回の状態から移れる次の状態は何かを考えれば。
今回の問題はn枚から始まったゲームが0になって終わる確率が、なぜn+1枚から始まって終わる確率
になるかが・・・
書いてたら分かりました。ありがとうございました
904132人目の素数さん:2012/09/22(土) 23:26:36.78
>>901
多くの回答者は複数のスレを見てるから、複数のスレに書き込んでも回答者の目に入りやすさは変わらないんだ
逆に同じ書き込みを見せられると回答する気が無くなるんだな
905132人目の素数さん:2012/09/22(土) 23:44:43.92
>>903
多分p(n)の定義を誤解している。
どういう確率をあらわしていると思っているかを、自分の言葉でここに書いてみな。
906132人目の素数さん:2012/09/22(土) 23:48:36.44
どうでもいいいけど、

くだらなねー高校レベルの確率の問題は別すれでやってくれ
907132人目の素数さん:2012/09/22(土) 23:56:23.45
マルチはいけない理由は何かあったはず
何だったのかは知らない
けど理由はあった
今はその理由が忘れ去られて形としての慣習だけが残っているだけだから
マルチしても別にいいんだよ
908132人目の素数さん:2012/09/23(日) 00:02:14.79
至急教えて下さい!!

ア〜シまでお願いします。

http://cdn.uploda.cc/img/img505dd14bdb457.jpg
909132人目の素数さん:2012/09/23(日) 00:04:23.67
>>904
すみませんでした。以後気をつけます

>>905
p(n)はn枚からゲームを始めて、持っているコインが10枚になる前に
無くなる(コインが0になる)確率、です。
やっぱり分かりませんでした。
910132人目の素数さん:2012/09/23(日) 00:12:56.07
状態の遷移というものを考えるのだよ。
n枚から始めて10枚になる前に0枚になってしまうのは
最初の一回のゲームでn+1枚になってそれからゲームを続けて10枚になる前に0枚になるか
或いは
最初の一回のゲームでn-1枚になってそれからゲームを続けて10枚になる前に0枚になるか
のどちらかだろ。
これで理解できないのなら、諦めろ。
911132人目の素数さん:2012/09/23(日) 00:14:30.06
>>907
おまえが答えてやれよ
912132人目の素数さん:2012/09/23(日) 00:44:35.81
マルチってことは気がつかない所にもマルチしてるってことだ
回答しても無駄になる可能性が数十倍もあったら無駄やる人がどれだけいるか
それを承知でマルチするなら好きにしな
913132人目の素数さん:2012/09/23(日) 00:47:32.47
マルチの指摘は気づかずに無駄回答する人へのサービス
914132人目の素数さん:2012/09/23(日) 00:51:11.92
教えたがりが解答させていただくスレを作ればいいんじゃね。
915132人目の素数さん:2012/09/23(日) 00:59:46.41
教えたがりではなくて復習なんだよ
勉強したことを確認するために書いていたりもする、たぶん
916132人目の素数さん:2012/09/23(日) 01:08:59.51
"マルチポスト いけない 理由"ででもぐぐればいいのにしないとか
確実に釣り針だろうが仕方がない

http://www.ml-info.com/weekly/archives/2009/091024o.html
http://ja.wikipedia.org/wiki/マルチポスト#.E3.83.9E.E3.83.8A.E3.83.BC.E9.81.95.E5.8F.8D.E3.81.AE.E7.90.86.E7.94.B1

知らなかったり忘れたりしたやつは今一度読んどけ
知ってて無視する奴はくたばればいいのに
917132人目の素数さん:2012/09/23(日) 08:40:54.55
http://images1.wikia.nocookie.net/anno2070/images/e/eb/Plot_512.png

狂気を感じるほど洗練されたレイアウト

今のおれなら将棋の羽生名人の気持ちがわかる

直感で作る、ピンときた瞬間に編み出される
神のレイアウト

いまのおれならABC予想も解ける
918132人目の素数さん:2012/09/23(日) 08:55:39.72
フェルマーの定理の次はABC予想か‥‥
919132人目の素数さん:2012/09/23(日) 11:30:19.97
半径rの円の中に収まる長方形で
面積が最大の物の縦と横の長さを求めよ。

これって高校の知識で解けますか?
よろしくおねがいします。

S = x * y = x * root {r^2 - x^2}

f(x) = r^2 - x^2 と置くと

S = x * f

S' = f(x) + x f ' = ...

わからん。
920132人目の素数さん:2012/09/23(日) 11:45:33.62
対角線が直径だから
ab/π(.5(a^2+b^2)^.5)^2
が最大
4/π((a/b)+(b/a))
4/π(x+1/x)
d(x+1/x)/dx=1-1/x^2=0,x=+/-1
a/b=1
a=b
921132人目の素数さん:2012/09/23(日) 11:53:08.44
p(n)=(1-r)p(n+1)+rp(n−1)
922132人目の素数さん:2012/09/23(日) 12:03:37.69
xp(n)x^n=(1-r)p(n+1)x^(n+1)+x^2rp(n−1)x^(n-1)
x(f-f0)=(1-r)(f-f1-f0)+x^2rf
f(x-(1-r)-x^2r)=xf0-(1-r)(f1+f0)
f=(xf0-(1-r)(f1+f0))/(-x^2r+x-(1-r))
=(x-(1-r)(f1+1))/(-x^2r+x-(1-r))
923132人目の素数さん:2012/09/23(日) 15:14:24.62
足算+における"原子"は1です。掛算*における原子は素数2,3,5,…です。
自然数は1と+によって生成されるのでした。しかし、自然数は素数と*によっても生成されるのでした。
我々のよく知る演算、+と*は測量のために生み出されたもので、直感的にも自然なものです。
足算から掛算への発展は直感的なのにもかかわらず、掛算の原子は足算のものとは比較にならないほど複雑さを増します。
924132人目の素数さん:2012/09/23(日) 15:17:49.46
足算のものとは比較にならないほど掛算の原子が複雑なのはどうしてですか?
925132人目の素数さん:2012/09/23(日) 17:09:31.20
しかしどのような複雑な掛け算の原子ですら
足し算によって簡単に構成できることがわかっている
926132人目の素数さん:2012/09/23(日) 17:18:15.65
>>919
初等幾何で解ける
長方形の対角線が直径だから半分の三角形で考えれば
底辺が直径で固定され底辺の中点と頂点の距離が半径の三角形を考えれば良い
面積最大は高さ最大だから頂点までの距離最大すなわち頂点は底辺の垂直2等分線上
927132人目の素数さん:2012/09/23(日) 17:26:53.48
928132人目の素数さん:2012/09/23(日) 17:42:23.35
>>919
>>919 さんの〈試み〉を修正すれば,次のようになります.

半径rの円に内接する長方形の2辺の長さを 2x,2y とおくと
x^2+y^2=r^2 が成り立ち,長方形の面積 S は
S=4xy=4x √(r^2-x^2)=4√(r^2 x^2-x^4)
と表される.f(x)=r^2 x^2-x^4 とおけば
f(x)=-(x^2-(r^2)/2)^2+(r^4)/4 となるので,
f(x) は x=r/√(2) のとき最大値 (r^4)/4 をとる.
つまり,S は x=y=r/√(2) のとき最大値 2(r^2) をとる.

x=r cos(θ), y=r sin(θ) (0<θ<π/2) とおいて,
S=4xy=4(r^2)cos(θ)sin(θ)=2(r^2)sin(2θ)
から結論を導くこともできます.
929132人目の素数さん:2012/09/23(日) 18:16:29.36
>> 920>>926 >>928
あざーっすy!!!
930132人目の素数さん:2012/09/23(日) 18:19:08.63
a x + b y + c z
931132人目の素数さん:2012/09/23(日) 18:48:02.95
>>907
今さらだが、自分が理由を忘れたから別にいいんだよてのは
リアルな社会では馬鹿扱いされるならまだ優しい扱い。
932132人目の素数さん:2012/09/23(日) 19:27:25.61
ある店に1回行くと何回かクジを引かせてもらえ、1回引くと20%の確率で当たりが出てアメ玉を1個貰えます
クジは1回目は無条件で引けますが、2回目以降も引けるかどうかは抽選になってあり、その当選確率は90%です
ただし、クジ引ける回数は最大で5回までとなっています
さて、その店に1回行ったらアメ玉を何個貰える事が期待できると言えるでしょうか?


↑と言う問題を解こうとしているのですが、最大5回の試行の各回を実施できる確率は、
100% / 90% / 81% / 72.9% / 65.61%で
各回でアメ玉の「当たらない」確率は80%なので、その店に行って1個も貰えない確率は、
(1 * 0.8) * (0.9 * 0.8) * (0.81 * 0.8) * (0.729 * 0.8) * (0.6561 * 0.8) ≒ 0.114255 ≒ 11.43%
この余事象である (1 - 0.114255) = 0.885745 ≒ 88.57%が意味するものは、
「その店に1回行ったときにアメ玉を最低1個は貰える確率」

ここまでは解るのですが、「1回行ったときに貰えるであろうアメ玉の数」、つまりは個数の期待値がどうも解りません
最終的に「○○個」という答えを導き出すには、どういった計算になるのでしょうか?
933132人目の素数さん:2012/09/23(日) 19:49:05.67
↑と聞いておいてあれですが、その店に1回行くと貰えるアメ玉の数は最大5個になるので、
5 * 0.8857 ≒ 4.43
で、答えは「4.43個」とすれば良いのでしょうか?
934132人目の素数さん:2012/09/23(日) 20:00:04.91
高校生すれで聞けよ

しょうもねー確率の問題は
935132人目の素数さん:2012/09/23(日) 20:00:05.17
(.2a+.8b)((.2a+.8b)(.9))^4
936132人目の素数さん:2012/09/23(日) 20:03:24.82
複素数体上の非特異射影代数多様体について、任意のホッジ類は、代数的サイクルの類の有理数係数の線形結合である。
937132人目の素数さん:2012/09/23(日) 20:06:17.01
すばらしい用語をいっぱいしってるね

えらいby慶応のおばちゃん
938132人目の素数さん:2012/09/23(日) 20:09:07.68
パイオニア・アノマリー
何が惑星探査機パイオニア 10 号と 11 号の明らかに説明のつかない太陽方向への加速を引き起こしているのか?[1]
939132人目の素数さん:2012/09/23(日) 21:15:24.35
ほいさっ!ほいさっ!
940132人目の素数さん:2012/09/23(日) 21:40:38.62
共分散行列の求め方なんですが
http://www.itl.nist.gov/div898/handbook/pmc/section5/pmc541.htm

http://stattrek.com/matrix-algebra/covariance-matrix.aspx
で分母がn-1とnとで違うのは何故でしょうか

手元にある本はnを使ってるようですがWolfram|Alphaやnumpyはn-1を使っているようで
答えが一致しなくて困っています
941132人目の素数さん:2012/09/23(日) 22:19:52.74
>>940
標本分散と母集団の分散とのちがいでは?
942132人目の素数さん:2012/09/23(日) 22:45:59.28
確かに上は不偏(unbias)分散共分散行列って書いてありますよね
numpyの共分散行列を求める関数にもbiasを設定するオプションがあって答えも合いました
MATLABもnが1の時以外は自動的にn-1で割るらしいし
統計関数では母集団を扱うことが珍しいってことですかね
ありがとうございました
943132人目の素数さん:2012/09/24(月) 00:16:22.64
>>942
母分散のほうが n-1 ですよ
944132人目の素数さん:2012/09/24(月) 00:36:07.48
え?標本を使うから不偏分散を使うのでは?
945132人目の素数さん:2012/09/24(月) 00:36:22.51
n-1 の 1 は自由度
自由度 k なら n-k
分散計算に使う基準値が k 次元なら自由度 k
基準値が 1 次元の平均値なら自由度 1
k 個の未知係数を持つ k-1 次式を引く場合などが自由度 k
946132人目の素数さん:2012/09/24(月) 00:37:16.74
分散計算に使う基準値とは?
947132人目の素数さん:2012/09/24(月) 02:00:25.39
>>923
素数って掛け算というパズルの
最小のピースなんだな、初めて知ったわ。

こういうのって高校で教えないの?

定義をみたけど、

「1とそれ自身でしか割り切れない数」

ってわかりにくいわw
もっと直観的にわかる表現にかきかえろよ。
948132人目の素数さん:2012/09/24(月) 02:58:38.97
>>947
素因数分解の一意性って習わなかったの?
949132人目の素数さん:2012/09/24(月) 03:10:09.60
ピチピチに張ったタイツやレオタードの一点をつまんで引っ張ると、山のようなカーブになりますが、あれは数学的にはなんという曲線なんですか?
弾性力によるものなので、単振動の方程式やカテナリーと同じく、指数関数の結合で表されるのでしょうか?
950132人目の素数さん:2012/09/24(月) 03:26:02.74
>>916
それ違うよ
形骸化して慣習化したものに適当な理由を付けただけだよ
精神論しか書いてないだろ、コミュニティーとか付いていけないしな

>>931
システムの都合でマルチで質問するとおかしくなるってのを聞いた
今はそんなシステムないのに、マルチがダメとか無意味なことやっているだけ
あとは勘違いした後続の人がデマを拡散しているだけだよ

あとは運営の都合を、利用者が代弁したりとかだ
951132人目の素数さん:2012/09/24(月) 03:38:14.73
>>948
習ったかどうかはよく覚えてない。
足し算引き算の基本ピースと掛け算割り算の基本ピースなんて考えたことないし。

学校の先生が次のように
説明してくれてたらもっと数学の偏差値上がってたと思う。


足し算の基本ピースを数字の1だとすると、
掛け算の世界でちょうどこれに相当するのが「素数たち」なんです、って。
952132人目の素数さん:2012/09/24(月) 03:49:14.74
線型代数でいうと、素数が基底、指数が係数に相当するんだな。
指数の範囲を整数に広げて、1を零元とみれば線型代数のように扱えるのだな。

0*2+0*3+0*5+…=1 (零元)
1*2+1*3+0*5+…=6

のように
953132人目の素数さん:2012/09/24(月) 04:08:06.03
足し算の反復が掛け算で、掛け算の反復が累乗
累乗は交換法則が成り立たないので、累乗は反復できない
954132人目の素数さん:2012/09/24(月) 04:37:00.10
>>950
そんなことはどうでもいい
無駄な事をさせられるのが嫌いだから無視するだけだ
当然、他人にも無駄になると警告する
後は当人たちの自由
955132人目の素数さん:2012/09/24(月) 06:20:09.73
マルチポストが嫌いなら無視すれば良い
マルチポストする奴が嫌いなら、回答を複数スレに分散して全部読まないと分からないようにしてやれ。
956132人目の素数さん:2012/09/24(月) 07:39:08.70
>>948
素数の定義は >>947 のとおり。素因数分解の一意性はただちにはわからない
それでも素因数分解の一意性を主張したければ証明しろ
957132人目の素数さん:2012/09/24(月) 08:36:40.87
>>956
文系脳にサポートする文系脳
958132人目の素数さん:2012/09/24(月) 08:44:17.37
習うかどうかとただちに分かるかどうかは別だろ
959132人目の素数さん:2012/09/24(月) 08:48:02.06
素因数分解の一意性は習うんじゃねえか?
受験の時には証明無しで使ってよかったと思うが。
2通り以上に分解出来るとすると矛盾することを示せばいいだけだけど。
960132人目の素数さん:2012/09/24(月) 09:17:47.22
分解可能性も示せよ
961132人目の素数さん:2012/09/24(月) 09:33:28.97
受験生は落ちろ

じゃなくて

馬鹿高校生すれへ行け
962132人目の素数さん:2012/09/24(月) 09:35:29.13
>>959
>2通り以上に分解出来るとすると矛盾することを示せばいいだけだけど。
「いいだけ」って,何も分かっていないな?
963132人目の素数さん:2012/09/24(月) 09:38:21.86
>>960
一意性って書いてあるだろ
日本語読めないのか?ゴミくずが^^;
964132人目の素数さん:2012/09/24(月) 09:42:22.83
え、「素因数分解の一意性」って言ったら、ふつうは「素因数分解できること」と「分解の仕方が一意的であること」を示すんじゃないの?
まあ、言葉の問題だから、分かってるならいいんだけどw
965132人目の素数さん:2012/09/24(月) 09:44:10.38
947はどうした?
966132人目の素数さん:2012/09/24(月) 09:45:23.93
分解可能性は自明じゃないのか。小さい素数から割っていけば
有限回の操作で分解できるだろ。
967132人目の素数さん:2012/09/24(月) 09:46:33.96
馬鹿登場
968132人目の素数さん:2012/09/24(月) 10:41:05.90
>>966
自然数ならな。
でも、似たような体系で「最小値が存在することが保証されない」体系も存在するし、
そういう体型ではその理屈は成り立たない。
969132人目の素数さん:2012/09/24(月) 10:50:48.71
おれは受験生じゃねぇよ

ち、ちなみに神戸卒TOEIC700です… (; '‘ω‘`)
970132人目の素数さん:2012/09/24(月) 10:56:25.96
神戸、そしてひとつが…

神戸どうやって卒業するんだ

三宮でわるさしていられなくなったか?
971132人目の素数さん:2012/09/24(月) 11:04:24.63
>>969
ユークリッド整域ってことだ。わかったか?
972132人目の素数さん:2012/09/24(月) 11:16:12.53
>>968
> 自然数ならな。
そういう前提からいちいち毎回全部書かなきゃダメなのかよ。こんな掲示板でw
973132人目の素数さん:2012/09/24(月) 11:21:54.69
問題
5枚の札をもつポーカーの手に、10以上の札が少なくとも3枚あるとして
7以下の札が全く無い確率を求めよ■

解答
Aを10以上の札がすく無くとも3枚ある事象、Bを7以下の札が全く無い事象とする。
P(A)=Σ[(i={3, 4 ,5} ] [16_C_i × 36_C_(i-5)] / 52_C_5
P(A?B)=Σ[(i={3, 4 ,5} ] [16_C_i × 8_C_(i-5) ]/ 52_C_5
P(B|A)=P(A?B)/P(A)=103/1258■

になったんですけど答えを見ると161/1456になってました。
どこが間違っているのか分かりません。
おしえてください。
974132人目の素数さん:2012/09/24(月) 11:54:37.14
?は共通部分です。
シグマの添え字のイコールは含む記号の代わりです。
975132人目の素数さん:2012/09/24(月) 11:55:39.11
神戸大にきまってんだろ、言わせんな
976132人目の素数さん:2012/09/24(月) 12:08:55.83
馬鹿に限って意気込む
977132人目の素数さん:2012/09/24(月) 12:19:53.61
微分に挑戦中なのですが、f(x)=x^2+2/x^3-5が解けません・・・。
どなたか教えて下さい!
978132人目の素数さん:2012/09/24(月) 12:22:25.14
まんどくせー

(x^n)' = nx^(n-1) (nは0でないとき)

を知らないの?
979132人目の素数さん:2012/09/24(月) 12:26:45.80
>>977
まるちすんな
980132人目の素数さん:2012/09/24(月) 12:27:23.66
>>950
> システムの都合でマルチで質問するとおかしくなるってのを聞いた

予断に基づいてルール無視すんなゴミクズ
981132人目の素数さん:2012/09/24(月) 12:40:44.79
>>962
分解可能性を証明できれば、あとはお前のいうとおりだ。
「素数をpとして積ab が p で割り切れるなら、a がp で割り切れるか b が p で割り切れるかのいずれかである」をどう証明するか?

仮に p が 6 とすると(当然素数ではないから)、a=9, b = 8 として ab は p でわりきれても、a も b も p では割り切れない。
素数だったらうまくいくのはなぜか?これは自明ではないだろう。
982132人目の素数さん:2012/09/24(月) 12:41:21.04
>>978
答えてくれてありがとう。
私は文系で、微分自体初めてに近くて。。。
983132人目の素数さん:2012/09/24(月) 12:41:53.09
>>963,964
一意性の証明の前提として分解可能性がある。
貴様のいう一意性の証明を出してみろ。分解可能性が必要であることを指摘してやろう
984132人目の素数さん:2012/09/24(月) 12:42:48.87
>>966
自明ではない。素数の定義は「約数を二つしかもたないこと」。
これと割り切れることとは直接にはなんのつながりもない。
985132人目の素数さん:2012/09/24(月) 12:51:28.98
横レスだけど、
自身より小さい素数で割り切れなければ自分自身が素数である
と素数の定義から明らかでしょ
今は有理整数の素元分解に限定して話をしているんだから、これだけでいいんだよ
986132人目の素数さん:2012/09/24(月) 12:57:19.65
>>973
をお願いします。
987986:2012/09/24(月) 13:25:33.54
もういいです。
他のスレで聞きます。
さようなら。
988132人目の素数さん:2012/09/24(月) 13:27:21.51
>>984
割り算のアルゴリズムを認めれば自明だよね。
それともユークリッド聖域性から説き起こせと?
989132人目の素数さん:2012/09/24(月) 13:33:00.11
一意分解性整域、すなわちUDFなら一意性があるだろ。
990132人目の素数さん:2012/09/24(月) 13:33:23.70
我々の聖域で手をだしたものは消えてもらおう

原子力村
991132人目の素数さん:2012/09/24(月) 13:41:00.44
どこまでが「自明」なのかは人によって違うからなぁ
○○は自明だ、自明じゃない、の議論は不毛
992132人目の素数さん:2012/09/24(月) 13:42:51.98
ユークリッド整域であることよりも整礎性の方が基本的だし、今の場合は証明にとって直接的でもある
有理整数の分解可能性が自明でないと言っている人は、代数学の教科書に載ってるUFD関連の記述に拘泥してるんでないかな
993132人目の素数さん:2012/09/24(月) 13:43:00.32
整数の話をわざわざ代数の話にする馬鹿
994132人目の素数さん:2012/09/24(月) 13:44:59.08
訂正
整礎性 → 整列集合であること
995132人目の素数さん:2012/09/24(月) 13:47:16.49
存在することを前提として一意性を言うんだから
前者が後者に必要なことはそれこそ自明だろ、阿呆か
996132人目の素数さん:2012/09/24(月) 13:50:05.97
というか、大抵、分解の可能性よりも一意性の方だけが重要なので、可能性についてはいちいち言及しないことが多い
997132人目の素数さん:2012/09/24(月) 13:54:24.01
微分方程式なんかでは存在も重要じゃね?
998132人目の素数さん:2012/09/24(月) 14:07:54.12
>>987
気が短いwww
999132人目の素数さん:2012/09/24(月) 14:08:54.98
質問スレと勘違いしてる奴多いよね
1000132人目の素数さん:2012/09/24(月) 14:09:06.60
1000getしたらabel prize
10011001
このスレッドは1000を超えました。
もう書けないので、新しいスレッドを立ててくださいです。。。